1st series [1] [2] [3] [4] [5] [6] [7] [8] [9] [10] [11] [12] [13] [14] [15] [16] [17] [18] [19] [20] [21] [22] [23] [24]  2nd series [1] [2] [3] [4] [5] [6] [7] [8] [9] [10] [11] [12] [13] [14] [15] [16] [17] [18] [19] [20] [21] [22] [23] [24] [25] [26] [27] [28] [29] [30] [31] [32] [33] [34] [35] [36] [37] [38] [39] [40] [41] [42] [43] [44] [45] [46] [47] [48] [49]

  View the latest questions and answers at askaphilosopher.org

Ask a Philosopher: Questions and Answers 24 (1st series)

Here are some of the questions that you Asked a Philosopher from December 2003 — March 2004:

  1. Spiritual life of Nietzsche's 'higher man'
  2. Why we adore pop stars
  3. The speed of darkness
  4. A philosopher for all people and all time
  5. Social justice for the starving in India
  6. How Christians should respond to racism
  7. Testing metaphysical laws
  8. Nietzsche in Kundera's Unbearable Lightness of Being
  9. Crucifixion and impaling in Plato's Republic
  10. Acknowledging creation and evolution
  11. Validity of religious experiences
  12. Strengths of the design argument
  13. Why Descartes burned Le Monde
  14. Why Descartes believed that animals have no souls
  15. On the difficulty of measuring the temperature of the earth's core
  16. Imagining other universes
  17. Adam, Eve and monkeys
  18. Should I take Philosophy?
  19. Why body and movement are necessary for self-consciousness
  20. Error in literal interpretations of Christian scripture
  21. Searching for the first philosophical Greek saying
  22. The sound of the Divine Word
  23. Tip for studying maths
  24. Where to find out what Ryle thought of Descartes
  25. The philosophical approach to Reiki
  26. The real reality
  27. A little reminder about time
  28. Fundamental properties of mind and matter
  29. God and the Myth of Sysiphus
  30. Where to find a 'logical' proof of God's existence
  31. Is it selfish to enjoy helping others?
  32. The right to have children
  33. Two puzzles about pain
  34. Some assorted questions
  35. Negative views on atheism
  36. Quantum mechanics and counterfactuals
  37. On a supposed proof of absolute motion
  38. Has anyone defined infinity?
  39. Readings on changes in self-definition
  40. How Plato is connected to the Caribbean
  41. Philosophy of list making
  42. A lesson in comparative religion
  43. Tell me how I can disagree with Descartes
  44. Homosexuality and Kant's Categorical Imperative
  45. "Krsna is God because Krsna is God"
  46. Philosopher on the docks of San Francisco
  47. Reading Nozick's Love's Bond
  48. Nameless love
  49. Philosophical advice to a lonely heart
  50. The bottomless cup
  51. Forgotten dreams and forgotten lives
  52. Tycho's model of the universe
  53. "I am the Pope"
  54. Nietzsche's case for the death of God
  55. Three questions on Emmanuel Levinas
  56. Philosophical advice for facing adversity
  57. A lesson in assertiveness
  58. Doubts about reincarnation and Von Daniken
  59. Where the universe ends
  60. Defining when a plant dies
  61. Relational vs absolute views of time
  62. In defence of meat
  63. Popper's concept of falsification
  64. The Holocaust as a problem for philosophy
  65. Thought experiment of human bisection
  66. Pirsig's metaphysics of Quality
  67. Hell as the fifth dimension
  68. Why we like our jeans blue
  69. Freewill vs determinism revisited
  70. Why two qualitatively identical individuals are still two
  71. Empty space and empty time
  72. "I want to live on another planet"
  73. Aristotle for children
  74. A definition of value
  75. The ship of Theseus
  76. Philosophical books to change your life
  77. Impossibility of proving an empirical generalization
  78. Ant's view of eternity
  79. Zeno's paradoxes of motion
  80. Kundera's Unbearable Lightness of Being
  81. Where "Liberty" comes from
  82. Is the value of human life diminishing?
  83. On the evolution of moral codes
  84. Theory about the meaning of life
  85. What is the answer to this question?
  86. Why people are not content with what they have
  87. The importance of philosophy
  88. Philosophy for Children — two useful texts
  89. Freedom of religion
  90. "How do I know you will not kill me in my sleep?"
  91. Cancer and AIDS as God's method of birth control
  92. On Karl Marx's childhood influences
  93. Fundamentalism
  94. Ethics of killing as a numbers game
  95. Questions about concepts and infinite series
  96. "God does not play dice with the universe"
  97. Weakness of the ontological argument
  98. Hobbes vs Locke
  99. Liberal democracy, rights, and peace
  100. Mind, brain and soul
  101. Reading on free will
  102. Nature of psychological theory
  103. Effectiveness and efficiency in management
  104. Pierce on philosophical soup kitchens
  105. Philosophers for breakfast
  106. Is the infinite an absolute or relative truth?
  107. Sartre's philosophy of boredom
  108. Philosophy for developing countries
  109. Puzzle about the expanding universe
  110. Earnest reflections in a school library
  111. How to recover from existential angst
  112. The career of emotivism
  113. What a mathematical proof proves
  114. John Rawls on justice
  115. Skinner's behaviourism on trial
  116. Materialism, interactionism and epiphenomenalism
  117. Chicken and egg
  118. Advice to an amorous 15 year old
  119. Justification for military conscription
  120. The practical proof of moral laws
  121. Consolation for flat earthers
  122. The meaning of "meaning"
  123. The sixth sensation
  124. Why it's wrong to lie
  125. Philosophies of ceramics
  126. Fate, chance and the turtle dropping scenario
  127. When is a towel wet?
  128. References on wave/particle duality
  129. Definition of reality and the existence of Satan
  130. Reading for a philosophically inquisitive 13 year old
  131. Hannah Arendt on "action" and "work"
  132. Epicurean quote on the problem of evil
  133. Why ethical relativists are intolerant
  134. What is history?
  135. Is there life after death?
  136. Theist vs atheist views of how the universe exists
  137. Do scientistic experiments create effects or discover them?
  138. Prize winning argument for the existence of God
  139. Klempner on "God" and "I"

Ed asked:

What did Nietzsche mean by the word "spiritual", used to describe the higher man?

Following Nietzsche's proclamation of the 'death of God', anything smelling of 'other-worldly', metaphysics like the soul, an afterlife, or a morality distinct from this worldly life is rejected. This world is all there is. 'Spirituality' is to be understood in this context. In answering the question I will firstly provide an overview of Nietzsche's account of how the civilised human being is created. Secondly, I will describe what spiritual means in relation to the higher man.

No longer the Crown of Creation

Nietzsche characterises reality as ontologically constituted by power (macht). Directed claims of power or Will to Power (wille zur macht) are manifested in instincts, drives and affects which are the fabric of all living phenomena. These are subject to a stratified 'development' and 'ramification'. Not being of divine, other-worldly origin but as part of nature, the human being is the product of such a development and ramification of the more primitive manifestations of the will to power. Nietzsche expounds how these instincts, drives and affects are developed to create the human subject in On the Genealogy of Morals [1887] especially in the second essay.

Primitive human beings acted blindly, following the healthy, regulatory instincts [such as procreation and nourishment] and the active 'forgetting' of physiological processes of the organs. Juxtaposed to this active forgetfulness, an active memory is bred. This is achieved by mnemo-techniques — utilisation pain through cruelty to create a memory based on pain. The prospect of pain inhibits the actions of certain drives/actions whilst permitting others. Thereby, an interiority of the human is created providing awareness of oneself, others and the world.

The facilitating of this awareness of oneself and ones actions enables the onset of the phenomenon that is called 'willing'. Here, thinking causally, anticipation of future events, the understanding of ends and means and the ability to calculate are inculcated into the human animal. Thereby, it becomes regular, calculable, and capable of making and keeping promises. It becomes responsible.

In accordance with certain tables of values or narratives, these mnemo-techniques burn into human beings the inhibition and repression of certain wild instincts and the expression of others. Upon this, there follows values, moral systems, ways of acting/not acting that make the human animal into the 'civilised' human being. The narratives are definite perspectives and valuations arising from the definite powers and institutions that enforce them. Powers, institutions such as the State, the Christian Church and its offshoots in modernity inculcate definite perspectives, valuations of the human instincts, drives and affects. Thus the creation of material into what we call the human being.

The Higher Man and Spirit

Drives are not only inhibited they are also spiritualised. As stated above, they are subject to stratified development and ramification. Their primitive expression is redirected into a more sophisticated form. The barbarian instincts of conquest can be spiritualised into the rarefied publications of the academic. Violent instincts and drives can be spiritualised into the performance of Olympian competition and games. Thus although sexual drives are condemned and their extirpation advocated by the Christian Church, Nietzsche heralds the post-Christian 'spiritualization' of sexuality called 'love'.

The spiritualisation of drives can occur as part of the general 'socialisation' process of narratives. Think for instance, of the primordial hunger of an infant for milk spiritualised into the aesthetic appreciation of food by the adult restaurant critic. Or, drives can be overcome and spiritualised individually by what Nietzsche variously calls the higher man, or the free man, or the Over man (Uber-menschen). The latter has wrongly been monopolised in translation as the 'super man' — allowing connotations of arrogant elitism and crypto-fascism.

Concerning the individual, the failure to withstand the demands of a strong drive is termed decadence by Nietzsche. This is defined as the inability to resist the stimulus of a drive. Here, where one drive acts as the tyrant to the exclusion of and disrupting the healthy economy of all other drives, it drags its victim down with the weight of its folly. Think of addictions and obsessions of various sorts. The opposite is the case with the Higher man.

The freedom and strength of an individual is measured by his/her ability to resist, regulate and re-employ or spiritualise those strong drives. The higher man can incorporate his experiences and thereby regulate his drives. To learn from all the experiences of life and affirm them characterises the most spiritual human beings. Perhaps Nietzsche incorporated and overcome all his experiences, his illness, his loneliness, his failed love affairs and spiritualised them into his philosophy?

Spirit, is translated from the German by Walter Kaufman as 'neither spirit alone nor intellect alone but also mind, wit and espirit'. It is not a separate mind to the body but expressive of the body. It is the reality of being one has, to understand the economy of one's drives and how they underpin what one does. It permits among other things, to know how one becomes who one is. This understanding allows the higher man of spirit to regulate his drives and actions. It also allows him/her to understand how one acted or responded to events in the past, depended on the then constitution of the drives: one could not have done any different.

In affirming on both accounts, one gives style to ones character. In affirming, even from the abyss, one is affirming the whole universe and one's part in it. Enter the eternal recurrence.

Martin Jenkins

back

Lesley asked:

Why do we adore pop-stars or celebrities or in fact any "great" person, although we know that they can not be as "perfect" or as "ideal" as we perceive them to be?

In general, what is this thing about being fascinated with certain things (like the female form, or a very expensive car) and then seeing them just as flesh (like a medical student will see) or like a manufactured good made up of engineering hours, and sheets of metal? (In this sense there seems to be two basic views to many things around us).

Which view should I hold/stick to while viewing things? One makes me feel imprisoned to being fooled by images (taking them ideal) the other makes me lose my desire to approach them.

I think you should stick with being fooled by idealisations — up to a point. A certain level of idealisation is normal.

We just don't normally, as a matter of fact, think of others in their full reality, as defecating and masturbating and things. I don't think that this is wrong. It enables us to look at another person without laughing or feeling revolted. Very few people are prepared to express their revulsion at the human body and other people, but some philosophers and writers have done this.

Such writers are dwelling overly much on realities that we normally ignore — but they are doing this in contemplative writing and this doesn't reflect on how they think in their ordinary human relationships. If you think of bodies as medical students think of them it is not healthy. Medical students might see a body as mere flesh when they are working on it, but they probably have a healthy unreality when not at work and can emphasise other features of the human body in real life and fall in love.

I think that it is natural instinct to idealise and this has functional value in real life, allowing us not to be revolted by others (and ourselves). But because we have this instinct it is possible for it to be exploited by the media. Many people think that this is not healthy and I think they are right. We have a natural instinct which is being exploited for the glorification of a few for commercial reasons. It can't be helped as we are fed an image of a person in idealised form.

Perhaps being fascinated by the female form is normal! I should hope so! It might be like being fascinated by an expensive car. The latter is probably a form of aesthetic evaluation since expensive cars (with some exceptions) are better looking than cheap ones. The former might be aesthetic too.

We are not static beings and can shift our views given that everything has several aspects. You can focus on the expensiveness of the car or on its beauty. You can think of people with revulsion if you focus on certain aspects. Best not to do this though! Train yourself out of it.

As with aesthetic appreciation when we feel a car has qualities that make it beautiful, so with a great person. It is easier with a great person to say what these qualities are. Aesthetic qualities are very difficult to explicate.

Rachel Browne

back

Alan asked:

Is the speed of darkness faster than the speed of light?

You might think darkness couldn't be any faster than light, since darkness can only arrive as quickly as light departs.

A rather different question to yours, and one which you might need to ask an indulgent physicist rather than a philosopher, is: Can the speed of a shadow be faster than the speed of light? It strikes me that the answer to this question might be different to the answer to the former question, and some simple minded ideas about what light is would suggest that they are different questions: in the former question one is asking about the speed of propagation of the particles or waves of light, and in the second question one is asking about the speed of propagation of patterns in the particles or waves of light.

With a layman's eye, it is at least tempting to think that the phenomena where the shadow of a car cast by the street lamp appears to approach and overtake the car should also apply at higher speeds, across vaster distances. Supposing a dark object passed between ourselves and the sun at a very high speed, perhaps a speed approaching the speed of light, at what speed might it's shadow move across the face of the earth? Faster than the speed of light? I do not know. It strikes me as the sort of question one would ask if one wanted to test a physics student. Might we seem to get no shadow at all? Might the speeding object seem elongated to us, casting a slow moving shadow? Might the speeding object have peculiar effects on gravity, bending the light and the image of the sun? I do not know. If you find out, tell me. It's the sort of question that Feynman used to address himself to.

Now, to get back to the kind of light Philosophers hope to deal in, there appears to be some further evidence to complicate the picture. For it has been observed:

"A lie gets halfway around the world before truth can even get its boots on." — Winston Churchill

David Robjant


The Speed of darkness? Does darkness have a speed? What is darkness? It is simply the absence of light. Now what happens when I stop seeing darkness and see light? At the beginning there are no light waves incident on my retina, thus I see nothing (see darkness) and then, light waves, moving with light velocity (3*10^8 m/s) reach my retina so I see light. Now, when light ceases falling on my retina, I will see nothing, i.e. see darkness. After all, this is a physical question, not a philosophical one.

Arthur Brown

back

Gerald asked:

Can a philosopher be a philosopher for all people and all time? Meaning: Is philosophy capable of being unified? Can you reconcile two various philosophies into one philosophy (and/ or philosophy structure)? And have there been attempts to do so?

A unified theory (of gravity and quanta) has been the goal of physics for a long, long time. It's actually very interesting that we would consider the possibility of a unified philosophy, but on what grounds and using which parameters? Seems to me that there are no right answers to this two questions, so I can only hazard my own opinion.

My view is that philosophy, over the centuries, has shown itself a "monster of two heads", so to speak. Those heads would be: idealism and individualism. On one hand you have large philosophical systems that deal with the realm of ideas and the deeper meaning of actions and/or concepts; on the other hand you have smaller or disconnected philosophical theories, that are not systems de per se, focusing on the individual human being.

At first glance the two heads are opposite in a way there seems impossible to ever unite them. Just take Idealism vs. Existentialism, it's enough to read the arguments used by the most famous philosophers in each camp (like Plato and Nietzsche), to see the task of uniting such opposite views. So, is it possible? Again, my opinion would be: yes. But not by any traditional ways of thinking. I'm not aware of any attempts in the past to do so, other than on occasion, no one true global attempt. It's kind of strange if you consider that one of the main problems of philosophy has been it's ever-changing nature — theories span a very short time and are seldom build by criticizing (and destroying) the previous one.

Here we establish a comparison with physics: is there a set of laws that govern the universe, or is the universe the domain of chaos and chance? We can ask the same of the analysis of life. Is it possible to build a book of life or is life a river of ever-changing phenomena and chaos? Maybe we need a consilium to discuss this, but I would say we could start with existentialism and try to end on Idealism. Not sure how, not sure it could be done. Maybe someone could try it sometime.

Nuno Hipolito

back

Rahul asked:

In India millions of people struggle to meet their minimum needs while a fortunate few do very well because of an unfair social and economic system where property, education and other 'social goods' were confined to very few people. Doesn't 'social justice' imply that attempts must be made to improve the lot of those who are unfairly deprived, even if the right to property of the rich is not as protected as Nozick would have wanted it?

Rather than answer your question I would rather answer with a question of my own. On what scale do you judge "minimum needs" The perception of people in India would make us all rich including our homeless, but in our society most people cannot meet what would be considered minimum needs as well. Perhaps you should change your wording to necessities of life and ask this "doesn't social justice imply that attempts must be made to improve the opportunities to those who are unfairly deprived?"

Ryan Burton

back

Anna asked:

How might a Christian respond to someone who has racist opinions?

I take it that this 'someone' might also be a Christian? Racism is a pernicious, destructive [philosophy that contributes nothing to the development of the human project. Indeed, it subverts it. It places its primary importance on the external determinants of race or colour and on the basis of that it seeks to make a statement about the human person. Colour is important, because it is constitutive of the person who is that colour, but it is important as a genetic indicator of race but not of one's humanity — only human is the indicator of one's humanity.

Racism sets itself up as a valid way of looking at the world, the world environment and the people in it through a method of interpreting place and role in world and society based on cultural or pigmentational factors. It is destructive since it says nothing, or conveys nothing positive about the human people it speaks of since it speaks of those human people in negative terms to the extent that they cease to be human in the eyes of racism and racists. They are 'black'; 'Asian'; 'Irish' or whatever, they are not a 'human person whose country and culture of origin is ....' Racism, therefore, contributes nothing to the on going development of the human project. It is a scourge on the soul of humanity and feeds like a destructive virus or parasite on peoples' fears, ignorance and prejudices (though, having said that, the analogy with the virus or parasite is not fair, even the most destructive parasite or virus is what it is as a thing in itself, one does not condemn a virus for being what it is in itself, but one condemns a racist for being other than what he/she may become in their terms of capability and potential as a human person since they limit the potential; for their own human growth).

The Christian response (bearing in mind that some Christians and Christian groups have been racist, are presently racist, and will be racist) must be based on the fundamental conviction that all men and women are equal before God. But equality does not mean sameness, that is why we have different skin colours, languages, cultures, literature, music and so on. These variations of the creativity of the Divine are testament to the splendour and glory of creation; they are the 'dappled things' that Gerard Manley Hopkins speaks off which give praise and glory to the Creator.

The Christian response, therefore, must be the response of the one in whom Christianity has its provenance, and in whom it makes the confession of belief to be the one, true, and only saviour or the whole of the cosmos, Jesus of Nazareth, which Christianity believes to be God incarnate. His treatment of the Centurion (a Roman, whose servants he heals); the woman at the well in the gospel of John ( a Samaritan woman whose 'race' was utterly despised by Jesus' own race) — it must be one of acceptance, respect and liberation. The Sermon on the Mount is a universal annunciation; Love God and your neighbour is a universal annunciation; love your enemies is a universal annunciation. While Jesus is born in a specific history, at a specific time, in a specific culture, he is nevertheless, the fundamental unifying principle of all humanity (I am speaking from my own credal confession). Thus, the simple fact of the matter is: one cannot be a Christian and be a racist. It is diametrically opposed to every principle and value that Jesus of Nazareth held to be the determinant of his own life. The Christian response to racism, thus, is one of challenge and confrontation since what demeans the human person is unacceptable in the eyes of the Divine and it undermines and makes a mockery of everything Jesus of Nazareth became, is, and ever will be. It is to be rejected outright as destructive, pernicious, insidious and an infection in the soul of the human spirit.

Fr. Seamus Mulholland OFM

back

Stephen asked:

If we accept the notion that there are fixed 'laws of nature' (ignoring the excessively skeptical approach of the likes of Hume), we can observe and test such laws, as far as they involve observable actions and processes (as in scientific theories), and thus we may have reasonable inductive grounds for believing in the existence of such laws.

However, it seems quite a different thing indeed to state that there are fixed metaphysical 'laws', somehow analogous to the laws of science. Also, scientific laws of nature invariably measure processes, and tend to concern "how" things happen, whilst metaphysics tends to be largely concerned with the "what". If a scientific theory postulates the existence of an entity, it is to account for an observable process. If a metaphysician posits a metaphysical entity, it is a different kettle of fish altogether — most often based on intuition critically analysed within a conceptual framework in the metaphysician's mind. Thus, although any such entities postulated by science may be viewed as explanations of genuine observed 'laws of nature', any entities postulated by metaphysicians are inescapably bound to the human thinker's perception of the world, and thus have absolutely nothing whatever to say about an actual 'nature of reality', and certainly not a 'law of nature' (a 'how'), as the metaphysical hypothesis is an attempt to describe the 'what'.

Although this may be getting dangerously close to a logical positivist view, which states that metaphysics is nonsense, the claim I am making here is not that metaphysics is worthless, but rather that even the best metaphysical hypothesis which could be proposed says nothing whatsoever about the 'nature of reality', but instead merely critically assesses the human perception of such a said 'reality', from a purely anthropocentric point of view.

Although scientific theories may be inescapably anthropocentric to some degree, they do succeed in describing phenomena from some kind of objective stance, happily divorced from the human mental perception of the events observed. Metaphysical speculations, on the other hand, are purely anthropocentric, and thus cannot hope to describe any more than a purely human perception of a "reality", and certainly NOT any such "reality" itself.

So my tendency is towards a naturalistic approach to metaphysics (which may be great towards philosophy's contribution to, say, cognitive science, in understanding human concepts from a naturalistic point of view). The conception that there are actual metaphysical 'laws' (or 'facts'), though, does not seem (to me) to have any firm base on which to stand. In speculating about what "actually is" metaphysically, as if we can actually describe a "nature of reality", we seem to be ridiculously optimistic about the potential of human understanding, and it seems foolhardy to imagine that we can actually know about such things.

(I seem to be reverting to a rather Humean naturalistic skepticism at this point.)

My claim, therefore, is that there is no such thing as a metaphysical law, as analogous to a scientific law, and thus that metaphysics at best describes nothing more than laws of human cognition (which, of course, is still an important area of study — it may not be possible to understand reality per se, but only OUR reality, which may be all that really matters in the end...:-))

Can someone critique my views on these issues?

Well, where do we start?! You are raising the entire philosophical debate on 'reality'. This is covered by several authors, several commentators, and some of the greatest philosophers the world has ever known. You state your own position quite clearly, and I can sympathise with some of what you say.

Laws of nature are not fundamental in the sense that we have proof that 'nature' is in any way responsible for establishing its own laws. What we do have are notions about laws derived from alleged empirical evidence. Perhaps, as you say, on the face of it there seems to be grounds for inductive reasoning about such laws. However, it is induction about notions, and does this not take us into metaphysics? Where is the border-line between what we understand as metaphysical contemplation and what we consider to be inductive reasoning about hypotheses? Hypotheses and theories are in themselves a priori assertions, the fact that they are constructs of scientific reasoning does not in any way endorse them with empirical veracity.

In fact, scientists themselves are now beginning to question the notion of an objective reality since the massive paradigm shift, caused first by the quantum theory and the subsequent emergence of the superstring concept. We would be hard put to it to deny that science was now in the realms of metaphysics. In which domain should we place parallel universes? It is indeed science that is discussing them!! It is many years now since physicists watched with amazement as matter reduced to energised particles then to photons of light, they were even more surprised to note particles appearing in cloud chambers where no matter had previously existed. All this drove one great physicist to declare that the universe was a great 'thought' rather than a great machine.

Perhaps you should be referring to 'rules' rather than laws; in which case it seems legitimate to discuss metaphysics within the accepted rules of logic, the simplest case being that conclusions must follow from premisses. This would of course be true for both scientific hypotheses and metaphysical concepts. It could also be pointed out that many propositions in science, just as in metaphysics are conditional, in other words, of the form, 'If X then Y'.

I agree that science tends to measure things, and makes statements about construction and location, this is, of course, working with superficial sense data, well below the level of 'explanation' and 'understanding', where both science and metaphysics require to construct acceptable arguments. I do not agree that metaphysics has nothing to say about the 'nature of reality', as even a superficial study of some of the great thinkers bears out: Kant, Fichte, Schelling and Hegel, to name but a few. The views of all these philosophers still cause many scientists to feel more than a little uncomfortable with the empirical and basic materialist notion of the 'nature of reality'. I also disagree with your suggestion that metaphysics has no interest in the question' How?' How did the universe come into being? Has intriguing links with non-causal speculation, quite opposed to scientific concerns with the necessity of causation in all material events.

I also challenge the idea that scientific theories describe phenomena objectively, divorced from mental perception. Scientists work from sense data enhanced by mental constructs. Bare sense data without mental manipulation has very little meaning. We can say that scientific theories themselves are mental constructs tested against empirical evidence.

The subsequent paradigms through which humanity has progressed are basically theoretical constructs within which science attempts to work. There was a time when anyone arguing against a flat earth would have been considered to be mad, every bit of available sense data completely supported the 'fact', and no doubt the 'natural laws' were there to give full support. Evolution has remained a theory for over a hundred years, within which science has presented an illusion of development. The 'Big Bang' is a theory which forms a basis for cosmological adventures. Quantum theory is the basis of our rapidly developing new paradigm. Like all paradigms it may possibly self-destruct in the light of the foundations for a new paradigm. At the end of it all we shall still be questioning Kant's 'things in themselves', metaphysicians will still be seeking to know 'what there is'. Berkeley will still be posing the question, 'Is there really anything out there?' Science will still be involved with its reductionism, whilst philosophy continues to point out that there is more to life than 'particles', and all known objects are more than the sum of their parts, particularly life forms.

John Brandon


Sure. You're starting with many metaphysical assumptions. You state:

"1) If we accept the notion that there are fixed 'laws of nature' (ignoring the excessively skeptical approach of the likes of Hume), 2) we can observe and test such laws, as far as they 3) involve observable actions and processes (as in scientific theories), and thus we may have reasonable 4) inductive grounds for believing in the 5) existence of such laws."

I count five assumptions just off the top of my head in that first sentence, all of which have been debated, some for merely a century or so, and some for thousands of years. But if you make those assumptions (which I've really not exhaustively analyzed — for example, just what does "reasonable" mean?) then yes, I agree with your conclusion... I think. I'm actually not really sure just what you mean by the term "metaphysics", except inasmuch as it relates to your assumptions. And by making those assumptions, you've pretty much closed the door on the possibility of other metaphysics, and thus I imagine of "metaphysical laws". But again, since those assumptions you're making are metaphysical, then laws following from them are metaphysical. Thus, what you term "inductive grounds" are in fact metaphysical laws, are they not? And so forth.

Steven Ravett Brown

back

Gabrielle asked:

"Discuss the Nietzschean Approach to the meaning of life in Milan Kundera's novel The Unbearable Lightness of Being."

Please, I am having trouble with this. What THEMES are evident? How do I CONSTRUCT the essay? What should I deal with first? How is the Nietzschean approach evident?

How are the themes of nihilism, existentialism and lightness and darkness evident?

Should I even focus on themes? I'm finding this very difficult.

It will be easy once you have done some reading and there should be enough on the internet: Search for Nietzsche and "The Unbearable Lightness of Being" — but if this is a literature essay, don't read too much but concentrate on the novel. This would be easier to answer if I knew the subject you are studying.

Start off by quoting Nietzsche on eternal recurrence which is the MAIN theme. This is what you have to consider in relation to the meaning in the characters' lives. You can find a full quote at: http://www.webster.edu/~corbetre/personal/reading/kundera-unbearable.html.

For one short outline of what Kundera sees as the "meaning of life" have a look at the plot overview at: http://www.sparknotes.com/lit/unberaablelightness/summary.html.

The claim here is that since we cannot return, we can only find meaning by comparing different lives, like those of Tomas and Sabina — I don't think this is fully correct. For sure, in comparing the lives of Tomas, Tereza and Sabina, Kundera is showing that the lightness which Tomas represents (which is really moral lightness or nihilism) is not to be favoured over heaviness and moral responsibility represented by Tereza. But he goes further and shows that while Tomas comes to accept heaviness through Tereza, Sabina doesn't come to accept it. Sabina ends up loveless and rootless and this implies that at the end of her life she wouldn't want to repeat it. Nietzsche's point is that if you think about the possibility of eternal recurrence, the question arises whether you would go for it or not, whether you would affirm your life. If this is a literature essay, have a look in The Cambridge Dictionary of Philosophy to find out about Nietzsche.

The plot overview claims that meaning has "uncertain existence" because of the lightness/heaviness dichotomy. But it doesn't make sense to say that meaning has "uncertain existence" because this is to abstract meaning from people's lives and then we have no idea what meaning can be. Using novelistic form, and considering the possibility of eternal return, Kundera makes the meaning of life concrete and particular rather than a general abstraction, which Nietzsche would approve of.

So the main thing is to consider "the meaning of life" in relationship to Nietzsche. You can consider the theme of nihilism in relation to the behaviour of Tomas. Nihilism was easy for Tomas, it was light, but he wasn't living fully, with commitment. Life was a game where he had a rule about when he could see his mistresses and for how long following a recurring pattern that was only a parody of real meaning. After he met Tereza's he became a person of more integrity and would not withdraw his published views.

You could consider idealism after nihilism — this is represented by Franz, who gets crushed. Idealism is very different from the real-world heaviness of Tereza. The meaning of life totally evades Franz — he is worse off than Tomas and Sabina. He has to adopt the fantasy of Sabina as a guide to what he should do and lives with a woman to whom he isn't fully committed. Sabina represented a false god for Franz. For Nietzsche, God — or perhaps any god, such as Sabina in Franz's case — was becoming meaningless and so there is a need to look at living life more fully (through thinking about eternal recurrence) and to look at reality — which is seen in the novelistic style Kundera adopts of looking at his characters from outside.

As far as I know, in limited knowledge of continental philosophy, Nietzsche isn't regarded as an existentialist. But you could look at Gregory Kimbrell's paper on Existentialism and The Unbearable Lightness of Being which can be found in the google search suggested above. According to Kimbrell who draws on Kundera's The Art of the Novel, Kundera adopts Heidegger's description of our existential situation as being a "spatial, temporal and cultural designation in which human beings are involved and are presented with a particular set of possibilities for the realization of their individual selves". But Kimbrell expounds Kundera's existential position in relation to Nietzsche's idea of eternal recurrence and repetition.

When you have read this answer and the papers on the internet you will find that we are not all saying the same thing. I think lightness and weight have a different meaning when considered in terms of nihilism and existentialism. So it is up to you to think about it for yourself.

Rachel Browne

back

Daniel asked:

I was reading in the Oxford Illustrated History of Christianity and came across the following statement: "Four centuries earlier Socrates at Athens observed that a really righteous person would be so unacceptable to human society that he would be subjected to every humiliation and crucified." Unfortunately, the statement is not footnoted. If it had appeared in a less prestigious publication than Oxford University Press I might not have thought much about it. But, I cannot for the life of me remember reading in any of Plato's writings, or any disciple of Socrates, any such statement. Can anyone point me to the text that this statement is referring to?

Yes:

"The just man, as we have pictured him, will be scourged, tortured, and imprisoned, his eyes will be put out, and after enduring every humiliation he will be crucified..."[Republic 361e5, voice of Glaucon, translation Desmond Lee].

This passage, which has seemed prophetic to some Christian readers, has a specific philosophical purpose for Plato, and even if it is prophetic, it isn't offered as prophecy in the way that Nostradamus offers prophecy. Plato doesn't think of himself as looking in to a crystal ball. Conceivably, there is the question of whether Jesus could have read or otherwise encountered the Republic, but I'll leave the currently popular debate about Jesus's role in bringing about his own fate exactly where I found it. Those who do trouble over the prophetic qualities of this passage in the Republic make much of translation issues, and whether the greek text best justifies "crucified", or whether "impaled" and so on. But in my view, that's entirely besides Plato's point.

What Plato is up to in the surrounding context is setting up a 'thought experiment', as we empirically minded anglo-saxons like to call useful flights of the imagination. The question he wants to answer is: is it better to be just or unjust? In the passage, Glaucon is helping Socrates clarify the meaning of this question. Plato's thought is that if we want to see whether the enviable life is the life of the just, or instead the life of the unjust, then we have to arrange for these things to be pictured in their extremes. Thus: "We must strip him of everything except his justice". Imagine a man reviled, accused, tortured, who still withal remains a just man (albeit one who cries out in pain). Should we praise and attempt to imitate that kind of man? Over the course of the Republic Plato argues that the answer to this question is 'yes', and this is another point of confluence (or influence) between Plato and the Christian tradition. The most unhappy and pitiable man, Plato thinks, is an unjust, tyrannical man who has come into power (managed to get away with it). This belief is much doubted, but Plato has his reasons to do with mental health and the perception of reality (reasons compared by Iris Murdoch to Buddhist thought) and I, for my part, after reading and re-reading the Republic, agree with him. Perhaps I agree with him with waning intensity when I have particularly bad pain in the ear and grow impatient with some unhelpful doctor, but this, I suppose, is only to say what is patently obvious, that in certain situations it becomes harder and harder to think at all.

David Robjant

In responding to this, David Robjant refers to Desmond Lee's translation of the Republic, which reads in part, "...and after enduring every humiliation he will be crucified..."

The actual Greek, which you can find at,

http://www.perseus.tufts.edu/cgi-bin/ptext?doc=Perseus:text:1999.01.0167:section=362a

doesn't refer to crucifixion, but to impaling. (A search for the Greek equivalent of 'crucifixion' in standard Greek-English lexica returned nothing.) I suspect that Desmond Lee is either using a more familiar way to express the horror of the truly just person's fate, or is siding with those who would like to 'Christianize' this great pagan philosopher. A more recent, widely-used translation is that of G. M. A. Grube, as revised by David Reeve [Hackett, 1992]:

"They'll say that a just person in such circumstances will be whipped, stretched on a rack, chained, blinded with fire, and...at the end, he'll be impaled..."

Crucifixion seems to have been a Roman invention.

Robert Paul
Professor of Philosophy and Humanities, Emeritus
Reed College

back

Paul asked:

Should God the bible and religion be given the same consideration by evolutionists as scientific theory?

It meets the criteria. And if so wouldn't the extremist who assert "all religion is nonsense" be deemed by both creationist and evolutionist as being out of step and illogical?

Part of the problem here is determining the role of both. The Bible does not deal with 'scientific' truth but with religious truth as it is experienced by that particular group of people. This makes its subjective but in its very subjectivity lies its claim to truth. The worth of religious truth claims, therefore, cannot be determined by science, but only by the ones who hold to that particular religious belief. Science can determine its asserted truths by recourse to empirical experimentation and demonstration that its hypotheses concur with the accepted laws of physics etc. but it cannot demonstrate that the biblical text is 'wrong' since it cannot apply the criteria for empirical or physical determination to the religious vision of the world made in the biblical text.

We are confronted with the age old problem of the relationship between faith and science. To accept the biblical vision is not to be at loggerheads with the scientific vision of the world and one would expect vice-versa. However, it seems to be the 'in thing' that proponents of the 'new science' attempt to claim that science and its discoveries can give us all the answers. Indeed it can. It can give us all the answers it possesses thus far in its discoveries of the structure of life and the universe, it can show us what the physical purpose, or biological purpose of phenomena are, but what it cannot do is tell us or show us what its meaning is beyond the phenomenological. Only a different 'world' vision can do that, and men and women of faith make this claim.

As a Franciscan I embrace what science can reveal to us of our brothers an sisters in the universe, embrace it and claim them as Brothers and Sisters, and marvel at the splendour of God's creation, where every living thing is a little logos of his mind. At the same time, I accept the theory of evolution and the 'Big Bang' as the best working hypotheses we have at the moment for the historical and existential manifestation of physical phenomena. But, as a person of faith and religious belief, I believe that it can be given shape and dimension that takes it from the realms of the physical to the realms of the mysterious and profound and awe. It is not a question of logic, or illogic, but of perception and belief.

I have no problem with evolutionist theories, but they may have a problem with my assertion that the manifestation of physical phenomena has its source in the creative dynamism of God who simply speaks the word 'Let it be' but does not tell that physical phenomena how 'to be'. That is the inherent freedom of the universe — and science cannot quantify that, only measure the consequences of its exercise of that freedom -evolution, the Big Bang, General Relativity, 1+1 will always equal 2 etc. Religion and science are not out of step, they are simply walking the same way by different roads.

Fr. Seamus Mulholland OFM

back

Taz asked:

How convincing are religious experiences in a secular age as a mean of justifying religious belief

Religious experiences are very personal events, hence, to convince others of such experiences is a very difficult task. It is, of course, easier to convince 'believers' than those set against such phenomena. In a secular society where the understanding is that the materialist views of science do not cater for spiritual revelation, religious experiences are usually taken to be hallucinations. Physical manifestation are usually judged to be coincidences, or at best interesting events which will eventually submit to scientific investigation, and prove to be 'natural' occurrences.

Justification of religious belief is best backed up by logical argument. Empirical proof, so far as I understand, is extremely hard to come by. In a secular society believers are usually linked with superstition, and exaggeration of their experiences. On the other hand secular views themselves are open to dispute, and just as believers usually find it difficult to prove religious experience, so opponents have an equal difficulty in refuting the claim.

John Brandon

back

Lucy asked:

What are the strengths of the design argument?

1. That it is based on the quite evident Complexity of the biological creatures.

2. That it makes use of the huge length of the evolutionary time scale, that evolution becomes almost totally unnoticed in our short lives.

3. That it makes use of a native (but most probably false) human question to ask, 'What is the use of this organ' since this promotes an idea that those organs were created on purpose, instead of saying 'a leg is made to make us walk' you'd better say 'a leg serves in walking' Since evolutionary speaking, an organ arises by random mutation and then the organism elaborates it, and makes use of its presence to survive via natural selection.

4. It is a very favorable substitute for the considered- repulsing view of evolution as a blind watchmaker, which thus means we were the products of a non-willing chance.

5. It makes good use of the fact that there could have been dozens of other worlds that could have been totally inhospitable to any form of life. If the electron charge was just a bit larger, all atoms could never unite to produce molecules. Or if there were 2 or 1 space dimensions instead of 3 or any other small changes biological life would be impossible since a simple GIT in a 2D creature would cut him into 2 pieces! Further the chance of DNA production is very small, etc etc..

6. It usually attributes creation to god, who, in spite of being thought of as being more complex, is invisible so it is not likely that someone would wonder how his complexity is present without being created. But finally, in spite of all those 'Apparent' Strengths of the design argument, almost no philosopher would hesitate to dismiss it as falsehood, and since you asked only for the strengths, I assume you already know the much longer list of weaknesses and thus will not discuss them.

Arthur Brown

back

Heather asked:

I was looking at Jurgen Lawrenz's Pathways article Discourse on Malady, and I have a question... Are MS and Le Monde one in the same (as a text) or what is their association? I am researching some of the persecutions that occurred during the early modern period of philosophy; and Descartes' burning of Le Monde is part of my research. I would appreciate any information you could give me. Thanks.

Heather Sokinas, a philosophy student.

My quick answer is that they're not the same. But this needs a little explanation.

The first thing you need to understand is that my piece on Descartes is a parody. The man died ages ago and it's not possible any more to "psychoanalyse" him. So the "MS" refers to the FICTION that an essay by Descartes has been discovered which looks like an early version of the Discourse. But this is just my writing, an imitation of the Discourse with some things changed, and the purpose was to give you, readers and students of Descartes, something unusual to think about: namely that he was very good at hiding himself from close scrutiny by others. I've come around to the view after reading much of his autobiographical texts and testimony of others that one of his really big problems was SUPERSTITION and I believe also that the "vision" he mentions in several of his writings and especially the story of the demon were a terrifying nightmare he went through on that day. So the design of the whole piece is intended to show that Descartes took up the struggle with himself to banish both superstition and the demon by giving us a MIND/BODY duality and a totally MECHANISTIC philosophy. In such a philosophy superstition and demons are impossible.

Now as regards Le Monde, this was Descartes' first book. It was due to be published in 1633, but after Galileo's condemnation, Descartes (who was a catholic) took fright that the same thing might happen to him and withdrew the book. The quotes I give from the book are genuine and show that he had good cause for worry: for he, Descartes, could be interpreted as having INVENTED HIS OWN GOD; after all, he practically dictates to God what laws of nature apply to any universe He might wish to create and so on. Now the last book he published, the "Principles of Philosophy" is actually Le Monde all over again, but by now completely rewritten and a bit of a tragicomedy, because he was trying the impossible of bending his own theories to make them innocuous to the theologians while still rescuing what he could of his own thoughts (and still trying to replace Aristotle). So this last book hasn't got the bite any more of the early one, because it has no teeth. Le Monde meanwhile has survived only as a fragmentary text.

In short, whatever I wrote Le Monde is factually correct. Just be careful you don't mix it up with my parody of the Discourse. As far as I am concerned, however, I have been trying to do what any conscientious fiction writer might do when confronted with big holes and inexplicable facts in a famous man's biography: plug them up with an imaginative reconstruction of what MIGHT truly have happened and could therefore explain many a puzzling fact about the man's known biography.

As far as Le Monde itself is concerned, you can chase up what Daniel Garber and Stephen Gaukroger have written about it; they are two authors who deal extensively with Descartes' physics.

Jürgen Lawrenz

back

Luciano asked:

Descartes believes that animals have no souls, and presumably no inner lives. Yet one can observe behavior in animals which seems to suggest that they do in fact experience emotions. How might Descartes attempt to reconcile his theories about animals with empirical evidence that seems to refute them? Do you think he could do this successfully?

Unfortunately, Luciano, I think he can. You can step on a dog's tail, and watch it yelling with pain, but you can't 'get into' it's consciousness to see that pain. Yes, you think that yelling can prove pain, but if you dream a dog is yelling you will be seeing it yelling without 'it' having any pain, simply because it doesn't exist! Thus the jump from the yelling of the dog which is the 'result of pain' into 'the dog is having pain' is not acceptable. Moreover, there is an even 'bolder' theory, by the name of solipsism, which states that 'I am the ONLY conscious creature' it negates even other people's consciousnesses! And it is logically valid, but not necessary just for the same reasons of Descartes ideas.

Descartes' cripple idea was regarded with delight by religious authorities at his time since it came in favor with the medieval ideas of humans being a 'special creation' totally different than the 'beasts of the fields'!

Arthur Brown

back

Butch asked:

My question is: Why are mathematics and theory the only practical way to determine the temperature of the Earth's core?

I've a feeling that what I'm trying to say is very simple, but I can't find a good way to express it. Perhaps you can help me figure out what I'm toying with here: If we call "Universe" the ultimate set of everything, from which nothing that exists is excluded (a common usage of "universe", I think) then we cannot imagine (posit, conceptualize, propose) any universe but this one — just as it is — because any imagined "other universe" becomes immediately a part of this one.

This scenario requires that we grant existence to ideas. (An uncontroversial claim, it seems to me, for while ideas do not exist in the same way that chairs exist, as existent things they are no less evident.)

Finally, a question (maybe): If we don't know what something is, can we nonetheless propose it?

Well you seem to have expressed it just fine. Ok, now what? You've defined the word "universe" in a particular way, and certain things follow from that definition. Yes, and...? Does this relate to the real world? How do you know?

The problem with your question is that it's not posed well. What do you mean by that little slippery word "is"? I don't know what an electron is, but I can talk about it, and even propose it because I know some of its properties. Are you asking whether you have to know something, no matter what, about something before you can talk about it? I would say yes, offhand. I suppose you could just say, "there's something which might be called a 'knuuxy', but I don't know anything at all about it beside that, and I'm not even sure of that." But you've just used "knuuxy" as a noun, and so we think, by that usage, that it's a thing, rather than a modifier, like "quick", for example. So we know something about it, and thus we know what it "is", to some extent. But since I don't know what you mean by "is", I don't know if I've answered your question.

Steven Ravett Brown


Of course! Beginning with the fundamentals, Socrates said "I know nothing" after this nothing is certain. To move forward as a society we have to make conclusions based on probability of truth rather than absolute truth itself. So what we do is create theories based on perceived knowledge that are in an ever changing state. When new information comes to contradict the old theory we reanalyze the theory and submit it again. This is considered a sort of microevolution of our species. Without accepting things as a 100 per cent truth there would be no way to move forward at all because we could never come to a conclusion without absolute truth. So in short, you can propose anything, but proving it is where its veracity come into question.

Ryan Burton

back

Hannah asked:

After Adam and Eve, where did all of the humans come from?

How did a monkey turn into a human?

Who are Adam and Eve? Are you expecting me to give philosophical (or even scientific) credibility to a fairy tale? Surely you understand that the story of Adam and Eve is a metaphor? As to the other question, no monkey ever turned into a human. You've either misread or been misinformed; and if the latter, your informant does not seem to understand how species of life evolve. I suggest you go to your local library and get hold of one of the many picture books they have on Life. If you're worried about monkeys etc., look up "mammals". Then you'll discover something about body plans and how nature employs very few variations on them to produce entirely new species and branches, many of which will consequently share anything between 50 per cent and 99 per cent of the same genes. Then you might discover (just as an example) why rats are used to test certain human medicines and much other fascinating information on what life is really all about.

Jürgen Lawrenz


(1) Africa

(2) You are still a "monkey". You are of the primate family. Specifically, you are a type of "proto-gorilla", as in, we and the gorillas share a common ancestor. We just call ourselves "humans".

Kim Boley

back

AJ asked:

I began my interest in philosophy the beginning of the school year. I am in my senior year of public high school and I am taking a class called global perspectives. In this class we learn about and identify "myths" in our society. We have read a Daniel Quinn book called Ishmael and I have read Sidartha and several various articles from a wide range.

I have noticed throughout the year that some people are not able to talk about these things on the same level as I because they are not very intelligent... I started to talk about this with my best friend who is very open minded. So I began to search for more open-minded people I could talk to, and I found that open- minded people are very receptive and are able to come up with good thoughts in discussion situations. I am also accepted to college and I'm going to major in Engineering. However I was thinking of philosophy or some type of theoretical mathematics to get a PhD in. What do you think about my career options and my thoughts?

I'd say that if your college allows it, that you should take a wide variety of courses in mathematics, philosophy, and the sciences (and yes, in the arts) for the first year or even two years, and find out just what you're most interested and talented in... and go with that. You won't make nearly the money in math and particularly in philosophy that you'd make in engineering, I can tell you. But if it's what you really want to do... do it.

Steven Ravett Brown

back

Annemarie asked:

Is movement and having a body essential for self-consciousness?

Well for me as a first year philosophy student with very little reading under my belt it sounds to me like the question is asking can a mind exist without a body, but I may well be and probably am way of the mark. I am thinking along the lines of computers or machines.

No, you aren't way off the mark.

Though as I see it the question also applies to disembodied existence and whether this is possible. We can conceive of a mind without a body which allows many people to believe in a soul. We can say it is possible in the sense that people can believe in the soul, but when you look into the nature of the mental it doesn't look like a natural possibility. Thought is now taken to be affective, not a purely rational process. If it was a purely rational process it couldn't give rise to an idea of "self" which perhaps arises from body awareness and emotionally charged thought. Whether thought can be affective, or emotionally charged, when a being is disembodied is physically doubtful.

Factually, the biological nature of an organic being and neural activity in the brain are known to give rise to both thought and emotion.

You could look at Damasio's Looking for Spinoza. Damasio thinks that body imaging is essential for self-consciousness. In particular it is necessary for feeling. Feeling, for Damasio, is a conscious or mental state, whereas emotion is physical. How could the emotion associated with fear — a sudden bodily rigidity and hastening of the heart beat be replicated in a computer so that it could give rise to feeling?

For Damasio, it is through feeling that we learn to go for things or avoid them. But you can't have feeling without bodily emotion. Can you imagine a disembodied being having feelings? What would a purely mental fear be or a mental love? Conceptually, fear implies being rooted to the spot or running away. Love implies physical movement towards that which is loved.

Consciousness is about mental states, but self-consciousness brings the idea of an individual. Self-consciousness is only possible if there is consciousness, but goes beyond it. A series of perceptions might amount to consciousness, but the concept of self-consciousness implies continuity: Memory and planning for the future in the light of the past which in turn suggests caring about the future and being in some subjective way related to one's own past.

It is unlikely that computers and machines could achieve this. Most people don't ascribe consciousness to machines. We don't suppose they have subjective experience, consciousness or a self.

On movement, you might consider perception and whether visual sensory experiences are sufficient for perception of spatiality. As a first year student it might be appropriate to look at Berkeley's New Theory of Vision. Also you might look at a modern paper published on the internet such as Rick Grush's "Skill and spatial content". This can be found through http://www.u.arizona.edu/~chalmers/online2.html. You will be able to find out about Searle's Chinese Room argument at this site too. Searle believes that nothing other than something built along the lines of an organic being can think. If you think otherwise, you will find support in arguments against him.

Rachel Browne

back

Frank asked:

To what extent can it be maintained that a literal interpretation of Christian scripture removes all danger of human error?

Ill give you a counter question to ponder. To what extent can it be maintained that a literal interpretation of Aristotle removes all danger of human error? If your answer is negative, then you have the answer to your question as well. Not forgetting that there's a lot more brain power in Aristotle than the Scriptures. In short: your question relates simply to what you, as a human being capable of thinking, make of the scripture texts. You have no assurance than another persons word (who in turn relies on yet another persons word and so on through the millennia) that anything whatever in the scriptures has a source outside a human mind. That's a lot less than science or philosophy would impose as the minimum standard of critical thinking.

Jürgen Lawrenz

back

Yanique asked:

I would like to know who said the first philosophical Greek saying. My professor gave us this paper with a Greek saying — but it is in Greek and I have tried every book and website I can think of but still unable to decipher it so I really do need your help.

Go here and look it up under Lexica: http://www.perseus.tufts.edu/cache/perscoll_Greco-Roman.html.

Steven Ravett Brown

back

Irving asked:

I am writing a book on mysticism focusing on the divine Word, Sound, Shabd, Logos. To me, Plato is referring to this in his Ethics/ Cosmology as the World Soul being a musical instrument. I find very little elaboration on this other than what I have just said above. I would appreciate being pointed to this where it is delineated in greater detail, in both Plato and other philosophers.

I just know a small detail that I thought would be of help to you. In the Indian Vedas, sometimes creation is spoken of as a result of sounds, that is all creatures are nothing but the embodiment of sounds and the whole universe is a symphony.

Arthur Brown

back

Alicia asked:

What is a simple method of studying maths?

Start at the beginning and copy.

Relax and use all the skills you learnt doing something else well to do this just as well and if you get real lucky you will find a good teacher who will help you with your problems.

Kim Boley

back

Brad asked:

Gilbert Ryle argues against the Cartesian account of mind and body. What is his challenge, what sort of mistake does he think the Cartesian is making, and what information is the Cartesian unable to explain? What is Ryle's own idea of the mind, and of mental states?

You might welcome the information that all the answers you are seeking can be found in the book Gilbert Ryle has written on the subject. Its called The Concept of Mind. Happy reading!

Jürgen Lawrenz

back

Stephen asked:

A while back, I took a course in Reiki healing (which I felt particularly drawn to). Although the course claimed explicitly that no special belief is required for the practice of Reiki, the course was steeped in new age metaphysics (chakras, meridians, auras, crystals, angels, energy fields etc.), all of which I am extremely sceptical of. (I regard myself as essentially an agnostic or atheist.)

For those who are not aware of it, the main metaphysical claim of the Reiki system is that the practitioner taps into a source of 'energy' from the universe called 'ki' (equivalent to 'chi' or 'prana' in other Eastern philosophies). 'Rei' refers to this energy's alleged divine source. Although I do not believe this literally, regarding it as somewhat of a 'figment of the imagination', I am aware of the psychological power of a mental image (creative visualisation), and so take the metaphysics as purely a (fictional) visualisation as an aid to practice. On practicing it, I have found it appears to be very effective, despite my scepticism. I thus seem to have a purely pragmatic approach to it.

However, some critics have claimed that it is impossible to divorce the practice of such arts from the associated metaphysical beliefs, especially the highly traditional forms, such as the style of Reiki I learned. Is it truly possible to practice such arts without 'buying into' the metaphysics to some extent (even it is only one's subconscious which accepts them)? If not, then how could a sceptic honestly practice without compromising his/ her rational belief system? (I'm ignoring the 'absent healing' aspect of the practice here, involving healing at a distance, which is purely a matter of faith).

My pragmatic approach to the subject does not feel very solidly grounded, however, and I sometimes wonder if a lot of what I'm doing is just pointless 'mumbo-jumbo'. I would therefore appreciate some philosophical guidance to shed some light on this issue, and to help put my mind at rest on this point!

Another important question is, would it be ethical for me to teach traditional Reiki (after taking the appropriate training) if I don't actually subscribe to the metaphysical beliefs which I am teaching? If I were to introduce a measure of scepticism (or pragmatism) into the training I were to offer, by saying that the metaphysics does not have to be taken literally, would this detract from the authenticity of the training I am providing, especially if I am offering it as training in its traditional form, which has never taken the metaphysics figuratively, to the best of my knowledge? (In traditional Reiki, for example, 'attunement' rituals are viewed as absolutely necessary for induction into the practice, in order to 'tune' the practitioner into the 'energy source', which is something I personally doubt is necessary, as a direct result of my scepticism).

I'd appreciate any help on these issues, as I'm wondering if a sceptic like me would be better suited to another career path!

If you find that Reiki is "very effective" perhaps there is a source of energy that is tapped into. This could have a divine source — in which case I would understand this energy source to be in some way external to the individual. As you say, it is easier to believe in a psychological power — but what would that be and why haven't psychologists founded this method of healing? What words would they use to describe the energy?

I would suggest that metaphysics isn't supposed to be taken literally. You are buying in to a way of speaking about an energy that can't be reduced to the physical and cannot be measured even if it is psychological. If you recognise this you are not compromising your rational belief system, but simply accepting that there are phenomena in the world that cannot be rationally explained.

The "mumbo-jumbo" hasn't been pointless. It is the origin of something you take to be an effective healing practice. It may seem obsolete, but you could try to respect the way the mystical origin has given rise to an effective practice.

Perhaps it isn't quite sincere to continue to practice traditional Reiki but I'm not sure it is really unethical to teach things you don't believe in. You could see teaching as being for the benefit of the one who learns and for whom the metaphysics might be acceptable.

If you were seriously sceptical you wouldn't have become a Reiki healer in the first place!

Rachel Browne


Probably. Most of that stuff is utter garbage. Let me put it this way: the placebo effect works about 30 per cent of the time. So if you can reliably demonstrate that you get better than 30 per cent "healing" or whatever from that stuff, you're above the baseline for plain suggestibility.

I'll put it another way. You can certainly justify practicing the above on the basis that if someone believes that it will help, it will, about 30 per cent of the time. If that's good enough for you, then, hey, go for it. Just be aware that anything else that people believe in will also help equally. The ethical question is whether it's moral to support beliefs in nonsense in order to achieve the real results that you will achieve through suggestion. A hard question to answer, since you won't convince most of the people coming to you that their beliefs are wrong. I wouldn't do it myself, but perhaps I'm wrong, and it's worth it for that effect, which is a real one. Be warned, however, that the placebo effect has its limits. You won't cure cancer and pretty much any sort of serious illness with it, and supporting a belief in what is basically superstition will bias people against seeking legitimate doctors who might actually help them. Aside from the dubious morality of supporting a belief in superstition in the first place.

You might take a look at a magazine called the Skeptical Inquirer on this sort of thing; it's on the web. Also, here are some references:

Giovannoli, J. 2000. The Biology of belief: how our biology biases our beliefs and perceptions. Rosetta Press, Inc.

Harrington, A., ed. 2000. The placebo effect: an interdisciplinary exploration. Cambridge, MA: Harvard University Press.

Hines, T. 1988. Pseudoscience and the paranormal: a critical examination of the evidence. Buffalo, NY: Prometheus Books.

Schick, T., Jr., and L. Vaughn. 1995. How to think about weird things: critical thinking for a new age. Mountain View, CA: Mayfield Publishing Company.

Shermer, M. 1997. Why people believe weird things: pseudoscience, superstition, and other confusions of our time. New York: W. H. Freeman and Co.

Young, A.W. 2000. "Wondrous Strange: The Neuropsychology of Abnormal Beliefs." Mind and Language 15 (1):47-73.

Steven Ravett Brown

back

Geert asked:

What is reality??

I don't know. Looking out of the window I would believe the tree is a tree and the grass is the grass. But I don't do physics. Probably no-one can say. What does reality mean? The only way we can take things when not hallucinating, or what underlies the way things are?

I think it is our shared world and not the physicists' world. Realities are not all about particles but all of the things mankind has in common. Apparently there are subjective, or personal, realities too.

Just because we have a word or concept, it doesn't mean we can say what it means.

Rachel Browne

back

Maiken asked:

What is time?

What is the fundamental property of mind? Of matter? I have been pondering these two questions and am trying to answer them to the best of my ability, but am stuck.

I'm assuming that you are not going to read the thousands of books and papers that have been written on this subject. Thus I am further assuming that you want a quick, unambiguous answer without all the complexities involved in this question. Ok then: The fundamental property of mind is agency and the fundamental property of matter is condensation. By agency is meant the capacity to move without being pushed; accordingly an agent (i.e. a being with a mind) does not need an external cause to move around, and can push objects just by willing and exercising the appropriate muscles. Conversely, by condensation we mean that matter is compounded of elementary stuff, such as atoms. Intrinsically matter is inert; in order to move, it needs to be pulled and shoved. Now gravity evidently does this, and thus we seem to be in a pickle: are we obliged to suppose that gravity is also mental? Fortunately the answer to this is relatively simple. Gravity doesn't actually pull or push in the same way that you might push a wheel cart, so its not related to agency (no mind involved). It is matter-like in the sense that condensation is a kind of basic law of the universe. All matter tends to compact and gravity can be understood as such a universal trend. You've probably heard that on one theory the universe "exploded" in a big bang and will contract again in a big "implosion" (big crunch). Therefore the opposite side of gravity is that explosion occurs when matter contracts and heats up so much that the particles have so much energy compressed in them that "banging" is the only way to go. I hope this gives you a sense of the essential difference that you seem to be asking for.

Jürgen Lawrenz

back

Audrey asked:

According to Robert Solomon, The Myth of Sisyphus is a philosophical theory, a vision of the absurd. Sisyphus really represents all of us as we spend our lives in futile quests. in the end, only personal experience is meaningful. Solomon asks if the absurd would disappear in the face of irrefutable evidence that God exists?

I've basically sort of given up answering questions like this... because I've done it so much. So first, go look in the archives on this site about questions on the meaning of life, the existence of god(s), etc. But your putting it in terms of "the absurd" is somewhat intriguing to me...

Let's say that we had that proof. Ok, something we'll call "god" exists. Now, that means, I assume, that we are, or should be, following some sort of "divine" plan that it has for us and the universe. Ok... so what? What makes that plan, or whatever, "meaningful"? Take a god... it's got this plan... and it's just made it up from nothing... after all, where would a god get a plan from, right? So it's made up some plan for life, the universe, and everything... and... so... what? What justifies it? What makes it any more meaningful than a plan you or I would make up? The fact that a god knows everything, and so its plan will work? Um... that's nice... but my plan might work also. I guess that a god knows that its plan will work, and so you and I know that its plan will work. In other words, god's rock will get to the top of the hill. Whoopee. Why should we care? Because there's no other game in town? No, not true, we can make up games also. Because god's game is better than ours, since we know it will work and we don't know that ours will? Well, that's some kind of criterion, all right... but it seems pretty weak to me. After all, maybe it's better not to know, then the game is more fun, right?

So, all in all... I just don't see that there's any more point, unless we want a sure win... and of course that assumes that god's plan gives us (and not just god) that win, which assumes that god is "good"... so we not only have to prove it (god) exists, we have to know its nature. And then, after we do that, we still have to feel that going with a winning game is somehow "meaningful", not "absurd"... and now that we're on that question, just what does make something "meaningful", after all? Or conversely, what makes something "absurd"? It doesn't seem to be having a plan, or a game... we can do that. Having a winning plan doesn't seem to add much... does it? So, what else?

Let me put it another way. There's a very interesting novel by Colin Wilson, The Philosopher's Stone, in which he asks somewhat the same question. His answer is that the mere existence of a being which is superior to us, plus our acknowledgment of it as superior, will motivate us to work for its ends. I think, based on the history of religions and of tyrants (which are actually not really separate issues), that he's correct. But the question is, should he be correct? Should we behave that way? We can look at studies of dominance/submission in animals for reasons that we behave like that, but those reasons have to do with animal survival, all well and good until now. But now we are able (to a certain rather feeble extent) to ask, answer, and act on higher-level, i.e., "existential" questions. God's providing us with a game-plan is the same thing, isn't it. You can bring the issue down to not wanting to go to hell, or some equivalent afterlife, but that's just avoiding the stick for the carrot... a refined version of animal training. Or you can say that the acceptance of some entity's dominance over us assures meaningfulness in our carrying out its goals. That seems pretty silly also, doesn't it. Or, as I say above, you can say that the assurance of being in a winning game provides meaningfulness. But that leaves the question of the game's goals open, not to mention why "winning" is meaningful, and not losing, or whatever... and so, you have to say that there's something about the end of the game that provides "meaningfulness".

Ok... what is it? Solomon, you say, answers that with "personal experience"... um... it's not the winning, it's the journey type of thing? Sorry, but I don't see that taking subgoals, the smaller steps on the way to the main goal, are any more meaningful than the main goal of the game. Maybe "personal experience" means things like friendship, love... etc.? Very nice, but why are those "meaningful", any more than anything else? Because we like them, they make us feel happy, satisfied? Well, yes, you can take that as your criterion for meaningfulness if you want... but why is that any more justified than any other criterion?

So this little meta-ethical romp does not seem to have come to a satisfactory conclusion. And I think that the reason, one reason, anyway, is that we started by asking the wrong kind of question. How, we asked, is meaningfulness tied to a god? Well... I guess it isn't, and so what? Then we have to look somewhere else, don't we... or ask just what we're talking about, anyway. Is "meaningfulness" a feeling? A goal? Something even useful to talk about? Where does that notion come from? What does it mean? I think that those questions might be more interesting to think about before we start with all the "god" stuff that people have gone around and around with for thousands of years.

Steven Ravett Brown


Not having read Solomons words I can only comment upon your quote and I can understand how you can interpret The Myth of Sisyphus in the way you describe but my own take is that the book already posits God/s existence, after all it was one who put Sisyphus there. So no, evidence for a God/s existence would not change his interpretation of the theory being a vision of the absurd.

However, I prefer to see the story as heroic as to me it presents the existence of freewill in Man regardless of the actions of any God/s. In his walk down Sisyphus is free and to me it shows that even if all our actions were to be determined or pre-destined, we would still have freewill.

Kim Boley

back

Christopher asked:

How does logic prove and disapprove the existence of God?

If you get satisfaction from helping others, dose that make you a selfish person? why or why not?

No. Selfish acts are those where the self is satisfied at the expense of another and his/her interests. They are acts which are detrimental to others. Acts done out of self-interest where the subject has an ulterior motive to an act that benefits others may be self-interested but I don't think they can be termed selfish.

Moreover Chad, the question you ask implies the existence of a pure, autonomous, self-contained metaphysical self, the unified self of consciousness and will. This is found classically in Descartes. However, from Hegel onwards, in Marx, in Husserl and in Heidegger we find the Cartesian view of the self challenged. Perhaps pursuing some of these thinkers will further answer your question or even, undermine it?

Martin Jenkins

back

Jose asked:

Do we really have a "right to have children"? Or is it the other way round, i. e., Do children have a right to have parents? If so, do they have a right to both a mother and a father? These questions have been nagging me ever since the "same-sex couples adopting kids or resorting to insemination" controversy started. Sorry to ask so many questions, but I think they are closely related.

What is a right? Perhaps it is an entitlement conferred by society and if this is so there is no right to have children in England, although I believe IVF is going to become available on the National Health, but you can only have one go and as far I know one go doesn't normally work so that can't be conferring a right. If you were entitled to have a child you should be able to go with IVF as long as you wish. If heterosexual couples don't have a right to have a child then same sex couples wouldn't either.

Just because there is no "right" in the sense of entitlement, this doesn't mean it is wrong.

We don't have a right to have parents. We just do. Whatever our parents are like, we just have to put up with it.

Rachel Browne

back

HongLi asked:

A couple of questions on physical pain:

Do you feel pain when you are unconscious? A common situation would be in surgery. I admit I am not speaking from medical experience, but I hypothesise (perhaps foolishly) that people do feel pain whilst unconscious. Except when they wake up from their anaesthesia, they forget the pain they endured, and move on. Am I somewhat correct? Or is there proof that surgical patients don't feel pain whilst in surgery? I would imagine they have conducted many brainwave tests and concluded from that, that no pain is felt, but I don't believe our knowledge of the brain is complete enough to make such a conclusion.

This thought leads to other questions, such as the nature of pain itself. Most of it is a remembrance, and only during an infinitely small slice of time do we actually 'feel' pain. So consider being sliced by a scalpel. As it glides through skin and flesh, no doubt it would naturally "hurt". But I argue that the pain arises mostly from your thoughts; as you are remembering the pain before, and anticipating the pain after... and wherever the scalpel is up to, its actual cutting of that part only contributes to a minute amount of the pain you experience. E.g. some of you may have been unexpectedly struck with something, perhaps sharp, very fast. It surely would not have significantly hurt at that instant. Even if, after that instant, pain is felt, say from the resultant wound, I would still imagine most of the pain is a combination of memory and anticipation of other instances of pain. Just a thought.

These are good questions, and haven't been answered to everyone's satisfaction. As far as surgery goes, there are three kinds of anesthetics. One makes you unconscious, one is an analgesic, and one wipes your memory. The first is dangerous; unconsciousness can kill you, because it puts you near risk for heart failure and other system failures. So anesthesiologists don't go all the way with it, usually. The second dulls pain, yes, but doesn't usually make you unconscious without doses high enough to be very risky. The third is really nasty... you seem pretty much totally conscious, you feel pain, whatever, but you have zero long-term memory. So, I don't know... 30 seconds later or whatever after you're screaming in agony you've completely, totally, utterly forgotten it. No memory at all. So that's used, much more than I'd like to contemplate, in conjunction with the others, in a very delicate balance, which is why we pay anesthesiologists so much money. So do you feel pain during surgery? Haha... as you can see it depends on what they use on you. What you don't do is remember whatever you felt (in surgery) after the surgery. But you could be feeling it during the cutting as far as all behavioral and physiological and EEG measures are concerned, depending on the ratios of what they're dosing you with. That's why the other drugs are also used, to cut that down.

What if you're to all intents and purposes totally out on the first type of anesthetic above. Well, we can check that with EEGs, and if you're down far enough, and still alive, there are virtually no brain responses to stimuli that would normally cause them, and cause pain, and there are no other behavioral responses. Do you feel pain in those circumstances? No. I mean, what do you want? That's about as good criteria as you're going to get. Besides, we know where the areas of the brain are that cause pain. But what about lighter anesthesia, where there are no behavioral responses, but some EEG? Now here's the grey area... and no one knows. You wake up with no memory of pain, you've made no response... but. Let me put it this way... I'm of the school which thinks that to feel pain you have to be conscious, to some extent; that you can't feel pain (or anything else, or have any phenomenological experiences at all) if you're not conscious. Can there be "unconscious" feelings, pains, etc.? Yes... given that "unconscious" means something like "partially conscious" or "participating in processes which are also involved with conscious processes" or something like that. But again the kicker here is just how conscious do you have to be? Haha, don't ask me... no one, and I mean no one, including anesthesiologists, knows... because one can argue that in case where there is some brain activity, we've felt some pain but don't remember. Well... how do you either support or refute that? Even waking the patient up at that point won't do it, because it turns out that we seem to need some consciousness in order to remember. So if they say they didn't feel pain, one can object, again, that they had enough consciousness to feel but not enough to remember.

Now, you claim that the "nature" of pain is that it's a "remembrance" and an "anticipating". You are not supported by the neurological evidence. The appropriate areas light up with activation when we feel pain. Think about it. An argument of this sort, that we really don't feel some sensation, is applicable to any sensation. So we don't really see, hear, feel... etc.? But that begs the question of what we're having a memory of. If we never feel much, where do our memories, which seem so intense, come from? No, I think that both logic and data weigh against your claim... which is not to say that both memory and anticipation don't play a part, just as they do in all sensation. It's certainly not a black-and-white situation.

I don't know the medical literature on pain, so I can't give you references there. Some of the philosophical literature is:

Baars, B. J. 2001. "The Brain Basis of a 'Consciousness Monitor': Scientific and Medical Significance". Consciousness and Cognition 10:159-164.

Benedetti, F., A. Pollo, L. Lopiano, M. Lanotte, S. Vighetti, and I. Rainero. 2003. "Conscious Expectation and Unconscious Conditioning in Analgesic, Motor, and Hormonal Placebo/Nocebo Responses". The Journal of Neuroscience 23 (10):4315-4323.

Darwin, C. 1998. The expression of the emotions in man and animals. 3rd ed. New York, NY: Oxford University Press. Original edition, 1872.

Herz, R.S. 1998. "An examination of objective and subjective measures of experience associated to odors, music, and paintings". Empirical Studies of the Arts 16 (2):137-152.

Nikolinakos, D.D. 2000. "Dennett on qualia: the case of pain, smell and taste'. Philosophical Psychology 13 (4):505-522.

You could start there.

Steven Ravett Brown


That's a pretty sophisticated bundle of questions, Hong Li! But first things first: It is extremely unlikely that anyone feels pain while they're unconscious. I am assuming (since you refer to an operative scenario) that you have general anaesthesia in mind, and now it is (if you like) an instrumental fact of the body's life that the gases used in anaesthesia "knock out" the transmitters of pain, i.e. disable them from performing their normal functions. So non-functioning nerves + partially knocked out brain translate into a pretty secure non-perceptual condition. Bear in mind, as a supportive argument, that the brain itself contains no pain sensors, so that brain surgery is often performed with the patient fully conscious. This argues pretty conclusively that nerves carry pain signals, and when they're gassed, they cant.

The nature of pain is a very difficult issue to deal with even for experts, but your suggestions in the second batch of questions are very wide of the mark. Let me ask you how you can remember a pain when you are suffering injury unexpectedly and while you're not aware of being injured: as when you step on broken glass in the dark or get bitten by an insect? Further: the delay which sometimes (indeed often) occurs before we feel pain in instances of severe injury has nothing to do with memory, but with a manner of localising trauma devised by your body to ensure that you do not pass out until your brain has at least begun to coordinate suitable defence strategies. In any case, you are really way off with the notion that pain signals are weak and need bolstering from memory: What memory? Of severe pain? Then how perceived??

On the contrary, signal strength is directly proportional to the severity and/or danger to the system of the injury, as well as the sensitivity of the organ being injured, which may differ according to criteria of which we know very little (if anything). You are in fact, confusing something you may have read about the way memory functions: For it is the case, very probably, that a great quantity of the stimuli which bombard us every second of consciousness are referred to memory once your sensorium/brain cooperative has decided that they are virtually identical to information previously collected. Then it is more economical for your brain just to replay the memory, instead of using expensive and time consuming "live" resources to evaluate a run of the mill stimulus. The point here is that the important exceptions to this manner of dealing with daily experience are novelties, surprises and emergencies.

In those cases, the stimuli are passed to the brain "naked", as it were, so that your experience is vivid (and sometimes very confusing) because it is unfiltered. Evidently pain is "emergency" type, often also "surprise" type, and the pain is therefore a signal to you to do something in a hurry. There is a further criterion. Nerves usually require a refractory period after being stimulated; and so one of the other great principles is that they report changes in stimuli, and logically "switch off" when there is no change to report. This is one reason why pain can often be monotonously intense: but this is not because the pain is continuously transmitted, but because nothing in its quality has changed to arouse a nerve bundle to retransmit. Accordingly the "pain switch" (whatever it is) in the brain stays in the "on" position. But plainly this is not a memory function.

The real problem with pain is "what it is". A signal is evidently a form of electrochemical transmission that could hardly be pain-like. It also doesn't carry the sensation from the point of injury to the brain, but only a signal. Maybe this is what lured you down this track. Clearly the signal must be "understood" as something concretely sensory by the brain, which then produces an appropriate feeling. But in what Im describing here for you, there is no intrinsic difference between hearing music or speech, tasting food, smelling a flower or seeing a vase. All these sensation are transmission of essentially similar kinds. Modality determines the type of stimulus and the cortex being addressed, which must all have been calibrated over millennia of evolutionary fine tuning. But to what effect and purpose?

Well, before you conclude, by elongating what I've just written into a scenario of a "black box" type central command, or a "ghost in the machine" or an "homunculus", which is responsible for your "feeling" and actually "produces" it (by what kind of trick?), so that in actuality you don't feel pain or happiness or a nice taste on your tongue, but just imagine or believe or delude yourself into this feeling, I hold that there is a deep-seated logical error in this theory. It is of a kind with the error that tries to "locate" the little man in the box who sees for you, since obviously "you" cant see anything, being the recipient of nothing but electrochemical transmissions . . .!

Well: I think that on the contrary, the ancient, intuitive version of pain etc. was in most of its essentials correct.

You feel a pain in the foot or in your kidney, because that's where it is. And you feel it there not because of some prestidigitation within your cortices (i.e. essentially illusory stuff) or because of magical potions of chemistry sloshing around in your brain, but because the brain constructs for you, while you are awake and conscious, a body map. It is because of this map that you can find the spot on your back that itches, so that you can scratch it, even though you cant see it. Thus the injured site transmits pain signals to that part of the brain to have the information processed where the pain is that you feel, which you could not know unless the brain had first devised this virtual reality inside you, that is made up of every point in your body which is connected to the brain by a sensor. So the pain you feel is not because your brain receives pain messages, but because your bodies nervous system suffers it while the brain is merely called upon to identify and bring to your consciousness the spot that hurts. (Again, in most cases you can find that spot in the dark.)

Whatever factual information we have is available in Ian Glynn, The Anatomy of Thought, in which a discussion of nerves occupies about a third of the length of the book. Mind you, its a totally materialistic theory, but at least it has the virtue of not trying to push you into a particular "philosophy". Antonio Damasios The Feeling of what happens is another fine book on sensation, perception, cognition and especially the role emotions play in this triad. If you really want to know something about the subject, get those books. They're written with the lay person in mind, and they give you what must be regarded to this date the "state of the art", including everything you want to find out about pain and memory.

Jürgen Lawrenz

back

Zenie asked:

I have these questions.

1) Is God's existence necessary for life?

2) Justice presupposes freedom.

3) Is happiness objective?

4) There is no peace without justice.

5) A phenomenon of hope for a better life.

1) No

2) Depends what you mean by "freedom". You can have justice within a prison population.

3) No

4) Not true — again it depends upon your definition of "peace" but you can have "peace" whilst living

in an unjust society, for example, Afghanistan under the Taliban

5) no idea what this question is asking?

Kim Boley

back

Katie asked:

Why is atheism viewed as such a negative thing in today's society?

1. Christianity, a theist religion still to a certain extent, has the upper hand in today's western society. And so is islam in the 3rd world countries.

2. Some people link atheism to immorality, lack of respect to society, anarchy, etc. since atheists usually lack the belief in afterlife and final judgment.

3. Because modern scientific atheism offers a view that is considered offensive by many people, we are not the creatures of god, we, as well as the universe are non intended objects created by random forces, and will end up destroyed sooner or later in the future, all human affairs, technology, morality, love, emotions, etc.. are as unimportant to the universe as the life of a mollusk. Simply because the universe is a non-sentient object being unable to feel or consider something important. We will end up dead, no such magnificent imaginations like heaven, or even hell! Pessimism is becoming — irrationally, I think — liked to atheism.

4. Atheism is viewed by some people as being antagonistic to romantic views, to emotions, to speculation, etc. some people think it is turning the world into a big non-sentient dull machine lacking both purpose and goal. This view is to certain extent expressed in Charles dickens tale Hard times.

Arthur Brown

back

Julian asked:

I'm studying necessity and modal logic, and thinking about whether the world could have been different than it is. I'm not sure if this is really a philosophy or a physics question, but here goes.

Imagine I have a process depending on random, quantum-mechanical effects: e.g. a radioactive decay. I set up a Geiger counter and I measure the number of decays per minute for 60 minutes, and record the numbers.

Now I do a thought experiment. I go back in time and repeat the experiment. Nothing is different, both the equipment and I are the same as before, in the same states. Will I get the same numbers?

I know that radioactive decay is random. It is not predictable, but follows probabilistic laws. To Einstein's dismay, there is no internal mechanism we can observe which is behind the probabilities and which explains them. Yet I find it hard to believe that I will not get the same numbers. Why shouldn't I?

More specifically

— Does it mean anything to say that the equipment is in the same state as before, if there is nothing "inside" which could be called a state?

— Although this though experiment is impossible, are there any real experiments which shed light on the question?

The experiment you've just described is known in the "business" as Schrodinger's Cat. Practically every introductory text on quantum physics talks about it (Stephen Hawking once said, "every time I hear about Schrodinger's Cat, I feel like reaching for my revolver!"). There is also a book by Jonathan Gribbin called *In Search of Schrodinger's Cat", worth reading. Most of these books also discuss further experiments, from EPR to Bells inequalities, to improve your perspective on the matter. I append a couple of remarks to point you in another direction of fascinating research.

But first, I don't quite understand your worry over not having a repeatable experiment on your hands. To say that equipment is in a particular state is probably the issue on which you've cut your teeth in vain. The equipment is not in any state whatever. It is the observer who is in a state, and identifies his own and the equipment's state as one quasi-symbiotic setup. In that combined state, you cannot, by any normal criterion of the meaning of "repeatable" repeat your experiment. The initial conditions defeat you. Radioactive decay is a random phenomenon precisely because the exact initial conditions of even a captive atom would be incalculable.

You must understand that the language being used in physics is the same language you speak every day in your neighbourhood, so that a special mental effort is usually required to rid yourself of the shackles of common usage to comprehend the changes of meaning. Asking for "the same numbers" in any such experiment performed twice or many times in a row is therefore asking for numbers with billions of digits after the decimal point matching each other one for one. In a word, whatever you may mean by "state", it is an intrinsically unique constellation of numbers I am tempted to say, impossible to unravel even for God.

One interesting and often pursued branching line of inquiry is the "possible worlds" scenario. It is arguable that each unique experiment, in failing to register the identical "state" of previous experiments, gives evidence of divergencies in the (sub)atomic texture of events. Again, this is an issue puzzled over in many introductory physics books; but be warned that many are close enough to science fiction to make you wonder on how much science they rely. At any rate, I suggest you pick up any basic physics text in your library and check the index for "possible worlds" and carry on from there.

One of the great philosophers, Leibniz, was knee-deep into such theories, which have recently been reinterred with a lot of fanfare by a number of physicists, e.g. Smolin, Barbour, as well as some fiction writers, e.g. Borges. All are worth chasing up for what they've written on the subject. (And, for what its worth, Pathways to Philosophy has a program of study devoted to "Possible Worlds", if you find yourself developing a taste for the philosophy of "possible worlds").

This is where I might leave it, before I start writing a treatise. Meanwhile: bon voyage!

Jürgen Lawrenz


But think about it... what assumptions are you making when you say you'll "go back in time"? First, you can't go back in time. Second, if you could in some way, say, observe what was in the past, you couldn't alter it. You are assuming that "going back in time" implies that you can literally do so and change things when you do. But there's nothing at all in physics to support that assumption. So if you "go back in time" the way you can do it, i.e., by remembering the situation, then of course everything will be identical and the exact decay pattern will "repeat" itself in your memory. So yes, you'll get the same numbers.

But also, there's a lot of fiddling right now with the whole conception of "hidden variables" and that sort of thing. Bell's Inequality, for example, has been shown (as far as I know) to apply to only a limited subset of physical situations. Further, there has been some work which seems to show that we can look at processes "below" the quantum limits, in a kind of indirect way. And there's always Boehm. As for nothing "inside"... what does that mean? Again, you're making the assumption that some sort of functional epistemological viewpoint is reflected in ontology, a very bad mistake, in my opinion, which people have made over and over. Take a look at this, for example:

Hess, K., and W. Philipp. 2001. "A possible loophole in the theorem of Bell". Proceedings of the National Academy of Sciences USA 98 (25):14224-14227.

Steven Ravett Brown


Whilst not a physicist, my own take on your "thought experiment":

If radioactive decay is random then it is not predictable but if it follows "probabilistic laws" then it must be probably predictable. Otherwise what is the point of these "laws".

If radioactive decay is random and probably predictable then, when using the random generator, we can reliably predict that we will probably generate a random number. So I would say that in your experiment you will have a high probability of getting a different set of numbers generated but you might not, although I would bet my dog on it!

My question to you is "What is it you find "hard to believe" when considering the case where you do not get the same numbers"? or "What is it that makes you think you should get the same numbers?"

My own difficulty is in thinking of a case where you would get the same numbers as, even if we ignore all the problems inherent with the idea of "going back in time", I find it difficult to conceive of a way to restart the experiment and it still not produce different numbers.

The only way I can see to get the result of generating the same numbers is to abandon the idea of "probabilistic laws" and look for something else that lets you explain whatever it is these "laws" explain and allows you to show how the"random-number" generator is not really random but deterministic in some sense. Then when you go back in time you can restart the experiment but with the tweak that you set the deterministic rules to work on the experimental state in just such a way as to produce the numbers you want.

Pretty thin but best I can do.

Kim Boley

back

Mike asked:

This may be a question for a Physics professor, but it does have some philosophical implications too!

Can physical things actually "move" without an external reference to relate its motion to? Let's imagine a universe where there are no other physical bodies (planets, stars, pogo sticks, Big Macs, etc) besides our own body. We have attached to our backside a rocket pack. We throttle-up the rocket and immediately feel the intense acceleration. After a good long burn we disable the rocket. Are we moving? If you say, "Of course you are moving! You felt the acceleration, didn't you?" Then prove it! Movement is defined as a continuous change in position RELATIVE to another physical object. So, without the existence of any other physical objects to relate to, is it possible to move?

Consider this situation: suppose you hold a bucket full of water in your hand, just letting it hang. Now, suppose the room, even the earth, rotates around that bucket. Will anything happen to the water in the bucket? No. Now suppose you rotate the bucket (not revolve, rotate around a central axis) and keep the room and the earth still. What happens? The water climbs the sides of the bucket, right? This seems to be an example where absolute motion is not reciprocal; where you can tell whether it's you or the universe which is moving, doesn't it. I'm still not sure how this is resolved... one proposal I've seen is that this kind of spin, if enough of the universe were moving around the bucket, would cause the water in the bucket to rise. Of course, another thing about this example is that the water in the bucket is accelerating, not moving uniformly (i.e., its motion vector is continually changing in direction, although not in length), and we're out of the realm of special relativity, which is what your question seems to assume, but actually is outside of, and into the realm of general relativity, where gravity as acceleration, time dilation, and various other nonlinear effects start to take place. To relate it to your example, in a continuously accelerating rocket, you would be able to tell that you were moving, because a) you'd experience weight from the acceleration, and b) when you got out of your rocket, after you returned, you'd look around and see that while 5 hours, say, had passed for you, 5 days, perhaps, would have passed for the rest of the universe while you accelerated, turned around, and decelerated back to your original starting point. This is the classical "time dilation" effect of general relativity, and there are other effects, e.g., a change in shape and increased mass. Now, the rest of the universe could not have been moving, because the time dilation effect is asymmetric, as you can see. If you had sat still and everyone else shot away from you into space, then returned, then everyone else would have had the dilation, while you would not. Things are symmetrical so long as acceleration is zero.

The question is, why does this happen? Is there an effect of the total mass of the universe on all its components (so that if the whole universe rotated around the bucket, the water would rise)? Is there a "fabric" of space (despite Michelson & Morley) which is being "pushed against"? Is the speed of light limitation causing the particles in your body, etc., to require more and more energy to accelerate as their speed increases (i.e., the energy has to "catch up" with them)? Well... none of these are employed today. The explanation of these effects is a geometrical one, involving the curvature of space and time combined: space-time curvature, and the math is rather complex.

Here are some websites that might help:

http://en.wikipedia.org/wiki/General_relativity

http:/math.ucr.edu/home/baez/RelWWW/wrong.html

http://www.astro.ucla.edu/~wright/relatvty.htm

Have fun!

Steven Ravett Brown


That's a very good observation, Mike! And at least in philosophy your question belongs among the oldest to have been discussed. I call it the "Paradox of Motion", for the same reason you do; and among the very first philosophers there were two, Heraclitus and Parmenides, who acquired "immortal fame" by proposing exactly opposite solutions. Very roughly, the first proposed that motion is the fundamental state of the universe anyway and that nothing ever sits still. The other said the whole thing is a sophisticated illusion, that we live in a static universe (single block) and movement is a play of light and shadows on our senses. Well, there's a little more to it than I've given here, but in any case theirs was not the last word ever spoken on the issue. It may not be beside the mark to mention in passing that in science the question tends, unfortunately, often to get shoved under the carpet.

In addition, a much deeper issue is, as it were, attached to it. As long as we speak about matter only, it could be argued that motion or no motion amounts pretty much to the same thing. If no conscious intelligence is there to register the motion, by what logic might we wish to say yea or nay? But there does exist a conscious intelligence; and now the deep issue I alluded to is this.

Imagine you are in a pitch black room which is also asensory in other respects. Suddenly you see two billiard balls, one white, the other red, stationery in the middle of nothing. (Assume their light source to be internal so that you still cannot see anything else.) Now you would, as a matter of course, assume that these balls are situated on a billiard table. So be it, even though for you it is mere conjecture. But all of a sudden, the white ball rolls rapidly towards the red and collides with it. In the result the white ball is deflected from its course and the red one takes off and disappear a fraction of a second later lets say into a pot. A question arises from this scenario which offers testimony either to an ingrained prejudice or lets you in on a fundamental truth. You choose.

It is this: if you were asked, what moved the white ball, you would without a seconds deliberation urge some cause: whether a third unseen (black) billiard ball, or an unperceived tilting in the table, or a sudden influx of directed air pressure etc. With any of these causes, however, you would immediately understand, without being prompted, that they in turn demand similar causal explanations; that tilting the table (for example) might be due to a crack in the floor, which occurred because of an earthquake, etc etc. This way you could, if that was your frame of mind, continue through to the Big Bang. Once there, of course, you're not allowed to ask for further causes: for this is the great deus ex machina that answers all questions.

(Mind you: the white ball hitting the red in such a way that it rolls straight into a pot is a bit much of a coincidence too!).

If, on the other hand, you were informed that, also unseen by you, a billiard player happened to be present and used an ordinary queue to push the white ball, then you would know the cause of motion of the white ball. But you would not enquire after, indeed hardly feel a need to do so, the cause of the queues motion. The billiard players motion does not require a causal explanation.

So there is an answer of a sort in all this thought experimenting. The motion of matter demands an explanation, and except in the trivial sense of supposing "impetus" or "momentum" to be conveyed from one object to another, we have none. All the standard explanations like gravitation, heat, electrical potential etc. simply beg the question or push it up from one explanatory level to the next, where the same question awaits you again. By contrast, agency requires no explanation beyond assuming a "free will", and this applies to bacilli as much as to us. In consequence some philosophers, aware of this fundamental discrepancy in causal chaining, have proposed either that the same causes operate hidden from us in organic matter as well, while others maintain that agency precedes matter, that without it, we could not explain the simplest facts of the universe.

If you're interested in pursuing this (but be warned it is a difficult as well as contentious subject matter), you might sample Schopenhauer (The World as Will and Imagination) or Leibniz (New System) together with some secondary literature (check Janaway or Magee on Schopenhauer, and the Oxford annotated edition of the New System), for versions of "agency first" idea; or you might delve into Barbour's The End of Time. And of course, let no-one discourage you from reading Heraclitus and Parmenides, who started this ball rolling (lots of cheap editions, e.g. The Origins of Scientific Thought by G. de Santillana in Mentor, or Presocratic Philosophers by Jonathan Barnes in Penguin).

Jürgen Lawrenz


Newton's law: to each action there is an equal and opposite reaction. Here you are moving relative to the contents of your your rocket exhaust (which is a physical object besides your own body). In practice, I think you will find it impossible to construct a thought experiment in which the traveller is both (a) accelerating and (b) the only object in the universe.

David Robjant

back

Edward asked:

The human mind is not capable of the infinity concept. So can anyone really define Infinity?

Cantor appears to have done a pretty thorough job of defining it in the field of Mathematics so at least one human mind has been capable of the infinity concept.

Kim Boley

back

Christopher asked:

Do any philosophies deal with the nature of the changes that take place in the individual's definition of themselves and the universe, as the individual moves from infancy to become a member of society?

Do these philosophers deal with nature of the issues that are dealt with, the sequence of these issues? The development of these ideas first as thesis and then as anti-thesis?

Let me explain in part:

1. The infant begins with the concept that it is the universe, then learns that it is dependent.

2. The infant begins with no definitions and learns definitions.

3. The infant begins with isolation and then experience unity.

These three issues define the inside and outside of a triangle the central issue at this stage determining which side of the triangle the infant's tendency will rest is trust/ mistrust.

What I am working on in a philosophy that include, as part of it, the development of a series of thesis/anti-thesis triangles at different stages of individual development. After a series of these processes are passed through the theory sees the development of branches of philosophy as thesis or antithesis formations.

Yes. But one problem with your question is that I don't know a) your background, and b) what you mean by "philosophies". So I'll give you a list of readings, but if you know anything about child development you've probably read them. Nonetheless:

Cowan, N., J.S. Saults, L.D. Nugent, and E.M. Elliott. 1999. "The Microanalysis of Memory Span and Its Development in Childhood". International Journal of Psychology 34 (516):353-358.

Dolgin, K. G., and M. Azmitia. 1985. The development of the ability to interpret emotional signals — what is and is not known, edited by G. Zivin. Orlando, FL: Academic Press, Inc.

Fogel, A. 1985. Coordinative structures in the development of expressive behavior in early infancy, edited by G. Zivin. Orlando, FL: Academic Press, Inc.

Gopnik, A., C. Glymour, D.M. Sobel, L.E. Schulz, T. Kushnir, and D. Danks. 2003. "A theory of causal learning in children: Causal maps and Bayes nets". Psychological Review In Press.

Gopnik, A., and A. Meltzoff. 1987. "The development of categorization in the second year and its relation to other cognitive and linguistic developments". Child Development 58:1523-1531.

Gopnik, A., and A.N. Meltzoff. 1998. Words, thoughts, and theories. Edited by L. Gleitman, S. Carey, E. Newport and E. Spekle, Learning, Development, and Conceptual Change. Cambridge, MA: The MIT Press.

Gopnik, A., and D.M. Sobel. 2000. "Detecting Blickets: How Young Children Use Information about Novel Causal Powers in Categorization and Induction". Child Development 71 (5):1205-1222.

Grady, J. 2000. "Cognitive mechanisms of conceptual integration". Cognitive Linguistics 11 (3/4):335-345.

Hickling, A.K., and H.M. Wellman. 2001. "The Emergence of Children's Causal Explanations and Theories: Evidence From Everyday Conversation". Developmental Psychology 37 (5):668-683.

Malatesta, C. Z. 1985. "Developmental course of emotion expression in the human infant." In The development of expressive behavior, edited by G. Zivin. Orlando, FL: Academic Press, Inc.

Matthews, G.B. 1987. "Concept formation and moral development". In Philosophical perspectives on developmental psychology, edited by J. Russell. Oxford, UK: Blackwell Publishers.

Piaget, J. 1959. Judgment and reasoning in the child. Translated by M. Warden. Edited by C. K. Ogden, International Library of Psychology, Philosophy and Scientific Method. Paterson, NJ: Littlefield, Adams & Co.

Piaget, J. 1971. The construction of reality in the child. 2nd ed. New York, NY: Ballantine Books, Inc.

Piaget, J. 1971. Insights and illusions of philosophy. Translated by W. Mays. New York, NY: The World Publishing Co.

Stern, D. N. 1985. The interpersonal world of the infant. New York, NY: Basic Books.

Wimmer, H., and J. Perner. 1983. "Beliefs about beliefs: representation and constraining function of wrong beliefs in young children's understanding of deception." Cognition 13:103-128.

Zelazo, P.D. 2004. "The development of conscious control in childhood". Trends in Cognitive Sciences 8 (1):12-17.

Zelazo, P.D., U. Muller, D. Frye, and S. Marcovitch. 2003. "The development of executive function in early childhood". Monographs of the Society for Research in Child Development 68 (3):vii-155.

I highly recommend Piaget (and what I have of him here is a tiny selection of his writings), Gopnik, and Zelazo, given what you're saying above.

Steven Ravett Brown

back

Single asked:

Are there any theories that Plato, Aristotle and Socrates have that are connected to the Caribbean?

Yes: Socrates spoke through the mouth of Plato about an island state called "Atlantis" in that region of the world; and when Aristotle came to discuss Plato's political theories, he said something like "Plato invented Atlantis only to destroy it again."

Jürgen Lawrenz

back

Joanna asked:

I was wondering if you have any theories on list making? Not shopping lists but the act of making a to-do list and then never getting round to doing the things on the list. Do you agree that by writing things down or talking about them you feel like they have been resolved? Do you have any idea why this could be?

Take a look at "Mind Maps" by Tony Buzan http://www.imindmap.com for a list making theory.

Yes I do agree that sometimes writing things down or talking about them does sometimes resolve them. Why this could be, seems to me tied up with the list-making idea, in that organising your thoughts in these ways allows you to take action.

Kim Boley

back

John asked:

Hello. Thank you for this great service.

My question is concerned with understanding why the experience or vision of accepted gods like Jesus, Buddha, Mohamed, Oden etc. does not occur in pagan cultures or to specific person(s) who do not believe in such a god. And if it does occur, why does it? I can only think of one documented example of this in history. (I'm sure there are probably many more?)

I try not to answer questions like this any more, because I've done it so much... but I'll go for it. Look, there are no cultures whatsoever where people do not have "visions", "experiences", "states", etc., etc., which are usually termed "mystical" or "religious". None. Zero. Those experiences are universal to human beings. But, the exact content of those experiences varies according to the culture, i.e., according to the beliefs of the people having them. If you believe in something strongly enough, you will experience it. I'll repeat that: If you believe in something strongly enough, you will experience it. If you believe in the god Thor, and you have religious experiences, you will have visions of Thor which seem utterly real to you. If you believe in Mary Magdalene, you will have visions of Mary Magdalene which seem utterly real to you. If you believe in Krishna, you will have visions of Krishna. If you believe in fairies, you will have visions of fairies. If you believe in flying saucers... etc., etc., etc. What I fail to understand is why people don't see this and say, hey, maybe there's something common to all this which we are producing from ourselves. Because virtually all these religious experiences are utterly incompatible with each other. Krishna and Thor cannot exist in the same universe. Yahweh and Buddha imply totally different cosmologies, characteristics, afterlives... And on, and on, through the thousands of different religions that humans have believed in since the dawn of... whatever. Why, oh why, do I need to go on about this? Isn't it glaringly obvious?

Well, fortunately, there are some people who have asked this question and are actually researching the basis of these experiences. Here are some readings:

background:

Eliade, M. 1961. The sacred and the profane. Translated by W. R. Trask, The Cloister Library. New York, NY: Harper & Row.

Frazer, J.G. 1951. The golden bough: a study in magic and religion. Third ed. New York, NY: The Macmillan Company.

religion:

Alper, M. 2001. The "God" part of the brain: a scientific interpretation of human spirituality and god. Brooklyn, NY: Rogue Press.

Azari, N.P., J. Nickel, G. Wunderlich, M. Niedeggen, H. Hefter, L. Tellmann, H. Herzog, P. Stoerig, D. Birnbacher, and R.J. Seitz. 2001. "Neural correlates of religious experience". European Journal of Neuroscience 13:1649-1652.

Giovannoli, J. 2000. The Biology of belief: how our biology biases our beliefs and perceptions Rosetta Press, Inc.

Langdon, R., and M. Coltheart. 2000. "The cognitive neuropsychology of delusions". Mind and Language 15 (1):184-218.

Laski, M. 1990. Ecstasy in secular and religious experiences. Los Angeles, CA: Jeremy P. Tarcher, Inc. Original edition, 1961.

general:

Hines, T. 1988. Pseudoscience and the paranormal: a critical examination of the evidence. Buffalo, NY: Prometheus Books.

Schick, T., Jr., and L. Vaughn. 1995. How to think about weird things: critical thinking for a new age. Mountain View, CA: Mayfield Publishing Company.

Shermer, M. 1997. Why people believe weird things: pseudoscience, superstition, and other confusions of our time. New York: W. H. Freeman and Co.

Young, A.W. 2000. "Wondrous Strange: The Neuropsychology of Abnormal Beliefs". Mind and Language 15 (1):47-73.

Please read some of these.

Steven Ravett Brown

back

Ridhi asked:

Please tell me how can I disagree with Meditations 1 and 2 written by Rene Descartes.

Read them and if you cannot disagree with anything or see any possible problems with the ideas then you might as well act as though you believe them and explore what they can bring to your understanding of the world. This may also cause you to disagree with them:)

Kim Boley

back

Matt asked:

I'm in the middle of a debate with someone who claims that, using Kant's categorical imperative, homosexuality is wrong. He says that you can easily will everyone to be heterosexual, but you can't will everyone to be homosexual because it would lead to the extinction of the human race. I find this a pretty stupid argument, to be honest with you, but I can't exactly say why. His way of saying it seems awfully clumsy and contrived to make his point. Also, I was thinking that I might be able to use the second formulation of the imperative to say that he can't use heterosexuals merely as a means to propagate the species. Anyway, I'm thoroughly frustrated and could definitely use a hand here.

Yes. This is a severe problem with Kant's ethics. Here's the situation:

Kant was coming from a time where there were basically no sciences as we know them, and people were collecting lots of little bits of data, and not being able to put them together into coherent pictures. The big exception was Newton, and Kant wanted to get philosophy on the same footing. Well, Newton developed a system of physics which started from first principles, i.e., Newton's Laws, and from which you could then derive the behavior of any (he hoped) physical system. But those laws were for ideal objects; point masses, frictionless surfaces, objects moving in a vacuum... etc... "ideal" objects, you see? But they worked pretty well for the real world, at least as far as measurements went in those days, and everyone could see that you had to fiddle these other factors to make the ideal equations work for real masses, and so forth, but that they were nevertheless the correct ideal equations.

So Kant wanted the same for philosophy. And what he did, then, was to try to derive the most abstract ethical principles he could. Then he thought once that was done, everyone could just work down and fiddle a little, and it would work out like Newton. Well. Not a bad idea, really... but when you look at what he came up with it's a bit silly, in my opinion, anyway. There are two basic principles, sort of... although they are intended to be aspects of one principle. There's "act as if everyone else would do the same thing"... a kind of way to fiddle out the detailed answer, I think; and the really general, abstract, ideal principle is that the willed action cannot be what might be termed "self-contradictory". That is, if you think you can steal, and you ask, "what if everyone stole?", and you find that if everyone stole society collapses, or something like that... then you have to say that stealing is in a way self-contradictory, because it results in a condition or a society in which stealing is impossible... because the society in which you asked the question could not exist if you answer the question in the affirmative. You see?

Basically, he's trying to show that unethical actions are in a rather bizarre way contradictions, or self-defeating, because they destroy the circumstances under which they originally start. An ingenious way to make ethics logical, right? Right. It's so full of holes you could drive a truck through it. For one thing, what about people who need to steal to survive, in some sort of exceptional circumstances? Well, then, you either have to say that they can't, period, or you have to say, well, we have to take their circumstances into account, and if everyone else stole, society would collapse... etc. Of course you can see the problems, here... just what do we end up with, finally, as situations which are "normal", so that we can't steal in them? And what criteria do we use to decide those criteria? Whoops... And all this is aside from the little problem of demonstrating that this self-defeating dynamic will actually happen, haha. Believe me, his three Critiques are much more impressive. So sure, if everyone were homosexual, there'd be no human race in a couple of generations... but everyone isn't homosexual, nor is even a majority likely to be (I just read about a study on homosexual sheep... yes, well, a useful study, actually... and about 8 percent of rams are homosexual; consistent with what's found in humans and other animals, I believe).

Steven Ravett Brown


The Categorical Imperative is a guide to moral action so it cannot be used to say that being a homosexual is wrong, only that a homosexual act is. It is an objection to Kant that you can universalise anything, but I don't understand it. We cannot universalise anything as rational beings. I don't see how the principle that you should commit a homosexual act can be valid as a law or an "objective principle valid for every rational being". Would a rational being hold the principle that everyone should commit a homosexual act? I would say that it was an individual's subjective principle of action which does not go on to the stage of being objective and becoming moral law. A subjective maxim is not moral law but "a practical rule determined by reason in accordance with the conditions of the subject (often his ignorance or again his inclinations)".

As to the argument you are trying to develop against using heterosexuals as a means to propagate it will probably fail. Kant says "Act in such a way that you always treat humanity... never simply as a means, but always at the same time as an end". If heterosexuals are used (by whom??) to propagate the species this is to treat persons as both means and ends.

Rachel Browne

back

Subal asked:

Krsna is God. This knowledge is self justified. I know Krsna is God because the Vedas say that Krsna is God and the Vedas are correct because they where spoken by Krsna.

Somebody who doubts that Krsna is God may say that it is incorrect to say, 'Krsna is God because Krsna is God.' Because it is incorrect to say that something is the cause of itself.

Why is it incorrect to say that something is self caused?

Um.... you do realize that every other religion is justified in the same way by its believers, using its bible? And that Hinduism and, say, Christianity propose radically different explanations for... well, everything?

Steven Ravett Brown

back

Rosanne asked:

I am trying to remember the name of a philosopher — in early 1950's — on the docks of San Francisco.

Eric Hoffer, author of The True Believer: Thoughts on the Nature of Mass Movements; The Ordeal of Change; The Passionate Mind... etc..

Steven Ravett Brown

back

Kay asked:

I'm having some trouble after reading Nozick's Love's Bond. Is Nozick really claiming that love cannot be for characteristics because such a love would be conditional and make the beloved replaceable? If so, I don't follow his argument because isn't love conditional, and don't people "trade up" all the time?

Is it really true that all people "trade up" all the time? If it were, then perhaps Nozick's conclusion would be that this is a loveless world. He intelligibly contrasts love for a person with love for, say, blonde hair. You want to say that love for qualities is just love, and as much love as there is. One might be tempted to ask questions about your experiments and sampling here, but nearer to the point, what you are engaged in here is 'persuasive definition'. That is, you have this word 'love', are wondering what to do with it, and you make your proposal. Nozick is doing the same. You then claim that a problem with Nozick's proposal is that his definition of 'love' can apply to no relationship. What I'd say to that, on N's behalf, is: how on earth would you know that? I suspect you have a complete rival theory of human nature here, rather than a specific empirical objection to Nozick.

David Robjant

back

Margaret asked:

Wittgenstein wrote something along the lines of, "If we had never heard of the word love we might never feel it." Can someone help me find the reference? Also... in the nominalist/ realist debate, is Wittgenstein a nominalist?

If you spoke another language this would be easy to answer. Have you ever experienced an emotion that you didn't have the words to explain? If we call an animal by the name we've giving it, does it know who were calling? Does something come into existence by us naming it or does it already exist?Did homo erectus feel love even though they didn't name it? In my opinion, even with the English language there are still not enough words to describe the different kinds of love, so do they exist?Yes.

Ryan Burton

back

Alice asked:

I really like this guy. I used to go out with him and now I have fallen for him again. but he's not very flirty. I think he likes my friend but people say he prefers me to her. I really like him a lot but I keep on denying to people that I like him. He can be a bit nasty to me sometimes but I can be the same back to him. How do I know if he likes me?

Well Alice, if knowledge is a priori, you will already know. Introspect and recollect the answer. If knowledge is a posteriori — acquired by experience — then you will have to ask the person. If you believe him, you will know the answer.

Martin Jenkins

back

Cliff asked:

Is the glass half empty or half full?

If you drank from the glass when it was full, it's half empty.

If you fill it half way when it's empty, it's half full.

Is this like a 'which came first' philosophy question? i.e. the chicken or the egg?

If you are trying to puzzle us with those questions, I think you can do that even better with this paragraph of the Chinese 'tao te ching', give it a look!

The Way (Tao) is like an empty vessel
That may yet be drawn from
Without ever needing to be filled.
It is bottomless: the very progenitor of all things in the world...

It is like a deep pool that never dries. I do not know whose child it could be. It looks as if it were prior to God.

Arthur Brown

back

Dave asked:

Hey, I wanna start off by saying I'm glad I found your site. I wanna hear what you have to say to a few of these ideas. This is all pertaining to life after death by the way and I often get stuck thinking these few things.

There seems to be a sense of logic in me thinking that if we die and all fades to black, with no memory of our life in fact...that we couldn't be alive right now. The only similar analogy is to a dream that we cannot remember. We've all had nights we awake and cannot recall all the dreams we've had, when it seems like as soon as your head hits the pillow at night it's then suddenly morning and only one minute went by. If those dreams are never remembered they hold no place in time. The dream was only in YOUR head, thusly "you" would be the only one who could possibly remember it. If that dream goes unrecalled until the end of that life, then I suppose it never occurred...it held no place in time and had no remembrance so it might has well of never been.

I find that whenever an idea seems stuck, it pays to write down what seem to be the significant points and the tentative conclusion(s). I usually have at least a vague idea of what the conclusion(s) might be, because otherwise I would not be motivated enough (excited enough or concerned enough) to keep thinking things through. I am not entirely sure where you think your argument is heading but I have attempted to write down and comment on your main points and conclusion.

Summary of your main points:

1. A forgotten dream does not matter and is not real

2. A forgotten life would be like a forgotten dream it would not matter and would not be real.

3. This life does matter and is real.

Conclusions:

4. This life is not forgotten

5. We must survive death otherwise this would be forgotten

Let's first look at the idea of something 'mattering' or 'being important'. It seems to me that things can matter to varying different degrees at different times. For example, when you get a pain in your tooth, the idea that this might lead to a more excruciating pain before long matters, when you actually experience the excruciating pain it matters very much, days later when you only have the vaguest memory of the event it doesn't matter. It also seems to me that one thing can matter in varying degrees to different people. For example, at the time when your toothache matters to you, it will not matter in the least to the millions of people who know nothing about it. Therefore, to be exact we have to qualify 'matters' not only by quantity but also by 'to' and 'when'. For example, the toothache mattered a great deal to me when I was experiencing it.

How should we qualify "matter' in your first point? Our dreams, especially our forgotten dreams, do not usually matter to other people. We also tend to believe that a forgotten dream could not have seemed important to us when we were dreaming it because otherwise we would have remembered it. Therefore, 'my forgotten dream' is an example of something that has never mattered or been important at any time to me or anyone else.

Now let's consider the point 'a forgotten life would be like a forgotten dream'. Certainly they are both forgotten. However, it seems to me that the similarity ends there. A forgotten dream has probably never mattered or been important at any time to anyone, but I don't think that a forgotten life is anything like that. When someone has completely forgotten a dream we can reasonably assume that the dream didn't matter to them, but when someone has forgotten their life we cannot assume that their life did not matter to them. Indeed when someone has forgotten their life we usually assume that their brain is not functioning properly. If there comes a time when my brain is no longer functioning properly and I forget my current life, it would be wrong to assume that my current life does not matter to my family, my friends and myself now. Therefore, in regards to what matters and is important, forgetting a dream seems to me nothing like forgetting your life. Unlike your dreams, your life is not the type of thing can become unimportant all the time to everyone just because you forget it. There does seem to me to be an inconsistency between points one, two, and three. One says dreams can be unimportant, three says life is important, and two says where importance is concerned dreams and life are the same.

It also seems to me that a forgotten life would be nothing like a forgotten dream if we had an after life and the quality of that after life depended on our actions in this life.In that case a forgotten life would have a huge impact on our future life and would be nothing like a forgotten dream that has no impact on our future

The final conclusion is "We must survive death otherwise this life would be forgotten".

It seems to me that we do not need to survive death in order not to forget our lives. Suppose that when I die I cease to exist, and therefore I am no longer around to remember my life. I don't think that in that case I will have necessarily forgotten my life. Claims that are perfectly reasonable about live people are not always reasonable about people that do not exist. For example, "If John is not in my room then John is somewhere else", is a reasonable claim if John is alive. However, if John does not exist then the claim is not reasonable. Similarly, "If John does not remember important events that happened to him, then John has forgotten them", is a reasonable claim if John is alive. However, it is not such a reasonable claim if John has ceased to exist. He might have remembered the events right up to the point where he ceased to exist, and have never forgotten them. He does not remember the events, not because he has forgotten them, but because he does not exist anymore. Therefore, even if all three of your points and the first conclusion were true, the final conclusion could be false.

I hope that my comments help to get your thoughts flowing again.

Wendy Weidel

back

Hopx asked:

What is the significance of Tycho's model for whether Galileo's model should have been judged as fitting the orbits of the planets? I don't see any negative evidences in Tycho's model.

What is wrong with the model?

Tycho was the most accurate observer of the heavens before the invention of the telescope, and his data as exact as they can be made by looking with the naked eye and measuring instruments. Therefore his tables are not in question. His measurements of the famous comet's trajectory demolished the thousands-year-old theory of a division between the sub- and superlunary sphere. That was a hard nut to swallow, although most people came around in short order to accepting the implications. The problem lies with the model of the solar system which he devised for the benefit of those who were scared of Copernicus' system. It was meant to be a compromise between Ptolemy and Copernicus, but it was a silly idea. Tycho left the Earth in the centre, circled by Mercury, Venus and the Sun, with all the outer planets revolving around the Sun. I'm sure even you will see at once how absurd a proposition this is if you draw a graph. (Hint: where is Saturn when it's nearest Earth in its orbit?). Galileo made mincemeat of it in his book on the World Systems; but the Church had come to a sort of accommodation with it and the Pope had explicitly asked Galileo not to mention it. In short, the issue was not scientific alone, but religious and political. Galileo was a hot-tempered, belligerent man and couldn't stop himself, despite having promised.

But you realise, I hope, that Galileo's model is none other than the Copernican. So I'm not altogether sure what your first sentence is all about, unless you mean that Kepler had already derived from the Tychonic tables the world-shattering news that planetary orbits are elliptical, not circular (as Copernicus still believed). But I don't think this was a problem for Galileo. So I hope, anyway, that the above tells you what you want to know.

Jürgen Lawrenz

back

Johann asked:

Premise: This argument is valid.
Conclusion: I am the Pope.

Am I? If the premise is false then the argument is valid, and thus the premise is true and the argument sound. So it seems that I am, therefore, the Pope.

If the premise is false, then it is false that the argument is valid, and you are not the Pope. You have not set up a paradox correctly.

The following:

1) This sentence below is NOT valid
2) I am the Pope.

and

1) This sentence below IS valid
2) I am the Pope.

are NOT paradoxes. The first is simply true, the second is simply false.

Try this one:

1) the sentence below is true
2) the sentence above is false.

You need self-referentiality to set up a paradox. You could add Popes or whatever to the last one if you want.

Steven Ravett Brown


Whether an argument is deemed formally valid has nothing at all to do with whether anything it says is true, but only has to do with whether the various things that are asserted in it are connected in a proper way. Conceivably and for the sake of argument, Johann's one premise argument passes that test, and if it does, then what we are saying is that any conclusion whatsoever could have been validly adduced from this initial premise. Now, it is important to remember that with a valid argument, you can put garbage in and get garbage out. This is because validity is entirely to do with the process from input to output, and not to do with the quality of either inputs or outputs.

Soundness is an entirely different matter. For an argument to be sound, it must be not only valid in its proper connection of premise(s) and conclusion, but also derive its conclusion from a true premise. For something to be a sound argument, you need to take sure to put truths in at the front end of a valid argument. Soundness is validity plus true premises.

Validity and Soundness properly distinguished, we can look again at Johann's two clause central assertion. Johann asserts:

"If the premise is false then the argument is valid..."

Well, we can accept this because we accept that the argument is valid, and that this is so irrespective of whether the premise in it is true or false.

But Johann then continues:

"...and thus the premise is true and the argument sound."

This is a wrong move. From the fact that the argument is valid, nothing whatsoever follows about the truth or falsity of the premise. Just because the argument is valid, it doesn't follow that the premise is true or that the argument is sound.

"So it seems that I am, therefore, the Pope."

No, Johann isn't the Pope. And he hasn't done anything to show that he is either, because his argument is confused about validity and soundness.

David Robjant


Formal logic is abstract and needs to be given content. In his book Logical Forms, Mark Sainsbury suggests that for an argument to be valid in ordinary language it must be "useful" so that is not enough for propositions to be true or false for validity. Your argument is not useful at all and there is no relevant connection between the premises. You would find Sainsbury's whole book very useful, in fact. He shows differences between ordinary language arguments and the workings of formal logic.

Otherwise put, by Avrum Stroll in a paper "Broadened Logic": An argument should be "cogent" as opposed to "nonsense" or "absurd". As ordinary language users we just know what is nonsensical and absurd, and we also have prior knowledge as to what is useful. Stroll says that traditional logic has been "acontextualised" by Russell and Frege and contains no information about the world. As humans beings we acquire knowledge about the world which has to be used as content in an argument in a way which makes sense.

Logic might not reflect the way we think at all. Experience in the world makes a difference to the inferences we are able to draw and formal logic is rather artificial as a way of describing natural inferences. "The kinds of logic described by logicians simply seem irrelevant to normal individuals. We do not construct truth tables and look up the result: We do not use formal rules of inference" — this is a only a statement made by Howard Gardner in The Mind's New Science — but you could read the whole book to check out the detailed way in which he backs this up.

When Aristotle introduced the idea of a syllogism, he didn't have as much knowledge of the way the mind works as we have today. I have always found the idea that logic underpins our use of language and form of argument highly persuasive, but it seems that it is time to rid ourselves of this idea.

Rachel Browne

back

Dave asked:

How good or bad a case does Nietzsche make for the Death of God?

I believe he makes an excellent case. Although existentialism is much more than just proving that God is dead. Existentialism is about freeing man so that with personal freedom the issues of personal responsibility and conscious choices can be discussed. Actually there is an old preconception about the death of God in Nietzsche. He never meant to kill God so he could be free of God and so be able to free man itself. His vision is beyond atheism. He does not ask that we do not believe in God — atheism — but we should feel how dangerous it would be to believe, we should feel this danger as a threat against life itself. In the Anti-Christ he says: "if someone demonstrated to us the existence of the Christian God, then we would believe in Him even less". The so called case of the death of God is so a much complex affair. Nietzsche has much higher goals than killing God. He wanted to kill of any possibility of an ideal world being the world that focused the attention of human minds. In a way it was so, since Plato, and mainly Hegel. The new man, the Super-man, is free from having to believe in the ideal world, and in that way is free from superstition and believes only in the natural body and forces of nature.

It's the end of a cycle. Before religion, man was alone, after religion man is once again alone. Nietzsche says: "there no longer is a God". This sentence confirms what I just said. The new man should never forget his past of struggles. The "lies of the church" play an import role in the new destiny of man, as his dark past and as the start of the "noon" of the new age. Man is alone, as a clean slate, ready for a new beginning.

The final warning of Nietzsche was that a new idol could replace religion, and that idol would be technology. One can only wonder if sometime in the near future someone can dare to kill technology.

Nuno Hipolito

back

Donneth asked:

1. Levinas distinguishes between "desire" and "need". what is the difference? Why should DESIRE be a metaphysical moment?

2. The Face is the situs of ethical. Would you explain in your own terms Levinas' phenomenology of the Face and reflect on why the Face-to-Face checks the 'egology' of the life of the Same?

3. WHat is the Same and what is its relation to enjoyment?

The difference between need and desire, simply put is that needs can be fulfilled, desire cannot. For Levinas there are two ways to talk about human life: one is as separate, individual, egocentric and independent, the other is as transcending towards what is Other and the good. The first dimension is what he calls the 'Same'.

This is the realm of the ego's grasp of the world, understood as consciousness' ability to incorporate "actually or potentially, that which lies outside it." According to Levinas, this movement of the suppression and possession of all otherness and difference into theory, knowing and being has been the predominant feature/ characteristic of the western philosophic tradition and leaves no possibility open for transcendence.

However consciousness does not exhaust the meaning of the Same for Levinas, there is also in the movement of the same an enjoyment of the world, which does not take the form of representations, but are lived as sensibility, at the level of the body. The realm of the Same is steeped in need. Needs are characterised as a lack that must be satisfied. Hunger is a prime example, hunger is experienced as a privation of sustenance, satisfying this lack however is not just a matter of finding nourishment. Eating, consuming is a matter of enjoyment, it is a kind of folding back on oneself where the world is absorbed by me such that ultimately only I exist. This world of the same is foremost an economy where there is a balancing of intention and actions, a coinciding of lack and fulfilment.

Desire on the other hand is structurally very different. It indicates the movement out of this ego-based economy, out of the world of the same, towards the transcendence of the Other. Whereas needs can be fulfilled, desires are insatiable. Desire is what remains when all the needs and wants of the ego are met, not in the sense of a 'there is more I can have' but more of a sense that what I have may not be the most valuable. That what is most valuable is what escapes (essentially) any and all encompassing, any and all economy. Desire is desire for what is different, outside, elsewhere. Desire is the desire for transcendence. Rather than a folding back on oneself it is a movement of extreme unfolding, so extreme in fact that there is no return to what is familiar and the Same. Ultimately it is desire for the Other. An Other who when we encounter him/ her reveals a world to me a new world a world that I am no longer justified in claiming as my own. A world in which the movements of the Same no longer suffice in order to make sense of. Why? Because the Other is a very strange experience as both imminence and transcendence, in other words as a Face.

The notion of the face is as you recognise very difficult and slippery. I think the latter feature is deliberate on Levinas' part. He is trying to capture in a detailed way the basic thought that when we are with someone, just by the simple fact that there are not ourselves there is something always out of our reach, the other is both with us and not with us, it is this slippery duality that leads to the difficulty in understanding Levinas' discourse. At one point, Levinas even says, "I do not know if one can speak of a phenomenology of the face, since phenomenology describes what appears" (Ethics and Infinity p83).

The face then is not the physical features of the eyes, nose, mouth. Levinas goes on to say that "The best way of encountering the Other is not even to notice the colour of her eyes!". The face is what escapes any description of the other by the self; the face is what points to that which cannot be brought into the confines of the ego's world. The face is transcendence. And yet the face is what is most immediate, most imminent, most real, because the face presents itself as 'destitute' as 'naked' and 'exposed' calling for help, my help to give up the coat on my back to cloth her, to give up my food to feed her. Before this appeal, I had never had to consider the way I lived my life. I just did as I pleased. The Other breaks into this status quo disrupting the pleasant flow of self-containment. When faced with the Other, however, there is a question that strikes to the very foundations: Do I have a right to exist? Does the fact that my living my life mean that someone else is suffering, if so what can I do? Of course I may not do anything I am under no compulsion to act, But then I have ignored the Other's call for help. If in fact I do help the other then ethics and the Good opens up.

In other words metaphysics, transcendence is made most real, is possible only at the intersecting of the imminent the physical, the everyday.

Levinas opens Totality and Infinity with voices other (wise) than his own: "The true life is absent". "But we are in the world ". The first sentence is a quote taken from Rimbaud and can be read as both an admission of the lack of value or meaning and simultaneously a longing or yearning for what is missing. The second sentence is an allusion to Heidegger, who characterises human life as being-in-the-world. Being-in-the-world means that we have needs and wants that we can fulfil and enjoy, and yet there is something that does not fit into this world, it is beyond or rather to big to fit into this world it overflows it. This 'it' is the good, the infinite, the other. This third sentence reads: "Metaphysics arises and is maintained in this alibi". Levinas understands 'metaphysics' in a literal sense as meta-physics that which comes after physics, the world, what is outside the familiar and the Same. Metaphysics is a transcendence again understood literally as a moving-over or across or rather a trans-ascendance a moving up and beyond. Desire as that which cannot find fulfilment as that which shows up after all needs have been fulfilled is metaphysical or rather metaphysics.

Brian Tee

back

Les asked:

Not so much of a question but a request, Where can I find information on a philosopher who has a theory on why we become stronger after adversity or hard times?

Answer: Try the Stoics, e.g. Seneca, Marcus Aurelius.

Also, try Boethius's Consolation of Philosophy.

Nowadays, try anything written against the psychotherapy profession, especially anything written by Frank Furedi.

Richard Craven


How about Nietzsche (Twilight of the Idols Maxims and Arrows):

From the military school of life. — What does not kill me makes me stronger.

Geoffrey Klempner

back

Hasan asked:

Why don't no one understand you when clearly you tell them your point?

WHAT?

Brian Tee


lol, of course you have already answered this question. But since we're poking fun at philosophical questions, I have one for you. "if a man speaks in the woods and no woman is around to hear him... is he still wrong?"

Ryan Burton

back

Pani asked:

I want to know, if reincarnation is true, why we only remember this life? And how we can explain population growth?

Also I have another question, How can we explain the "gods" in the beginning of life? When I compare my knowledge with Von Daniken's theory, I really mix up.

You've already answered your first question by yourself, so there is nothing more to say. As for Daniken, my best advice is to try and forget what he wrote as fast as you can. Don't blame yourself for being mixed up; chances are the guy was even more mixed up than you are. As I, for example, discovered to my chagrin by checking his sources, he also had the problem that he quotes many that just don't exist. Same rule goes for reincarnation, I'm afraid. The idea is a hangover from very primitive days, when people had no science whatever and tried as best they could to explain to themselves what happens to their precious little self when the body dies. If you're interested in ancient Gods, read about them in books on mythology. There is a huge choice of them in every library in the world.

Jürgen Lawrenz


Actually you're doing very well. These are just the kinds of questions you should be asking, that is, questions about the basis and implications of assertions that people make to you. There are no satisfactory answers to these questions, because 1) there is nothing at all supporting reincarnation; there are just a lot of people who believe in it. There used to be a lot of people who believed the earth was flat, also. If wishes were fishes... For one thing, if reincarnation is true, then Christianity, Islam, and Judaism, to name only three major religions, are false. Why don't people see this? Um...

2) Von Daniken either very cynically cashed in on hordes of gullible people to sell his books, or he was simply a crackpot. There is not a shred of historical data supporting his, uh, "theories".

Keep on with your skepticism! Read about these theories! Think critically about them! Now, the next step is to do the same for the beliefs you were brought up with... painful as that might be.

Steven Ravett Brown

back

Geraldine asked:

What is at the end of the universe...and don't say that it goes on forever...if it does what is behind forever?

Why shouldn't I say it goes on forever? Why shouldn't it? What if I said it went on for X zillion light-years... then what? Look, suppose that whatever it is that "space" is, stopped after some distance. Now, how would we be able to know that, since we can only measure within and with space? Let's say that the universe is a 4-dimensional sphere, 100 zillion light-years across (a zillion is a number that I've just made up which is less than infinity, ok?). So it's finite. But we can only travel around inside it, around and around. We can't see out, we can't go out, and when we get to an edge, we just get turned around so that we're headed back inside. All we can do is go around and around. What's the "end" of that? Space has no end, then, does it. Is there an "outside"? But there's no way to have one, is there, because space stops there. Or to put it another way, the universe, if it's like that, is any size, and no size, because there's no way to measure it, since it's all the space there is. There's nothing to compare it to, so its "outside" size is anything, or nothing, or a meaningless idea.

Alternatively, let's say that the universe is a finite sized sphere, and there is space outside it. How large is it? Well... a zillion light-years. What's outside it? Space, empty space... why? Because I just said so.

Alternatively, let's say that there are multiple "universes", in different dimensions, or something like that. Ok, now where does that stop? Hey, there are a zillion of them, right? Or maybe there are an infinite number... but what's the difference between saying that and saying that this universe goes on forever... which you don't want me to say. Maybe there are a finite number of finite-sized universes; a finite number of infinite-sized universes in different dimensions; an infinite number of finite-sized universes...; an infinite number of infinite-sized universes. I'm just running through the possibilities here...

Let's put this another way. If you read Kant and similarly-minded philosophers, what they say is that "space" is a construct of the human mind, created by us to understand the flux we're immersed in. That's all. So it's something we created to understand something else... which we can only understand in terms of that concept; we don't have any other way to understand it. So... the "size", the "edge", the "behind" of the world, and so, the universe... those are all things that come out of us, to help us cope with whatever's around us. But whatever that is, doesn't have "size" by itself, or "edges", or any of those things... we just make those up to help us get a handle on it. So then your question must either relate to us, human beings, or it's just simply meaningless, because we don't know what the world "really" is. But if it relates to us, then what's at the end of the universe is what we put there... which brings us back to the "zillion" discussion above.

Steven Ravett Brown

back

Erin asked:

How would I now if my plant has died?

There is a video in the Tate Modern gallery in London showing the decay of an apple.

We don't know — I think the answer is. It is a process of decay rather than death in an apple and perhaps also a decaying process in a plant's movement from being a living thing to a dead one and so there is no moment of death. With the human I think the heart stops, but plants don't have hearts.

Just remember to water the plant!

Rachel Browne

back

Matthew asked:

I have long maintained a notion that time is a dimension in the manner of space. 'Moments' do not disappear but rather occupy different positions within this dimension. Our everyday perception/ experience of time is thus due to a limited perspective from which it is impossible to escape. I would not claim to necessarily be able to prove such a notion, but it has a strong intuitive appeal to me. Can anyone elucidate this intuition by referring to thinkers who possess similar ideas, or via its refutation?

Your idea is at least 300 years old and was (and is) still in print in the Correspondence between Clarke and Leibniz. In this exchange of letters, in which Clarke represented Newton's side, Leibniz said just this: that time is not real, but a relation between events. To understand time properly, it should therefore be geometrised, which is the same as treating it as a dimension superadded to 3D space. Of course, Einstein's theory uses pretty much the same notion, the difference being that time is not added to, but integrated with space. You might be interested in Barbour's book The End of Time, which gives you the latest wrinkles on the idea. Mind you, it is indispensable to have some elementary physics under your belt before you read this. If you do, you'll find it totally fascinating.

Jürgen Lawrenz


It's called the "block universe" theory, and comes rather directly from special relativity. Fred Hoyle, the astronomer and sci-fi writer, wrote a rather nice novel around the idea, called October the First is Too Late. You might check it out. One problem with the theory is explaining our experience of the "movement" of time, or "through" time. If the universe is a block in, say 4 or 5 dimensions, of which time is one, then why do we have the perception (technically, the "apprehension") that we move at all? In his book, Hoyle proposed a solution in which consciousness successively "illuminated" different "slots", so to speak, of mind in the time dimension. The problem with that, of course, is that in order to do that, consciousness would then have to have its own time dimension in order to move from slot to slot, to explain our experienced time dimension. But the problem was to explain movement in the first place. So that doesn't work, unless you're into infinite regressions which never actually settle on the solution.

No, I'm afraid that no one has actually come up with a way to get from the block to our experience of motion. Yes, sure, we can say that our consciousness is spread out through the time dimension, but however you cut it, you're still not getting from non-motion to motion.

But there are other problems with this notion. The "time is space" idea is a metaphor derived from ways ("image schemas", according to Lakoff and Johnson — see below) of dealing with the passage of time that we employ as children, which ultimately gives rise to a certain mathematical description of time (or to intuitions which are derived from that metaphor). Well, it's all very nice for mathematical description, up to a point, anyway (the point where special relativity turns into general relativity), but does that mean, just because it lets us do some calculations, that this metaphor actually describes reality... or is it just a convenience to let us calculate? For a nice analysis of metaphors on time, you might check out: Lakoff, G. 1990. Women, fire, and dangerous things. 2nd ed. Chicago, IL: The University of Chicago Press; and: Lakoff, G., and M. Johnson. 1999. Philosophy in the flesh: the embodied mind and its challenge to western thought. 1st ed. New York, NY: Basic Books.

There are of course many other ways to conceive of time... none very satisfactory as explanation of the basic apprehension of motion. For a very difficult, but in my opinion absolutely the best, analysis of the subjective experience of time, try: Husserl, E. 1990. On the phenomenology of the consciousness of internal time. Edited by R. Bernet. Vol. IV, Edmund Husserl: Collected Works. Dordrecht, The Netherlands: Kluwer Academic Publishers. Not easy reading, but an extraordinary work.

Steven Ravett Brown

back

Daisy asked:

We're wondering if you can help us on our debate whether 'Eating meat is right'. We are For meat and are wondering if you have any philosophical arguments that eating meat is right and eating meat is not cruel.

Well, eating meat isn't really cruel. It is the method of killing which would be held to be cruel. It can also be held to be cruel to keep poultry and animals in bad conditions and to fatten them up for eating. When animals are not farmed they can become extinct, as in the case of fish. This can be seen as an act of cruelty towards creatures who naturally feed on fish.

You might look at Roger Scruton's book Animal Rights and Wrongs. In this book, Scruton says (and I must quote this as it is so funny):

"I find myself driven by my love of animals to favour eating them. Most of the animals which graze in our fields are there because we eat them. Sheep and beef cattle are, in the conditions which prevail in English pastures, well-fed, comfortable and protected, cared for when disease afflicts them, and, after a quiet life among their natural companions, despatched in ways which human beings, if they are rational, must surely envy. There is nothing immoral in this. On the contrary, it is one of the most vivid triumphs of comfort over suffering in the entire animal world. It seems to me, therefore, that it is not just permissible, but positively right, to eat these animals whose comforts depend up on our doing so."

This picture of English pastures and comfort doesn't complete the story. Do animals enjoy being unnaturally herded into a lorry and carted off to the abattoir where we call their manner of death "slaughter"? If Scruton loves animals that much you would think he could allow that we could keep them for aesthetic reasons as a means of enhancing the countryside.

You should look at Richard Sorabji's paper "Thou Shalt Not Kill — Not Even Animals" at http://www.gresham.ac.uk/topright/thoushalt.htm. After summarising different points of view, Sorabji concludes that there are many moral considerations but no determinate answer.

I would argue that if animals are truly kept in conditions suitable for their natures and killed in a truly humane way then there is no cruelty. Whether this is "right" when, we have alternatives to meat, will depend on how necessary meat is to our health. If you are going to win your debate you will need to armed with research on whether vegetarians are more sickly, have shorter life-spans and fail to reproduce as much as meat-eaters. This must be set against an argument that it is our moral duty to flourish and survive.

Rachel Browne


There is nothing wrong philosophically with eating meat. The only criterion is whether your digestion can cope with it. So it ends up being a matter of personal alimentary hygiene. As for cannibalism, there is of course an ethical argument against it. But in saying this, I have to be straight with you and repeat my first sentence. People have been known to practise cannibalism. I don't know how they justified it, but certainly not philosophically.

There is another side to this issue. Life eating life is the rule of life. Every living creature on earth earns its keep by eating some other creature. Vegetarians often eat fish and assert that this is somehow (ethically) different from eating beef. I don't get it. For that matter, if you happen to ever have the misfortune of being stranded in a wild place where lions or polar bears roam, you'll be part of the food chain, so beware. In short, the chemistry of life is infinitely recycled. Without this sort of activity you and I would not be alive. You must surely be aware that our guts are compost heaps, playing host to billions of bacteria. It is easy to forget, especially when you delve into philosophy, that humans are animals to start with. Eating is a creature function. And eating meat is just recycling biological matter. So except for the ethical dimension noted above, eating meat is a personal choice.

Jürgen Lawrenz

back

David asked:

Would it be fair to say that Karl Popper invented the concept of Falsification?

Or might it be fairer to say that it didn't exist as an applied methodology before Popper?

Or are both of these propositions doubtful?

As far as I am aware, Falsifiability is generally held to have overcome the problem of induction, and acts as the criterion of demarcation between science and non-science, despite its critics.

Popper invented it. But in the rest of your question you're barking up the wrong tree. Falsification has to do with proving scientific hypotheses. In his youth, members of the Vienna Circle had propagandised the notion that philosophy should accept nothing as true that cannot be positively proved to be true. When Popper examined this issue he realised that this is impossible: science can prove practically nothing to be true, it can only prove that something is false. So it is not a methodology as such, but an idea or principle, to be applied to every scientific experiment: the experiment, if it successfully proves some hypothesis or theory false, can be said to have achieved a truth. Finally, the connection to induction is that truth is impossible of proof precisely because 1000 or a million experiments can never be accepted as truth as long as the possibility exists of just one contradictory result. Induction therefore is only what Bacon originally wanted it to be: a collection of facts usable for a working hypothesis. It confers no guarantees.

Jürgen Lawrenz

back

Dimienne asked:

What theological questions were raised by the experience of the Holocaust and what attempts have been made to resolve them?

The problem of the existence of evil and how to reconcile it with God's (supposed) qualities as good and all loving, omnipotent and intervening has been around for a long time and very many answers trying to harmoniously incorporate them into justification have been proposed. For example: that evil is the result of human beings use of there (God given) freewill and that for God to change that would be to change the special status of humans. That Evil is not really evil, but part of some divine master plan to test the faithful, that evil is a punishment for our sins. None of these are very convincing from a philosophical point of view and many have concluded that evil is a proof against the existence of God. After the Holocaust these issues were raised again with a new urgency and with a justifiable indignation. Many writers and witnesses to the Holocaust joining Nietzsche in affirming the Death of God. And that after the Holocaust all human endeavour, activity and production must be carried out in its shadow such that "No statement, theological or otherwise should be made that would not be credible in the presence of burning children" (Greenberg).

But what questions does the Holocaust forces us to consider? Can we any new questions, questions unique to the Holocaust, or unique in that they would not have been asked if the Holocaust had not happened, but questions that impress on us now and demand to be reckoned with? One question is has the relation between God and humanity as a whole undergone revolution? Can we anymore bow down and worship God, can we think of God as a father and we as children after the Holocaust or have the children grow up such that they are in a position to question the Father?

Of course there are questions Jews (specifically and believers generally) have to face; Has the nature of the covenant changed, has God simply abandoned his chosen people? Leading on from this we can ask if the nature of religious vocation has changed, Is the impetus on God to agree to our conditions now, or is a voluntary renewal of the covenant a more ethical pact a more trusting endeavour in the face of the Holocaust? I don't think I am equipped to answer such questions, but there is a kind of question more suited to a philosopher: it is whether we have really ever had a conception of God as God? Or was the God that died in the death camps only merely an illusion of God created by humans?

What I mean is that talk of God as supremely powerful, as guardian, as all loving is an anthropomorphism a God made in the image of man. The perfect man, a fully extended version of man's self-image, This was the God that died and deservedly so along with the death of the image of man himself. If it is correct to say that the Holocaust forces us change every aspect of human existence, so that it will not happen again, this means humanity must rethink what it is to be human, not as free, independent, self-contained and whole unto itself, but as dependent, fragile, responsible. It also means that maybe the God who died in Auschwitz was not the real God, but a God that humanity projected: God as the perfect being. But as I have said previously in these pages (to save repeating myself try a search for the other answers), maybe Being and God don't go together. Rather than being that entity that has Being in abundance the most real being, maybe God is the most delicate entity, exiting not as a thing, not as the biggest and best but as the fleeting and contingent relation between humans who care for one another. What this implies is that God of philosophy and theology and pre-Holocaust humanity is a fiction and a useless fiction after the Holocaust. It is not God's existence that we need to be convinced of since God is outside the realms of being and not-being. And therefore was in no position to physically stop all the children burning. The question becomes inappropriate, it is the wrong question to ask, even if it's hard to realise that, because then the psychological safety net disappears.

What we need to be concerned with is the little goodness that is made possible by recognising that other people as special and needing our help, maybe that's where God lives, such that even in the death camps God was still alive?

A good collection to view is: Holocaust religious and philosophical implications. Edited J.K.Roth & M.Berenbaum

Brian Tee

back

Li asked:

If a person is sliced symmetrically in half instantaneously about the vertical axis, before that person's inevitable death, where would his consciousness lie? Since the brain is also symmetrically divided perfectly, would it be possible to have two entities of that person in that moment? A "logical" and a "creative" version of that person, so to speak?

Your question presupposes that the conscious "I" is somehow independent of the brain in such a manner that it could function with only half of one. I challenge this assumption as a position for which we have no evidence. So my answer would be that the consciousness would die in the instant that the brain was divided.

There is a significant amount of evidence, to which you make passing reference, that the two halves of the cerebrum contribute different sorts of mental functions to consciousness. But there is no evidence that consciousness could successfully function without either set of those functions.

There is evidence from victims of severe schizophrenia, that the corpus callosum can be severed without impairing consciousness. The corpus callosum is the structure deep in the brain that connects the right and left hemispheres of the cerebrum. But that is not the only pathway that connects the two halves of the brain. There is more to the brain than the cerebrum. Patients who have undergone such treatment have never displayed dual personalities. Rather they have demonstrated that there must be alternate pathways for information to flow from one side of the cerebrum to the other.

There is also evidence from people with various sorts of brain damage, that consciousness can get along reasonably well without major portions of the brain. But none of that evidence would suggest that one half of a functioning consciousness could suddenly get along without the other. Survivors of major brain-cell losses take years to even partially recover. All evidence of this sort would suggest that loosing any piece of the brain causes severe mental problems for the unfortunate victim. Loosing one-half of the brain-cells would undoubtedly be instantly fatal.

For more discussion of this issue, in greater depth and with greater expertise than I can muster here, I refer you to Consciousness Explained by Daniel C. Dennett.

Stuart Burns


Well actually your experiment has been done. There are people with very severe (i.e., virtually continuous) epilepsy who, in order to stop the spread of the neural discharges over their brain, have surgery in which the two halves of their brain are separated, by cutting the connection called the corpus callosum. This results in "split-brain" people, who do in fact seem to have two consciousnesses in one body. There is a huge literature on split-brain patients... just go look it up. The "logic" and "creative" dichotomy is not really the issue; the major dichotomy is between verbal and non-verbal thinking and expression, since the verbal areas are on the left side of the brain. This makes communicating with the non-verbal personality difficult, but possible.

Steven Ravett Brown


One problem: the brain is not functionally symmetrical. But in any case, your version of a logical and creative brain is so old and obsolete now that I urgently recommend you update your reading. Try, for example, Eccles' Evolution of the Brain, Creation of the Self.

Jürgen Lawrenz

back

Justin asked:

What is the flaw in Robert Pirsig's metaphysics of quality?

Is he just another idealist? maybe the linguistic and cognitive vagueness of his writing fooled me, but I thought he was convincing enough to at least justify his belief.

Having read both Zen and the Art of Motorcycle Maintenance and Lila I share your liking for Pirsig. And it's absolutely the right thing to do, once you've read something persuasive, to turn about for competing views that help one mull things over and come to a view that's really one's own. And I've a couple of suggestions there, if you follow me to the bottom of this. All the same, I wonder if the way you put that here is a bit hard on Pirsig, maybe even on Philosophers in general.

First off, why speak as if you were looking for some damming 'flaw'? Of course we all talk in terms of flaws in arguments and so on, but the application here to a philosopher's belief system as a whole is a metaphorical one that is pushing a certain picture of how philosophy is, or ought to be — and that brings with it problems. Literally, a flaw is a line of weakness in a material along which it will break when put under stress. That's fine for metals in aeroplane wings where the job that the thing is supposed to do is agreed upon: controlled flight. But in Philosophy it's often precisely this sort of thing that is up for debate: i.e. it is the task of philosophy, and so also what would constitute its failure, that is up for debate. What is philosophical reasoning, and what is it for? These are themselves two important topics of philosophical thought. So, there will very often be room for disagreement about whether something is a 'flaw', if only because there is unclarity about what job an 'unflawed' piece of philosophy must do, and about what kinds of stress it may legitimately be required to withstand without fracture. So the imagery of 'flaw' is a bit stretched here. For that reason, I suggest that it may be better to speak of philosophical failures not as 'flaws' but, more precisely, as being variously: factual inaccuracies, contradictions, unclarities, loose ends, imaginative failures and so on, whichever may happen to apply.

It's true that professional academics have been sniffy about Pirsig's work, to varying degrees. One of the philosophical failures likely to be identified in Pirsig is that he hasn't properly read all the great books or the current journals, and so isn't writing from a position of knowledge about the traditions of the past or the controversies of the day. In defence of Pirsig on this count, I'd say first that while studying philosophers is extremely helpful for tackling philosophical questions, I'm not convinced by the position taken by some that it must be actually impossible to do original and interesting philosophy out with a directed study of the tradition — just difficult and unlikely. Second, I'd say that while Pirsig's retelling of the history of Philosophy is polemical and has huge gaps, he does show evidence of having read understood and inwardly digested in a rather interesting way at least two important figures, namely, Plato and William James. Whether you think Pirsig's unexpected and innovative attempt to integrate Platonism and American Pragmatism makes up for inadequacies in philosophical understanding elsewhere may depend upon your own philosophical judgements and interests. In my case, I happen to like both James and Plato, and this has something to do with my appreciation of Pirsig when I discovered him.

The three kinds of philosophical failure which can perhaps justly be laid at Pirsig's door are: ambiguity, leaving loose ends, and failing to tackle some questions of interest to philosophers (pick your question of interest). You might want to call some of this "linguistic and cognitive vagueness", except that such things are both relative (Pirsig is a lot less linguistically and cognitively vague than most of what passes for news) and widespread (most philosophers accuse other philosophers of vagueness). There are certainly a good quantity of loose ends and ambiguities in Pirsig's work to be getting on with, and one sign of this is the continuous round of disagreement between folk who think of themselves as Pirsigians. For which see http://www.moq.org. There you will also find some competition between views, all the better to form your own. Two good plans to run in parallel are: (1) visit the fractious http://www.moq.org, and (2) read lots by and about both Plato and James and all the philosophers in between. For the latter you may usefully call on the help of a Philosophy Department, who can help you to identify important books and questions, give you a sounding board and an audience for your developing thoughts, and generally introduce you to all kinds of enlightenment and fun.

David Robjant

back

Cliff asked:

1) Theoretically, if something could travel faster than the speed of light, is time travel possible? If you could go faster than the speed of light time would slow down, right?

2) Are heaven and hell a dimension? when you die, you leave the first 3 (maybe 4: considering time is only a conscious thing) dimensions behind.

I'm a 21 year old musician and now this website has inspired me to look into philosophy as a higher learning tool. Thanks.

First: time travel is not possible. Fundamentally, the idea offends against the principle of simultaneity. An object cannot be in two places at once. Although this is commonly swept under the carpet in science fiction, well that's fiction. Scientists often speculate on time travel and devise all sorts of wonderful ideas to circumvent the logical conflict, but (much as I hate to be a spoil sport) the facts — all the facts we know are dead against it. So, fiction again.

For what it's worth, time travel also contradicts another basic law. The speed of light in a vacuum is the grain of time. Thus light is essentially energy, void of information. To travel at that speed means you would have to convert yourself into pure energy. But then, you erase the information that enables your reconstruction some place else. So much for stepping under a canopy etc etc. In any case, these fictions never keep you informed that even at the speed of light, time travel would still take thousands of years from one galaxy to another.

Faster than light particles are known, they're called tachyons. And yes, as you guessed, they enable us in certain specialised areas of research, to "slow down time" and study complexity related phenomena at a more humanly graspable speed than "real time". If you're interested, there is a chapter on this in Ian Stewart's book Does God Play Dice?

What heaven and hell are you better try to find out from a theologian. Philosophers are at times inclined to assert that the planet we live on is hell; but as for heaven, although there are hundreds of rivalling accounts from all the different religions, no-one ever received a doubt-free message from the place, so we can't know what it "is". (Theologians will, of course, refer to revelation, but some recalcitrant philosophers will doubt the reliability of this report as well). Now when you die, you don't leave any dimensions behind at all. Strictly speaking, dimensions are a facon de parler; they are not real in the sense that your keyboard is real. But in any case, the crucial thing is not that you "leave", but that you cease to be. Out like a candle, as Shakespeare said. Now many religious people hold fast to the idea that "you" don't really die, only your body disintegrates. But I think there is an awful confusion behind this notion. From my experience this belief invariably entails the survival of the personality, but this cannot be, because the kind of personality you are is completely fashioned in life and carried across time in biological memories which die along with your body.

I think, truly, that one of the great issues in philosophy is just this: death. We've tried for thousands of years to get to grips with it, but because we are creatures, there is inbuilt resistance to our calm acceptance of the ineluctable and utter finality of individual non-being. Maybe you should have a look at my Pathways paper, entitled Death, Free Will, Value. I'd like to think there's something in it for a person who is not frightened to contemplate the meaning of death.

Jürgen Lawrenz


1) Time slows down, sort of, when you move at any speed at all, during your acceleration and deceleration, not when your speed is constant. In the latter case, you can't tell whether it's you or the rest of the world which is moving. Therefore, time also slows down in gravitational fields, since gravity is equivalent to acceleration. You just don't see the slowdown when you're driving in your car, because it's utterly insignificant (but present) at those speeds. As to whether "time" really slows down... no. You slow down, i.e., the motion of the particles making you up... because they become more massive as you accelerate. Look, all this is very nice to sort of push words around with, but everything I'm saying is just pretty vague approximations to the real physics. You need to do some reading on this to really understand it. Take a look at something by Asimov, or the early popularizations by Gamow.

2) No.

I'm glad... but please learn some more before you take a running jump into sheer speculation.

Steven Ravett Brown

back

Christopher asked:

This is intended to be somewhat philosophical, since I haven't found the answer anywhere else: Why are blue jeans blue? I'm not asking about the techniques for dyeing them blue. My question is rather why do we want them blue? why (at least until very recently) do we want our jeans to be blue, in spite of available technology to make them as good in any colour?

This is not a philosophical question, sorry. My memory on details about jeans is vague now, but I'm not going research this matter on your behalf. For as it happens blue jeans became just a fashion craze a couple of decades ago, and they were originally the product of a maker whose factory was in some French town that lent its name to the product [Serge de Nimes]. So your quest for a "deep" meaning is in the same league as the "left wing politics" (why left wing? Because their originals sat on the left side of the house of parliament); or "bernaise sauce" (from Bern) or "saxophone" (from Adolphe Sax, its inventor) and innumerable other instances.

Jürgen Lawrenz


You might have a look at this site: http://www.designboom.com/eng/education/denim2.html.

Jeans were first worn by 18th century slaves and then by 19th century gold miners because the material is tough, but the site says that blue was a good colour when frequent washing wasn't possible.

In the 1950s blue jeans became a "symbol of teenage rebellion".

It might follow from this that older people want to wear blue jeans to express something about their youthful personalities and non-conformity. Perhaps the general statement made by wearing blue jeans, by the young and old, is that clothes washing isn't very important to them although this doesn't seem likely since jeans are the easiest things to wash and normally don't have to be ironed.

But people probably want to wear blue jeans because they are uniform and classless and easy and seem to go with everything. Blue jeans are worn by fashion models and builders, by the rich and poor.

Not everyone wants all their pairs of jeans to be blue. I have a preference for black jeans in very cold weather and white jeans in the summer. The problem with white jeans is that you look too white if you wear a white t-shirt. They are limited in a way that blue and black are not. But black jeans are perhaps not as uniform and classless as blue jeans because they are not as commonly available.

This can be considered to be philosophical question and perhaps it can be explained by Pierre Bourdieu (Distinction: A Social Critique of Judgement) who posits cultural communities — we distinguish ourselves from others (the suited) and come to belong to a group of people who don't want to be seen to be making too much effort by wearing easily available and easy to wash blue jeans.

It is also of psychological interest. As a teenager, perhaps wishing to stand out, I had a pair of black and pink stripy jeans which was probably an expression of a wish not to be considered ordinary or the same as everyone else. Though oddly, this was at a traumatic period in family life and my mother actually also had a pair of these jeans, but they were yellow and black. These jeans were quite fashionable at the time, but not common. But the point is that not conforming in dress can express emotional trauma. From experience I have noticed that when people are particularly traumatised they do not conform to an expected dress code. I knew someone who was going through such a difficult period emotionally that he could not buy a ready-made shirt and had to create his own shirt and send it to a tailor to copy.

Rachel Browne

back

Zak asked:

Someone give me a convincing argument against the reasoning that humans have "free will". I've heard the argument that if you could calculate everything at a certain moment you could predict the future. But that doesn't mean we don't have free will, it just means we can predict the future. I think that a lot of philosophers take the "everything has been determined" standpoint because they are pretentious.

I don't know that I can supply what you are after regarding Freewill. But take care about formulations of Determinism.

You pick upon that popular picture of the clockwork universe in which the pre-arranged future is pictured as being rather like the modern weather forecast only better. Observations about present conditions are made and then computations according to known patterns and laws predict the predetermined future states. Now, the weather forecast sometimes gets things wrong. But you think of the predetermined future as that plan of events which a thoroughly perfect scientific understanding of everything that holds in the present would have predicted: "if you could calculate everything at a certain moment you could predict the future".

Problematically, your choice of words here makes the truth of determinism dependant upon the conceivability (not practicality or actuality, in the way of which there are several obvious obstacles, but conceivability) of 'a perfect scientific understanding of the present'. And I think that there are several reasons why this perfect understanding isn't conceivable. The first and most serious is that the concept of a perfect measurement can't be made sense of. Even supposing that you could get magically instantaneous and non-intervening methods for measuring the position and spin and so on of every single atom in the universe, there would still be the problem that the measurements from this survey would have to be returned in numerical and computable form. If they are to be computed then the number of decimal points in the observations, and thus the degree of accuracy, could not be infinite. So, any set of data from which calculations could be performed would of necessity contain imperfection and error. This necessary imperfection in the information could only lead to a necessary imperfection in the prediction. Thus you cannot (and not even conceivably) "calculate everything at a certain moment [and] predict the [determined] future".

Now, this is just to say that you cannot (as it is usually supposed) understand determinism in terms of an ideal science, with the determined future as that which a perfect science would correctly predict. Maybe it is possible to understand determinism in a different way, but since the attraction of determinism seems to derive mostly from faith in science and technology (the "clockwork universe"), nobody much seems to be trying.

David Robjant


Did you choose to be born? Have you a choice between mortality and immortality? Can you wilfully increase or decrease your height? etc. etc. etc.

It seems that any free will we are able to exercise falls within set parameters. To illustrate with a simple analogy, Consider yourself locked in a windowless room with a few pieces of furniture, your choices will be limited to moving the furniture around, the walls and the locked door will ensure that the number of choices available are determined within these constraining parameters. If your captors control the light switch outside your room, your light and dark (night and day) will also be outside your choice. As I say, a very simple analogy, but the choices in life are rather like this, a limited freedom within a determined scenario.

We can make good choices in life, or we can make choices we afterwards regret, but, over all, we find some powerful restricting boundaries in nature and very much under the influence of cause and effect. Most people live their lives burdened with unfulfilled desires.

I find it difficult to respond to your accusation that philosophers who support determinism are "pretentious". I require to see the premises of your argument, the claim you make is an unsupported conclusion.

John Brandon


Try Elbow Room: The Varieties of Free Will Worth Wanting by Daniel C. Dennett. I think you will find the author very readable, and his text directly addresses your question.

Stuart Burns

back

Ryan asked:

Eliminating the soul. If two people (twins clones whatever) share the exact same experiences would they be the same person?

Any "no" or contrasted answers to this question would be greatly appreciated.

No, they would not. For apart from being souls, they are also objects, and as such they cannot pursue the identical trajectory in spacetime. This is not contradicted by those rare instances of twin bodies being joined to each other, like the twins who recently made the headlines. At the microscopic level, for all that they may have shared the same genes, implementation (or translation) will produce (large-scale) errors on an individual cell by cell basis, and by that fact alone, even clones would differ from each other at least in the way they suffer the same experiences. It is possible, perhaps, that subjectively the two may believe in the identity of their shared experiences. But I doubt it: for at the very least the case I alluded to shows that their experience of right and left must be incompatible.

Jürgen Lawrenz

back

Joe asked:

First of all, I have been asking the exact same questions. The universe without matter to fill it, is still something. Space itself exists, and can exist without matter to fill it. Therefore one can not place a time limit on area itself only on the matter that exist within it. I have tried to get answers back from websites for many years now. Scientists will not touch that question. I do not understand how a scientist with doctor's degree can try to tell me the universe is approximately a few billions years old, when the expansion or reverse could have happen over and over. I believe that there really is no such thing as nothing. The conscience goes back into the void. Genetic traits are apart of the machine that is matter. There is a creator that places our conscience in where ever is necessary.


Joe also asked:

I have noticed something in my life time. Most everything is either a circle or cycle. All of matter seems to be revolving in a circle, and that means all of the matter in all of space because it is so vast one does not know. Seasons cycle, galaxies cycle. Why do scientist stick to any kind of big bang theory when the very same ones claim that stars are born in the nebula Orion from dust clouds that they can not explain? Their speculation have no foundations. The string gravity theory intrigues me though.


and Sam asked:

I am trying to figure out the origin of the universe. Why does there have to be a beginning? Is it possible that the universe has been in existence for eternity?

I'm sorry, but your initial assertion is already mistaken. Space without anything in it is a logical absurdity. It is true that Newton believed it, but only on a "let's pretend" basis so that his equation might come out right. His critic Leibniz showed why this is absurd, and also gave an excellent account of time, which you might find yourself agreeing with.

I'm puzzled at your claim that "scientists will not touch" your question. I've been in correspondence with several of the leading lights in physics and found no resistance whatever to the discussion of this issue. Maybe you should read An Introduction to the Philosophy of Time and Space by Van Fraassen. Newton and Leibniz figure large in the book, as well as more recent theories on the same subject.

The last three sentences of your question elude me. Maybe it's in the spelling, but I can't make out what you're saying. Try again, perhaps?

Joe's second question:

You may be surprised to learn that phenomena moving in circles and cycles is just about the oldest observation in the oldest records of mankind. Now though perhaps you should be glad to have discovered it again on your own bat, it would have been more economical, surely, to have read some appropriate literature first. Don't you think?

When you speak of scientists and big bangs, you really should be more precise about which scientists you have in mind. For it is a fact that many of them do not stick to the big bang. The latter is just one of a fairly large number of current creation scenarios, and it is partly a matter of accident that the big bang theory acquired popularity. I'll stick my neck out and suggest that the reason is the most stupid one anyone could think of: the name. Its punchy and easy to remember, and doesn't everyone love the idea of a huge cosmic explosion? Undoubtedly. However, you're mistaken in asserting they are speculations without foundations. They have the best foundations of any cosmological theory in human history. But it seems to me that you're not acquainted with them. So by all means carry on with string theory. Amid all this, it occurs to me that we haven't mentioned philosophy with one word. So what was your question really all about?

Jürgen Lawrenz


If the universe were to pass out of existence, you could ask, "What were its dimensions, its length, width, and height, when it existed," but asking "don't those dimensions still exist in the form of space, even though they're empty" may be like asking, "What was the angle between the roadway and the supporting pier on that bridge they demolished last week, and does that angle still exist, even though the bridge is gone?"

Douglas Barber


Personally, I cannot envisage a universe without matter, in the empirical sense that matter is generally understood. I believe that I can understand the point that you are making, that space and time are absolute realities. However, does space — time in isolation provide a sufficient concept for the constitution of a universe? What would it relate to? How could it be understood to eliminate a vacuum? Perhaps a physicist could explain a four dimension space — time in a vacuum. I think they would be hard put to it, I find it beyond my capabilities.

The structure of your question presents you with immediate problems. What is meant by "a universe without matter to fill it"? I understand a universe, whichever aspect of philosophy we support, to be something we can observe, 'matter' distributed within a space — time concept. (This is a general view, but as a supporter of Kant I see it in a rather special way.) The naive view that the universe fits into some sort of container, like a gas in a spherical flask, seems to be at the seat of your question. What is the flask made of? What are its dimensions?

I feel that I have to agree with you that space—time only has value in relation to the displacement and measurement of 'material' objects. As for the notion expounded by scientists to describe the origin of the universe, these are theories based on current observations. Unfortunately, what we often find in science is a search for evidence to support a pre-conceived notion, over time the notion becomes established as the likely explanation and, despite discoveries giving possible/ probable support to alternative theories, these alternatives are largely ignored. Evolution is another case in point.

You believe that there is no such thing as nothing, yet you say that the conscience goes back into the void. Is this not contradictory? I agree that life means rather more than genetic manifestation. I also concede that you are entitled to believe in a creator until it is proved beyond a shadow of doubt that no such entity exists, and this is likely to be a never ending quest. However, you will require a very persuasive argument to overcome the secular ideas of the present age.

As you say the 'cycle' is very prevalent in nature. However, we cannot make assertions about happenings in the universe beyond the range of our observations. With regard to the birth of stars, these are events occurring following the alleged Big Bang, and in no way affect the notion of the Big Bang. The Big Bang is alleged to have set everything in motion, e g the expansion of the universe and the constant re-distribution of matter.

I agree that the String Theory is very interesting and stretches the imagination to extremes. For good coverage of the theory I recommend you read Brian Greene's The Elegant Universe (Jonathan Cape, London. ISBN 0-224-05299-3).

John Brandon

back

Ted asked:

If you had the choice live forever in a healthy state would you take it. I definitely would because there is infinite knowledge and the human race is so exciting to be a part of, especially with the prospect of interstellar travel and the colonization of other planets. I don't see how anyone wouldn't want to be apart of that...one would have to be a fool...

If you like life so much why not be content with living on this planet?

Looking at the past, on this planet, has colonization really been a good thing?

Rachel Browne

back

Jeff asked:

I'm reading Aristotle's Topics. In it, he says that all propositions or problems consist of an assertion of either a definition, a property, a genus or an accident. Assuming he is right, wouldn't it be useful for children and adults to grow up learning that what we say can be divided into these four categories? Wouldn't it clear up our thinking, talking, debating, arguing, etc., to have this as an understood basis on which to begin? I don't get why Aristotle's teachings are not on the lips of every talker in our society.

The short answer is that not every philosopher agrees with Aristotle. So why teach something to your kids that you don't agree with? The slightly longer answer is that sure, Aristotle came up with four categories which seem reasonable, but you yourself, and others, can and have come up with many other ways to categorize propositions. Which are correct? Well, probably most of them... depending on what you want to use them for. So then the question becomes, not necessarily how to categorize propositions, problems, sensations, thoughts, emotions, objects.... and on and on. The question splits into things like: what is it to categorize? what does that mean? how does one go about it in a way that's useful to some specific endeavor? should we attempt to find "universal" categories? why even think in such terms?

And what kind of thinking, generally, is a good way to go about doing the above kind of thinking? That is, how do we teach our kids, not so much to use someone else's categories, but to think of their own, in constructive ways? I, for one, would be much more interested in teaching the latter rather than teaching a fixed set of forms to apply to situations that Aristotle could not have dreamed of.

Steven Ravett Brown

back

Doug asked:

I propose that "The ultimate value is to preserve and enrich the capacity to value." This allows a criticism of values which are inimical to human life (since life is a condition necessary to the capacity to value), and also demands that I respect the capacity of others to pursue all those values they may articulate for themselves which do not contradict this single ultimate value.

I expect this isn't a new idea (though I haven't encountered it in my reading), and would like to know where I might find it expressed, and what arguments have been offered against it.

I'll be so egoistical as to suggest you read my Pathways paper Death, Free Will, Value.

Jürgen Lawrenz

back

Richard asked:

If one has a ship, and one replaces a single part of the ship every year, until all the original parts have been replaced, is it the same ship?

If so, if you rebuilt the original ship from the removed parts, would the two ships be the same?

If not, at what point did the ship cease being the original ship?

It depends what you mean by "the same". If you mean "the same object", then obviously the answer is no in the first case and yes in the second. If you mean, can there be two identical ships, then the answer is obviously yes. To get any mileage from your questions, you should pre-empt such ambiguities before you ask. I suspect, in fact, that you're not asking about ships, but about something else. So beware that your question, whatever its real subject, does relate to objects and not (say) to designs. Objects can be identical, but numerically distinct. But (for example) if you look at 50 identical squares on a sheet of paper it can be argued that you're actually looking at just one square, inasmuch as you're looking at 50 representations of a single concept, not at an object. And if you extend this idea to embrace biological specimens, the answer is "no" throughout. Thus you could not replace a spider's limbs with prostheses and proceed as in the ship example. A small matter called death would intervene somewhere along that line.

Jürgen Lawrenz


Define "same". If "same" means: made of the identical atoms, etc., what is the answer? If "same" means: looks and functions identically, what is the answer? What definition of "same" do you want? Pick one, or make one up, and answer the question yourself.

Steven Ravett Brown

back

Jared asked:

I am trying to drastically change my life and gain a better understanding of life in general. What philosophical books have changed your life? I would prefer to read books that I can readily apply to my social life.

Here is my own offering of the three most life changing books I have read — (1) Atlas Shrugged by Ayn Rand; (2) The Selfish Gene by Richard Dawkins (and I recommend you follow this with The Extended Phenotype by the same author, in order to complete the story started in the first book); and (3) Men are from Mars and Women are from Venus by Dr. John Grey. All are very readable, and targeted at the average reader. They are not heavy philosophical works.

Once you have managed those, try these heavier philosophical works — Darwin's Dangerous Idea and Consciousness Explained by Daniel C. Dennett; The Wealth and Poverty of Nations by David S. Landes; Culture Matters by Lawrence E. Harrison and Samuel P. Huntington.

Add to this list a good History of Western Philosophy. My own favorite is A History of Western Philosophy by Bertrand Russell, but it is a little dated (first published in 1945), stops with William James and John Dewey and the early Logical Positivists, and should be supplemented with a text that covers the Twentieth Century. With this as a base, I think you will be well positioned to start your own reading list.

Stuart Burns

back

Leautaud asked:

I have something in my mind that's troubling me. Somehow I have managed to see that generalizations are not correct, since we know nothing about the universe and there could be something out there that would make that generalization incorrect. I don't know, but I strongly believe that generalizations are a major problem in people's way of thinking. What's your opinion about this?

What you have identified is more commonly known (in philosophical circles at least) as "The Problem of Induction". An inductive conclusion is a generalization from a sequence of observed particulars to a general conclusion. The "Problem of Induction" is that the truth of such conclusions can only be demonstrated Inductively. This can be considered either as arguing in circles, or "bootstrapping" — depending on how sceptical you are feeling.

For example — I see a swan and it is white. I wander about my environment and observe that all the swans that I see are white. After compiling what I feel is adequate evidence to justify the conclusion I generalize to the inductive conclusion that "All swans are white". This, of course, is incorrect. My conclusion will be invalidated the first time I encounter a black Australian swan.

When one employs induction to reach such generalizations, one must keep in mind that the conclusion reached is only probably true, and not absolutely true. The degree of confidence one can have in the conclusion depends on the nature of the evidence supporting the conclusion. The more the evidence, the more detailed the understanding of the causations involved, the more confidence one can have in the truth of the conclusions. But inductive conclusions are thereby unlike "religious" truths that are taken to be absolutely, totally, and unchallengeably true.

On the other hand, all statements about the nature of Reality are Inductive conclusions. Far from being a major problem in people's way of thinking, Inductive Reasoning is an essential part of how we deal with the slings and arrows of daily life. Do you set the alarm clock to wake you on time tomorrow morning? Do you turn the steering wheel left (rather than right) when you want to make a left turn? Do you eagerly anticipate that mouth-wateringly delicious fork-full of (whatever) at dinner time? If you do any of these things, then you are basing your actions (and your emotions) on the inductive generalization that the future will be like the past, and that things will continue to work the way that you have experienced them working in the past. That the sun will rise tomorrow (and the Universe will not disappear in a puff of sub-quantum smoke) is an inductive generalization.

But I think I might know why you are arriving at your belief that such generalizations are a major problem in people's thinking. Most people are brought up within an Authoritative social environment. Parents, schools and most especially religious organizations are constantly bombarding the poor student with messages such as — "Do what I tell you because I tell you", "Believe what I tell you because I tell you", and most destructively "These are the Truths, and they are absolute!". As a result, too many people begin to think that any statement that is accepted as "true" is "Truth Absolute". Too many people forget the conditional on inductive generalizations — they are only probably true, are always subject to revision and correction, and they are only as useful as the evidence that justifies them.

Stuart Burns

back

Danny asked:

Let's pretend that I am an ant who lives in a colony in the middle of this really big plank of wood. Then I started to walk to one end of the plank and could not reach the end even until my death. My children and their children's children and a hundred generations after tried the same thing and still did not reach the end. Would I, as an ant, be correct in stating that this plank stretches to eternity?

In a word, no. First, you don't define the term "eternity", so really, your question can't even be answered. Second, if "eternity" means "an infinite length", so that the question becomes, "can we know that something has infinite length by empirical measurements of finite increments?", the answer is still "NO". Here's the classic metaphor: suppose you have a hotel with an infinite number of rooms. Ok? All the rooms are full. A guest comes to the desk and wants a room. The clerk says, "fine", and moves guest number 1 to room number 2, the guest in number 2 to number 3, and so forth... the guest now stays in room number 1, and everyone else still has a room. That's an illustration of what "infinity" means. If you really want to know, read G. Cantor on orders of infinity.

Steven Ravett Brown

back

Ethan asked:

If Zeno's paradox attempts to show existence is static and that motion and change are impossible, why hasn't anyone postulated the opposite — i.e. that nothing is at rest and change is happening all the time? This seems to me the obvious way of solving the problem. And sorry I think all the mathematical solutions just don't cut it (no pun intended).

Two points. The first about your opening 'If', and the second about your central 'why?'

(1) To begin with, it is not clear that "Zeno's paradox attempts to show existence is static and that motion and change are impossible".

The alternative possibility which we should consider is that Zeno is attempting to give what is called a "reductio" of a view he rejects, by showing that motion would be impossible if one assumed what his opponent assumes about space and time. A "reductio ad absurdum" where one shows that a set of claims is contradictory. The moral of a "reductio" here might be that some people have an incoherent picture about motion, rather than that there is no motion on any picture of motion whatsoever.

There is room for controversy about exactly what the objectionable set of assumptions is for Zeno. All of our evidence about Zeno is second or third hand, the principal sources being critics of Zeno, namely Plato and Aristotle. But there are grounds, in this evidence, for thinking that Zeno is presenting a reductio ('there is no motion if we assume what you assume') rather than a positive thesis ('there is no motion').

Without going through the whole thing, I'll point to two suggestive texts. See (i) the passage in Aristotle's Physics where he reports that absurd conclusion that the arrow is stationary in flight "follows if it is assumed that time is composed of nows". Is this Zeno's assumption, or an assumption made by someone that Zeno is attacking? Aristotle isn't clear about this. But what Plato says about Zeno's motivations may be relevant. See (ii) the opening section of Plato's dialogue Parmenides, where Zeno is presented as advancing no positive thesis about anything at all. In that dialogue, Zeno protests that he has been misunderstood as advancing some thesis of his own, whereas what he meant to do was to show that what the pluralists believe leads to just as many absurdities as the position of Zeno's teacher, the monist Parmenides. If one considers that Parmenides' monism amounted to the thesis that there is just one thing, and is tempted by the definition of movement as the change of one things position relative to another thing, one can see that Parmenides' monism might have been attacked as denying an apparent fact, namely, that things move. If Zeno's aims were throughout exactly those destructive ones attributed to him by Plato, then it would not make sense for Zeno to assume that "time is composed of nows". The point of a reductio is to compare your opponents assumptions, not to add assumptions of your own.

If interested, you are welcome to consider my MPhil thesis treatment of Zeno at http://www.plato.plus.com/c1.html and http://www.plato.plus.com/c2.html. In some respects I was over harsh about Aristotle there, but I stand by the thought that Zeno was presenting a reductio, and I stand by the account of that reductio and it's target.

(2) You then ask "why hasn't anyone postulated... that nothing is at rest and change is happening all the time?". They have, a chorus will announce. Usually, the central historical proponents of this kind of view are supposed to be either Heraclitus or Cratylus or both according to your reading and your translation. A complication about these usual supposings, one complication amongst many, is that the thesis that "nothing is at rest" is not equivalent to the thesis that "change is happening all the time". To see this, note that changes happen to things. Therefore, the thesis that "change is happening all the time" commits one to the existence of particular things in a way that "nothing is at rest" does not (and in my opinion the latter idea is more central to the Heraclitean notion of flux than the former idea is). This distinction matters because there are a large collection of reasons why, of late, philosophers have been averse to that formulation of your thought which speaks of change.

You want to say that far from the universe being stable all the time in every respect, it's just "the opposite". So, what you are attracted by is the idea that everything is changing all the time in every respect. In brief, there are three kinds of reason for objecting to this idea: (a) that motion is relative, (b) that the meaning of 'change' requires a contrast, and (c) that talk of change requires a context in which things are to some degree stable.

(a) Something or other, perhaps a station, has to be assumed to be stationary if the talk of a change of position is to have any sense. This consideration about the relativity of position change excludes the possibility that one can talk of everything changing position. 'Changing position relative to what?', would be the question. And in answering it one cannot but assume that something or other counts as the station for the purpose of discerning a change of position. One drawback of this line of argument is that it invites the curious debate (popular with Kantians) about absolute and relative space. In short, does one actually need a thing relative to which one is moving, or only something called an 'absolute' location?

(b) If "Everything is changing all the time in every respect" there would be nothing which we could pick out as not changing. The philosophically interesting move then is that 'this is changing' can only mean something if the contrasting negative claim 'this is not changing' could also be meaningfully applied somewhere or to something. Something about this point looks to be right. On the other hand, this contrast-constraint on meaning is undeveloped here, and every development of it seems to take us off in philosophically difficult directions. For instance, should we say that the contrast upon which a word's meaning depends must be possible for each thing to which the word is to be applied? Why? Or should we say that the contrast upon which a word's meaning depends must be possible within all the things there are? What things are there then? Or is it just some of the things that there are that matter here? Which then, and why? What is to be contrasted with what? For instance, might it be allowed to give a contrast-meaning to 'change' to say that while everything on earth is changing all the time in every respect God does not change? Or that there are contrastingly unchanging things 'in the intelligible realm'? Maybe not, but the difficulty is that giving reasons for this will involve developing a large body of thought about meaning and metaphysics and so on.

(c) If we can see that change happens to things, then there will be limits on how this idea of 'change' can reasonably be used. In order for us to talk of a change, we must be able to talk of a thing. And here comes the decisive move: it is some stability in a thing's qualities that permits us to refer to that thing, to identify it. In order for us to able to talk of a thing, there must be some stability in the qualities by which we are able to pick that thing out as being that thing. Some stability is essential to talk of things, and talk of things is essential to talk of change. Therefore, the idea that the universe is changing so much that it is "the opposite" of stable can't be made sense of.

Reason (c) is usually thought to be the most decisive against your proposal. But as I said at the outset, it matters for these purposes whether you talk of everything changing all the time, or, which is a different thesis with different implications, that nothing rests.

David Robjant


If you know something about Zeno, how come you missed out on Heraclitus? His philosophy was "panta rhei", i.e. "nothing ever stops still". Will this satisfy you?

Jürgen Lawrenz

back

Ray asked:

What are Milan Kundera's philosophical ideas as expressed in The Unbearable Lightness of Being, and what ramifications do they have on our lives?

Thank you very much for getting me to read this again. There are many philosophical ideas raised in Kundera's novel.

The main philosophical theme is of lightness and weight, an opposition which Kundera tells us derives from Parmenides and Nietzsche. Talk of lightness and weight is a metaphorical way of speaking of the positive and negative. Kundera brings these concepts to life by showing them in the lives of his characters. The philosophical question is which is good, lightness or weight, and this is a question of what we should morally value. Is it the lightness and freedom which we should value or is it the weight and the pain of moral responsibility? Tomas and Sabina stand for the lightness of being and Teresa stands for heaviness, or the weight of moral responsibility.

Kundera questions whether Parmenides was right to think that lightness is positive.

How to assess which is best? When Tomas started out with Sabina he had freedom and light but no strong relationships and no love. When he meets Teresa he is torn between lightness and weight, and begins to feel pain. Lightness seems better from a pleasure/pain point of view, but it is a withdrawal from human relationships and the heaviness they bring.

The moral of the book, though, is that heaviness is to be valued, which Kundera shows through the development of Tomas' relationship with Teresa with whom he ends up living happily. His infidelities and betrayals, when he lives outside the heaviness of the dimensions of conventional moral restraint, are shown to be meaningless when he finds he cannot remember the faces of the women he has been with. While Teresa presents herself to him as a moral obligation and a burden, limiting his freedom, he loves her intensely. Kundera says that "The heavier the burden, the closer our lives come to the earth, the more real they become".

Kundera shows that if we value the heaviness of intensity over the lightness of indifference we will become more fulfilled, like Tomas. To value indifference is to end up alone, like Sabina. Tomas comes to love the countryside, whereas Sabina ends up in a country in which she doesn't feel at home. In Nietzschean terms, weight is life-affirming in that to live with positive intensity is to live in a way you'd be prepared to repeat. The emptiness of Sabina's life, and that she wanted to "die in lightness" — which is to say that she is indifferent to her life — shows that she would not want to repeat her life and would not accept eternal return.

Eternal return, unlike values of lightness and weight, cannot show itself so easily in our everyday lives, so it doesn't seem to come out as an obvious theme, but it is connected to heaviness. Teresa, who lives under the burden of weight, has recurring dreams. Tomas, when he meets Teresa, returns again and again from lightness to weight or veers between seriousness and indifference before he is able to live with true intensity. The philosophical or metaphysical possibility of eternal return expresses the seriousness of weight. As Kundera says, if Hitler returned again and again, it would be a much more serious matter than his living just once. As it is, he has become an historical figure and an object of study and theory.

Another philosophical issue raised by Kundera is that the novel is not real life. This is raises questions in the realm of aesthetics and place of the imagination. Kundera tells us that Teresa was not born physically but issued from an idea. But he is talking of her character as a mind or soul and we accept her character. She is a coherent and consistent "person". This also raises the mind-body problem, another central issue in philosophy. Because this is a novel, we don't see her and physically she is not there. This supports mind-body dualism. We can imagine disembodied persons. As far as aesthetics goes, it can be claimed that it is "true" within the novel itself that Teresa is a person, as opposed to a dog, say. If we imagine Teresa as a real person, would not this not make it true within the novel? Kundera claims not.

But the individuation of individuals as a problem is also raised. This is another philosophical problem. Teresa tries to identify herself through her body by continuously looking in the mirror, but sometimes she sees a look of her mother. Does that mean we cannot be identical with our bodies, if we also look like another person. If we identify ourselves physically we would have to find a look specific to ourselves, but how do we separate off the look of another.

These latter problems are purely philosophical and don't really have much in the way of ramifications for real life and so it seems to that your question points to a truth that as far as life is concerned, morality and value are of central importance in our lives. Kundera might even want to say that they are "light" issues — matters of study and theory.

Rachel Browne

back

Dana asked:

I would like someone to answer this question I have been pondering for a long time. Where exactly does our (your) "Liberty" come from? As many responses would help.

P.S. Not just the Constitutional argument for Liberty, but is there a natural right to Liberty?

I am going to provide my answer from one particular philosophical perspective. There are other philosophical perspectives that will result in quite different answers. So I really hope that you get more than my answer to this question.

First, I must lay some foundation. "Life" is characterized by the unique fact that living things change and move — "act" — through the directed application of internally collected, stored, converted, and channeled energy. At a very fundamental level, the goal of all living behaviour is the maintenance of the life that is behaving. It is that (not necessarily contiguous) stretch of the DNA (or RNA) molecule that can be labelled as a Gene that is what must be recognized as the entity that survives and proliferates — continuation of which is the goal of Life's actions. The actually observed behaviour of all living creatures, both in general and individually, is highly flexible and variable but within the broad genetically defined limits of continued genetic survival.

As an example of life, as an example of the species Homo sapiens, and as an individual consciousness, our purpose is to ensure the continued survival and proliferation of our genes. To be "Good" at anything is to do a quality job at fulfilling the purpose of that thing. A good Human Being is efficient and effective, and fulfils with quality, the purpose for which the Human Being was built — to ensure the continued survival and proliferation of our genes. To ensure the continued survival and proliferation of our genes is a never ending struggle. There is never enough assurance that the job is complete. There is always something extra that can be done, some marginal increase of assurance that can be found. The struggle continues whether or not the individual is consciously aware of why they are striving, or what they are striving for. Even if they are striving under misconceptions, misinformation, or mistaken assumptions, the human animal is built to strive. The best situation is to be consciously aware of why you are striving, and employ the best of your intellectual abilities to make conscious rational choices of what to strive for. Happiness comes from knowing you are doing a good job.

In order to best provide for the welfare of himself and his family, mankind has discovered that it is a good thing to have the freedom to pursue whatever means seem to be the best available at the moment. At the same time, however, mankind has also discovered that he can better the welfare of himself and his family by co-operating with his fellow humans in projects of mutual benefit. A co-operating social group, however, must necessarily impose some restrictions on the individual freedoms of each member of the group. Otherwise, nothing would get done co-operatively. The concept of individual personal freedom within the confines of a co-operating social group is the source of the concept "liberty".

So — where does "liberty" come from? It comes from the mutual agreements arrived at by the co-operating members of a social group. "Liberty" is the freedom of action allowed by the group to each member of the group. More importantly, constraints on liberty are imposed by mutual agreement amongst the members of the group on their respective individual freedom in order that they each may best achieve the mutual benefits attainable from group co-operation.

So much for the meta-political theory. In actual practice, of course, especially in modern societies, there is little in the way of "mutual agreement" in the development of what are deemed necessary constraints on individual freedom in the interests of best achieving supposedly mutually beneficial co-operative goals. In any event, our liberty comes from the determinations of the political processes as to the extent of individual freedom we ought to have while we remain part of a social group.

I hope that the foregoing also adequately explains why we have no "natural right" to liberty.

Stuart Burns

back

Derek asked:

With Medical science advancing greatly even creating artificial life is in the realms of possibilities and probabilities. Nanotechnology is opening more opportunities for research and with the scale of human deaths reported in natural disasters there is a perception that the value of human life is diminishing. Or have I got that perceptive wrong?

In my opinion, you've got it wrong. I think that if anything humans place a teeny bit more value on life than in the past. Not much, mind you. Just a little. If you read history, you read about the history of war, for the most part. If you look at ancient methods of punishment, they routinely involve torture and slow, public death, and in many situations were occasion for parties, etc. Mostly, we don't do that so much now. One problem, of course, is that there are so many people now, and methods of killing have gotten so efficient, that in sheer number we've surpassed anything the ancient world was capable of. I've recently seen an estimate of death by war (only war) in the 20th century, at roughly 160 million. But out of 2-4 billion, that's a low percentage ("low", haha, at roughly 5 per cent). Anyway, back when the world population was numbered in the millions, the death of thousands was relatively much more telling. Whoopee, right? What can I say? Human beings, in my opinion, are, all of us (yes, me included), an insane species. About all one can do is be aware that we're insane, and try to compensate as best one can.

Steven Ravett Brown

back

Ken asked:

On the premise that early homo sapiens tribal cultures preceded the religions of the world, how/ what shaped the evolution of their moral codes/ ethics, to those of 'civilizations' today?

Has right and wrong evolved as a matter of perspective of culture, religion or the needs for survival of the species/ tribal groups?

What part has the development of speech (therefore sharing ideas/ thoughts) had on this?

Can one speculate that the pre-speech grunts of early man equate to those of a hierarchical group of apes who demonstrate a kind of code that maintains order? Or is it reason, the power of speech and thought? Can only man have religion and does it matter what religion? (back to the earlier point on evolution from tribal ethics).

It may just be infatuation with your web page, that I have put my spontaneous thoughts down here but they are the sort of thoughts that I often have. It's hard to bounce ideas off others and get their thoughts without being accused of killing off conversations with heavy topics etc.

I'm wary of theological types (perhaps unfairly) who may want to convert me and those that have a literal belief in Genesis. It may be that I just need to read the right books as a novice thinker or am I just lacking external stimulation or both.

You appear to have fallen directly into the 'evolution' trap. This is suggested by use of statements like, "evolution of moral codes", "evolution of right and wrong", "survival of the species", "the development of speech" (implying an evolutionary notion), "pre-speech grunts of early man". There also seems to be an implied notion that Genesis is the only alternative to the Theory of Evolution. One does not have to be conventionally religious to oppose the evolutionary theory. In fact, one does not have to subscribe to the often presented notion that unless an alternative to evolution can be supplied we have to accept the theory. This is a fallacy.

Before the question, "What shaped the evolution of primitive moral codes/ ethics?" can be answered, it has first to be proved that moral codes/ ethics actually evolved.In fact it is difficult to conceive of an evolution of morals/ ethics. Consider that humans are born with an innate awareness of right and wrong, in other words, if it were a fact that right and wrong are absolutes: How do they evolve? How can they evolve? Is it not the case that moral codes established within societies are dependent upon the world view established within that society, including a by no means unimportant environmental factor?

Conception of morality within a primitive society is one thing, and conception of morality in a so called developed/ civilized society is another, that the primitive concept could evolve into the civilized concept is a fallacy. In fact, members of a primitive society placed in a position where they could observe the moral attributes of modern 'civilized' society would probably hurry back into the jungle as fast as their legs would carry them.

Most concepts within a society are based on the intrinsic political and environmental foundations of that society, and are strictly pertinent to these conditions. Changing the basic structure of a society is likely to change its moral code, however, this is not evolution but adaptation. Changes can flow in any direction, backwards, forwards, side ways, etc., and adaptation follows it, hence moral concepts can actually reverse to more primitive considerations. Surely, all this is superficial to the inbuilt concept of right and wrong in the human psyche, i e absolute moral awareness. One case in point, and there are many, is the atrocious concept of morality entertained in Germany under the control of the Nazi party. This party created a very special concept of morality within its domain. Wiping out Jewish people, communists, sexual deviants, the mentally afflicted, the physically handicapped, etc., to produce a 'better' society was seen as highly desirable and very 'right'. We must not overlook the fact that this was a modern/civilized society. However, observers outside the society saw a very different picture, and judged it by the basic human intrinsic values of right and wrong. Had the Nazi notion of morality evolved? Of course not. The Nazi concept of morality was as superficial as any other politically and environmentally controlled system, and had no more effect on changing absolute morality than any other man-made political regime.

You mention speech. It seems obvious that speech is essential to humans, and certainly has an indisputable effect on the progress of humanity. Were there ever pre-speech grunts, as your bias towards evolution suggests? I don't know, and neither does anyone else. Did humans originally exist in ape cultures? I don't know, but feel that it was unlikely. You see I am biased against the Darwin/ Huxley concept of evolution, for which there is little or no proof.

I presume that only 'man' is concerned with religious concepts. Religious awareness in other forms of animal life seems unlikely. Whether one religion is better than another may depend on its moral interpretations. Some religions try to obtain their ends by violence, others by passive persuasion. However, perhaps we should not confuse the common understanding of religion with spiritual awareness, the concept entertained by Jesus and others. Unfortunately, Jesus and other great prophets have become hi-jacked by religion.

By all means read the books that take your interest, and try to work out to your own satisfaction which concepts that you encounter lead you in the direction you would wish to go. If you do not like Genesis keep away from it. However, you will find it hard to argue against someone who has read it. Ignoring reading that we do not like can often leave us in a poor position when we come to argue against it. This is the big difference between knowledge and understanding. You may have knowledge of Genesis, but the person who has read it will have understanding of Genesis. This difference between knowledge and understanding is a very important point in epistemology which is often overlooked.

John Brandon


The answer would be 'no'. For one thing, 'evolution' doesn't denote a stately progress towards better. Consult your own experience. It's not at all guaranteed to be the 'fittest' who survive, except on the triviality-constructing assumption that what 'fittest' means is just 'those who happen to survive'. On that assumption, 'the fittest survive' turns out to amount to the value free statement 'the survivors survive'.

When I say that it is 'value free' I mean that 'the survivors survive' and any equivalent statement doesn't say anything about what is good, better, best. It is important to add that 'the survivors survive' isn't at all an interest free tautology: on the contrary, it is a good example of an in interesting tautology, because the fact that the survivors survive to generate more like them is one important part of an explanation for the shape and diversity of life on earth. But that the survivors survive doesn't prove that the survivors are better than the dead morally, aesthetically, intellectually or in any other way. It just show that they survived. This applies to beliefs as well as to persons.

Now clearly, moral codes which survive, survive. But so what? That something survives is an entirely separate question from whether it is right that it should survive.

So, one might think, to attempt to account for right and wrong in terms of survival is uniquely misguided. When we think of Right and Wrong, supposing that there is such a thing, we think of notions with a binding force on all of us, namely that you ought to do right and ought not to do wrong. But if we take the evolutionary line and talk about rules for behaviour that have survived, then the mere fact of their survival up to here does nothing to mandate any attitude or response to these rules on our part. From this it might be deduced that whatever evolutionary sociologists/psychologists are talking about when they talk about the 'evolution' of right and wrong, it isn't what we are talking about when we use the words 'right' and 'wrong'.

In sum:

That it is wrong to kill people is not derived from the contingent fact that the rule 'don't kill people' happens to have survived (on and off). If anything, it is precisely the other way about.

David Robjant

back

Krishna asked:

Dear sir, I want to know the purpose of life. I am of the opinion that, if anyhow we're going to die, then why such anxiety to work while living?.Why don't I die, instead of living? Is it my basic human nature or some other mysterious principle that commands me to exist like this? Please provide a solution. I have been worrying about it for the past 2 years.

First, an important disclaimer. I am a realist/ materialist. I am not an idealist or a dualist. So my answer to your question will exclude any reference to religious or spiritual concepts. For answers from those perspectives, you will have to seek guidance from your friendly priest, minister, or spiritual advisor. I am sure you will have no problem finding a suitable representative of whatever religious faith appeals to you (or that you happen to stumble across). And they will tell you that your purpose in life is to unselfishly and altruistically dedicate your existence to the glorification of whatever notion of God they propose. You will have to take their word for it, of course.

On the other hand, if you are seeking an answer that you can check out for yourself, then you are seeking a materialist answer where science and evidence have a meaningful role to play. The answer I provide here will not be met with agreement by many. It does, however, have the advantage of being consistent with all that we currently know about biology, evolution, and psychology.

The first step in answering your question from this perspective is to acknowledge that you are a member of the species Homo sapiens. As such, you are a primate, a mammal, an animal, and a living organism with a 3 to 4 billion year evolutionary history behind you. (I refer you to any of the numerous works on evolutionary biology for further argument on this point). The argument goes like this:

Life is Action. "Life" is characterized by the unique fact that living things change and move — "act" — through the directed application of internally collected, stored, converted, and channeled energy. > Life's Actions are Teleological (Goal Oriented). At a very fundamental level, the goal of all living behaviour is the maintenance of the life that is behaving.

The second step is to acknowledge that the "thing" that has been evolving over the myriad of generations that have lived since the dawn of life on Earth, is the genetic code and not the individual. You, yourself, are but a bio-chemical machine. You were constructed by the fertilised cell that was the result of the union of your mother's ovum and your father's sperm. And you were constructed in accordance with the recipe encoded in your genes. You are a survival machine for the genes in your DNA. (I refer you to the works of Richard Dawkins for further argument on this point.) The argument goes like this:

The Gene is the Unit of Life. It is that (not necessarily contiguous) stretch of the DNA (or RNA) molecule that can be labelled as a Gene that is what must be recognized as the entity that survives and proliferates — continuation of which is the goal of Life's Actions.

The Reproductive Imperative. The actually observed behaviour of all living creatures, both in general and individually, is highly flexible and variable but within the broad genetically defined limits of continued genetic survival. > Our Purpose. As an example of life, as an example of the species Homo sapiens, and as an individual consciousness, our purpose is to ensure the continued survival and proliferation of our genes.

The Definition of Good. To be "Good" at anything is to do a quality job at fulfilling the purpose of that thing. A good Human Being is efficient and effective, and fulfils with quality, the purpose for which the Human Being was built — to ensure the continued survival and proliferation of our genes. > Better is Never Enough. To ensure the continued survival and proliferation of our genes is a never ending struggle. There is never enough assurance that the job is complete. There is always something extra that can be done, some marginal increase of assurance that can be found.

And Finally, the struggle continues whether or not the individual is consciously aware of why they are striving, or what they are striving for. Even if they are striving under misconceptions, misinformation, or mistaken assumptions, the human animal is built to strive. The best situation is to be consciously aware of why you are striving, and employ the best of your intellectual abilities to make conscious rational choices of what to strive for. Happiness comes from knowing you are doing a good job.

That then, is your answer. The meaning of your life, your function, your purpose, the reason you exist, is to ensure that your genes get transmitted to the next generation. The point of it all is the welfare of your genetic descendants (over the long run, of course). Go to work because it is the best means available to you at this time, and in this place, to prepare you to do well by your children. You are not here to be good for society. You are not here to become whatever God might have intended. You wake up every morning and tackle the day because you have a function to perform. Friends, family, and society matter only to the extent that they can contribute to your ultimate purpose in life.

Many people will object to this answer, including many professional philosophers and of course anyone with a religious/ spiritual bent. But any alternative they offer to my answer will come either from their religious or spiritual premises (which I have specifically disavowed), or from out of thin air. As humans we are gifted with the ability to choose alternative goals in life. And you are free to pursue whatever ends tickle your fancy.

However, regardless of what other goals may be offered instead, if you are not successful at fulfilling this evolutionary meaning of your life, then your genetic codes (and their 3 to 4 billion years of ancestry) will vanish from the future. You are here to ask the question you asked because your parents (and their parents, and their parents, etc.) were good at their job. The future will be populated by individuals whose ancestors were successful at this evolutionary purpose. Are you going to be an ancestor, or a dead end?

Stuart Burns

back

Alex asked:

What is the answer to this question?

This is.

Kim Boley


It lacks content.

Rachel Browne


Hey, neato.

1) Why should every question have an answer?

2) "What is the answer to this question?" = "What is the answer to, "What is the answer to, 'What is the answer to,"..."'"?"

3) As you can see from formulation 2), in actuality, this question is never asked, i.e., you never get to the end of the regression to find the actual subject of the question. So until you do, there is no answer; or better, really: until you do, there is no question.

Steven Ravett Brown

back

Mildred asked:

Why are people not contented with what they have? Even when they're already successful they still aren't happy, they're are still searching for something. So why is that?

The answer follows on from my answer to Krishna, above.

To be "Good" at anything is to do a quality job at fulfilling the purpose of that thing. A good Human Being is efficient and effective, and fulfils with quality, the purpose for which the Human Being was built — to ensure the continued survival and proliferation of our genes.

To ensure the continued survival and proliferation of our genes is a never ending struggle. There is never enough assurance that the job is complete. There is always something extra that can be done, some marginal increase of assurance that can be found.

The struggle continues whether or not the individual is consciously aware of why they are striving, or what they are striving for. Even if they are striving under misconceptions, misinformation, or mistaken assumptions, the human animal is built to strive. The best situation is to be consciously aware of why you are striving, and employ the best of your intellectual abilities to make conscious rational choices of what to strive for. Happiness comes from knowing you are doing a good job.

Stuart Burns

back

Madeleine asked:

Is there a place for philosophy in the modern world?

Paul asked:

Why is philosophy important?

April asked:

The mind is a beautiful thing...

How does it fit into philosophy, and why is philosophy so popular?

David asked:

I would like to ask you, Why did you choose to be philosophers? and why do you think philosophy is such as important part in our lives, and if philosophy didn't exist what would be of us?

Mauricio asked:

How does science modify philosophy?

Shemel asked:

What does it take to be a philosopher? Is the philosopher someone of great importance?

MD asked:

What is a philosopher? what does a philosopher do?

Sabrina asked:

What type of questions does a philosopher ask?

Yes, there is an important place for philosophy in the modern world and it has to do precisely with those things that our culture presupposes and assumes uncritically. The mind may be a beautiful thing but the beliefs and concepts that constitute it need maintenance if they are to cohere and cope with changes. Some would say they need a more radical overhaul.

You only have to look at the questions on this site to see why philosophy is important. Questions about truth, reality, meaning, God, selfhood, justice,love. These are the things that matter to us, not just as curious individuals but as a species — an endangered and not altogether happy species. Despite our resourcefulness and optimism, as manifested in artistic and technological achievements, we are vulnerable to disaster. One source of disaster is the natural world, but another is our own conceptions. Ways of thinking change. Values change. Every so often we reconceive the relationship between nature, society and individuals. Religion gives ways to materialism, then materialism itself is undermined. Absolutism gives way to relativism and that to subjectivism: there is no ultimate truth, so believe what you like. New technologies make possible what previously would have been considered miraculous, but being possible doesn't make something right or prudent. Philosophy is a discipline of thought that tries to keep track of the conceptual changes and to cling to whatever unchanging conceptual truths or necessities it can discover. This is an important task. In every age there must be people who can say "We don't have to think like this" — who can conceive of different conceptual schemes, who can orientate current concepts of truth, reality, selfhood, justice against previous and alternative schemes. No doubt, sometimes, in trying to sort out the knots that conceptual change gets us into, philosophy contributes to tighter entangling.

It is not necessarily philosophers who communicate their ideas best to the general public. Their ideas become incorporated into the intellectual chatter and the literature and art of the day. They become part of a wider cultural project of explanation and justification. Artists and scientists cannot avoid philosophy, whether they read it directly or not.

Here are some big questions that philosophers have a role in discussing. What is consciousness? What, if anything, can I ever know about myself? Do I ever act freely? Is a human being more than an animal; are the differences ones of degree or of kind? If scientists could manipulate human genetics to the extent of making us immortal, should we permit it, should we aim towards it — and what would be the consequences for other concepts? Is deliberate human extinction, for the sake of the survival of other life, compatible with morality? Would the world be a better place if everyone shared the same beliefs? Are we living in a make-belief world, with a false view of reality and a false view, even, of our own subjective experience?

Of course, professional philosophers, like professional scientists, are working at a highly technical and precise level on sub-questions of the bigger questions. They may sometimes lose sight of the big questions. They will generally be working in a university and will have teaching and administrative duties. They will have to keep themselves up to date in a much wider sphere of learning than just their own specialism. But for some of their time they will be thinking and writing, rethinking and rewriting. They will discuss their ideas with colleagues, then think and write again. It is, like most things that are worth doing — despite what Bart and Homer Simpson may tell us — a painstaking occupation. But, the importance and influence of a major philosophical work or theory (to which many philosophers may have contributed) should not be underestimated. It can reorientate us to the world we live in and to the way we experience it.

Graham Nutbrown

back

Stephen asked:

I'm looking for a good way to introduce philosophy to children. I have 3 younger siblings, aged 7, 9 and 12, who are very inquisitive, and wanted to know what I was learning when I was taking my philosophy course, but unfortunately found the subject simply confusing when I attempted to explain it to them!

I've read Sophie's World myself, but can't see them reading it themselves, and I think it would be better to introduce the subject to them in bitesize chunks, and on more contemporary issues, rather than elaborating on its history as that book does (which, although perhaps a necessary approach for scholars, is likely to bore young children!).

Does anyone know of any basic introductory books to the subject for children of those ages? Or, preferably, simply some clues as to how to get started? I particularly want to teach my siblings how to be open-minded and to think for themselves (although perhaps not too rebelliously!), as well as to appreciate (and understand) other worldviews, and it seems a bit of philosophy would be fairly good in that department.

Unfortunately, my family (the adults in particular, besides me that is) seem generally not interested in thinking philosophically, and tend towards blindly accepting black-and-white views of the world (or perhaps just get into lazy thinking habits!). The kids are naturally curious about the world, however philosophy is unfortunately not a subject on the school curriculum.

So, any ideas how to approach the subject with them?

If you look at the Pathways site you will find a link to SAPERE.net which provides a guide to teaching philosophy to children. In Download free e-texts you will also find the Continental Community for Inquiry which is being successfully used in Australia for teenagers in school and you could probably make use of this — there is an introduction on how to go about using the chapters, each of which is thought provoking. You can also look at the papers which are printed on Pathways and see that Katharine Hunt is also interested in teaching children.

>From experience, I have found that only some people are interested in philosophy. The desire to think philosophically seems to be a sort of disposition some people have, like a disposition to religion or science, and I'm not sure it should be imposed. Perhaps the curiosity of your siblings isn't a philosophical curiosity.

So, good luck.

Rachel Browne


I think the best approach to teaching children to think philosophically is to start with getting them to think logically and enjoy using deductive skills. A truly great book is Symbolic Logic by the Reverend Dodgson (Lewis Carroll, to you and me). I hope it is still in print because it is full of logic puzzles that would appeal to the age range you mentioned.

Kim Boley

back

Trisha asked:

Should religious doctrines and practices be regulated according to their moral worthiness, or should religion be permitted to operate free of outside interference?

How would Socrates respond to this question?

As to your first question — the answer is — Neither!

What characterises almost all religious beliefs is the adoption of some particular form of moral standard. In fact, one might almost define a "religion" as the adoption of some particular moral standard. Certainly one can identify the flavor of religious belief involved by knowing the particular moral standard adhered to. The problem is that, by the very nature of religious belief, there is nothing that might tend to generate any commonality of moral standards between different religious beliefs.

Now, I am assuming here that you are intending your question to be one of regulating or freeing the doctrines and practices of different religions. Obviously (I hope), within the confines of one particular religion the moral standards of that religion should be (quite properly) used to regulate the particular doctrinal interpretations and practices of its adherents.

But since moral standards are particular to specific religions, it would be impossible to regulate the doctrines and practices of one group of religious believers using the moral standards of another religion. Not, at least, without being (properly) accused of arbitrariness and unjustifiable coercion. How do you think a Christian would react to being governed according to the moral standards of Hinduism? Or vice versa? (How do you think MacDonalds might fair in a regulatory environment where cattle are sacred?)

On the other hand, granting unrestrained liberty to any religious belief is not the answer either. Properly interpreting the "Word of God" (whichever God might apply) is an exercise in unconstrained imagination. There are absolutely no limits to the doctrines and practices that can be dreamed up and claimed as "religious". Rastafarians smoke marijuana as part of their religious ceremonies. Voodoo demands animal sacrifices. Certain sects of Devil Worship and some ancient Incan religious beliefs demand human sacrifices. Most so-called "civilised" societies frown on such behaviors.

The proper approach, I think, is to identify a single natural moral standard that is based on nature and not on religious beliefs, and then constrain people's flights of religious fancy by that moral standard. I would suggest to you that the proper moral standards to apply are "no initiation of the use of force" and "accept responsibility for the consequences of your actions". With these two secular moral standards enforced, I think society can tolerate just about any other religious doctrine or practice.

As to your second question — Who cares?

Socrates lived over two millennia ago, in a social environment pretty homogeneous (everybody he ever met believed either in the Greek Gods or the Egyptian Gods — and there was not a lot of difference between Greek and Egyptian moral standards). Socrates never had to face the religious diversity one can find in any moderately sized modern metropolis. How could his thoughts on the matter have any practical bearing on the modern problem?

Stuart Burns

back

Paco asked:

How do I know that you will not kill me in my sleep?

This has got to be one of the oddest, silliest questions I've seen here, and that's really the only reason I'm addressing it...

Now, in answer to your question: there is no absolute certainty, if that's what you want "knowledge" to be, that I won't. There you go, simple and complete, right? You also don't know that someone else won't kill you in your sleep, or for that matter, that I won't pour maple syrup over you in your sleep. In addition, the ceiling might fall down and turn into rose petals as it falls. Or snakes might rise up from the cracks in your floor, do a dance, and turn into vines around your bed. Or if you want another paranoid fantasy, perhaps you're being filmed through hidden cameras in the nearest walls as you sleep... um, by the CIA, or maybe the NFL. Why not? Hey, as long as we're speculating... There is nothing at all that makes anything absolutely certain... we just assume that the world won't wobble and fall into the sun in the next few minutes. We have all sorts of "laws", i.e., inferences, that convince us, for example, that our fingernails won't start showing cable TV channels... but they're all empirical, and so subject to doubt. So go right ahead and spin whatever fantasies you want to... or you might go out and buy some good science fiction...

On the other hand, you might rethink what you should mean by "know". It is possible that a realization to the effect that you are at this moment seeing a shade of red is certain knowledge; or that you are at this moment feeling pain... some will argue that. Not me, however. But I don't hold that kind of certainty as a standard for knowledge, either. By my standards, I know that the earth will not fall into the sun, and that I will not kill you in your sleep. I go by likelihoods... and so does science. That's about the best you can do... and that's knowledge.

Steven Ravett Brown

back

Harry asked:

If God Exists, are cancer, aids, other diseases and such his idea of birth control /population control for the Earth, in order to achieve the greater good?

If they are, then I think God's open to quite a bit of criticism. You'd think that a perfectly good omnipotent being could design a more humane method of birth control — or a world that can support more people.

Jonathan Ichikawa

back

Carla asked:

What were the childhood influences that led Karl Marx in his political direction?

Is it fair to say that religious fundamentalism is evil?

Well, that depends!!

It depends first on just what you mean by "fair". And it depends even more on the standards of good and evil that you wish to apply. Certainly, to a religious fundamentalist the answer is a resounding "No!!".

But then, by the very nature of your question, I would have to assume that you are not a religious fundamentalist. So the answer depends primarily on the standards of good and evil that you apply. According to some standards, the answer would be "Yes — religious fundamentalism is evil". By others, the answer would be "No — religious fundamentalism is not evil".

By my own personal moral standards, I would come down on the "Yes" side. I hold the highest moral regard for knowledge and learning about how Reality behaves. I therefore consider the studied ignorance of religious fundamentalism to be a particularly bad way of learning how to predict which way the tiger will jump. But then, my moral standards are not very common, and many people would disagree with me.

Stuart Burns

back

Annie asked:

My ethics class recently read Thomson's "Killing, Letting Die and the Trolley Problem", Kagan's "The Additive Fallacy" and Taurek's "Should the Numbers Count?" I'm writing a paper which will examine the idea that, when making moral decisions or morally evaluating a situation, the numbers shouldn't matter, suffering isn't additive, etc. Can you suggest any other readings which may be helpful, either supporting or opposing the views in these articles? I wish to focus on Taurek's "Drug" scenario: Taurek owns the only dose of a certain drug; one person needs the whole dose to live; five people each need 1/5th of a dose to live; none of the six have any claim on the drug or connection to Taurek; to whom should he give the drug? Taurek claims it's morally permissible to give the drug to the one. I understand his reasoning yet still find it difficult to agree with Taurek, that five people dying is not a worse outcome than one person dying.

For a non-aggregating view that does require that the five people be saved in the drug case, check out Tim Scanlon, What We Owe to Each Other.

And for a good rebuttal of both Taurek's and Scanlon's non-aggregation, see Alastair Norcross, "Contractualism and Aggregation http://www.ruf.rice.edu/~norcross/ContractualismAggregation.pdf, Social Theory and Practice Vol. 28, No. 2, April 2002, available on his web site: http://www.ruf.rice.edu/~norcross/Norcrosscv.html

Jonathan Ichikawa

back

Brian asked:

Question 1. I keep reading that a signified cannot exist without a signifier. Does this mean that concepts cannot exist without symbols? And more important (at least to me) is their any proof for this assertion?

Question 2. From reading about the Grandi series I learned that 1+1-1+1...ad infinitum equals 1 and 2! I have also read about a thing called a Thompson lamp. This is a lamp that is on for half a minute, then off for 15 seconds... each time being on or off for half the preceding time. Combined with the logic of the Grandi series this means that the lamp is both on and off at the end of the minute! In logic if 'P and not-P' is true then anything can be proven. Therefore, if time is infinitely divisible, the planet Pluto is furry! How can logic avoid this result? Or is this proof that time cannot be infinitely divisible?

Your first question is an interesting one... but I'm not sure how to answer because I don't know how much you've read. We could get into the whole Fregean vs. Husserlian vs. cognitive vs. etc., intentionality debate here, which is pretty ghastly. The symbol vs. the referent vs. the meaning... well, it's just too much for the space here. You might look up Frege, and then his commentators, on this if you want to stay up nights thinking about it.

Let me say this much, however. If what you've read has to do with epistemology, then you've got a debating point; i.e., if in, "a signified cannot exist without a signifier", the word "exist" refers not to some ontological claim but to a claim about knowledge, then I think that whoever you've read is wrong, but that you have tapped into a very large debate. Check out the stuff above. I actually think the claim is rather easily refuted by looking at cognitive science, viz., infant and animal studies, but that's another point. If what you've read is making some sort of ontological or metaphysical claim, i.e., that you can't claim something exists in any sense unless you can symbolize it, then, zowie, you're into hard-core blather as far as I'm concerned. Unless you have a very broad definition of "symbolize" or "signify", which includes things like plants using the sun, etc. In other words, does the sun exist for a plant? Um... yes. Does the plant know the sun exists? No.

Do concepts exist without symbols? Ok, you tell me what the term "symbol", um, refers to, and I'll be happy to answer that. But first, before you do, think about animals and infants. Do they have concepts? Whoops... now you have to be clear on what a "concept" is, don't you. It's beginning to look like a vicious circle, to say that you can't have concepts without symbols, because it turns out that the usual understanding of a concept is something that can be... yes, symbolized. So how do you get around that? Well, as I keep saying, you look at creatures without symbols, i.e., animals and infants. Do they have concepts? Yes, yes, yes. Just look at the literature.

Q2: Much less interesting. You need to read about orders of infinity, i.e., G. Cantor, and you also need to read some basic math about convergent series, Zeno's Paradox, etc. I'm not going to get into this, except to say that you clearly do not have the background at this point to understand the situation. Read about convergent vs. divergent series in any basic math textbook.

Steven Ravett Brown

back

Niki asked:

Examine Einstein's claim that "God does not play dice with the universe" in the context of the teleological argument for the existence of God. Consider both strengths and weaknesses of the argument.

How valid do you think the argument is as proof of the existence of God?

The "teleological argument" (or an argument from design) is an argument for the existence of God based on apparent evidence of design in nature. Although there are variations, the best known formulation was done by William Paley (British philosopher-theologian 1743-1805) in his "Natural Theology", published in 1800. For a thorough defeat of this argument, see Daniel C Dennett's "Darwin's Dangerous Idea".

In a general way, the argument goes:

1. In all things we have experienced that exhibit design, we have experienced a designer of that artifact.

2. The universe exhibits order and design.

3. Given 1, the universe must have a designer.

4. The designer of the universe is God.

Within this context, Einstein's comment (that "God does not play dice with the universe") can be interpreted as an objection that the randomness of Quantum Mechanics violates the second premise — that the Universe exhibits order and design — and therefore must be wrong. One would have to understand thereby that Einstein considered the randomness of dice and Quantum Mechanics as a demonstration of disorder.

Interpreting Einstein's comment within this context is a rather weak approach to understanding his point. A better approach would be to understand Einstein's concept of the Universe and God's role within the context of Newtonian mechanics. Until the development of Quantum Mechanics, the popular conception of the Universe was of a clock-work mechanism. A deterministic mechanism wherein one could predict the infinite future if one knew the position course and speed of all the multitude of parts. Einstein grew up within this conception of the Universe, and a God that created and/or managed such a mechanism. For him, therefore, the randomness of Quantum Mechanics was an affront that violated his conception of God as a "Devine Mechanic" and the Universe as predictable and deterministic.

The only strength that the Teleological argument has left for it is its apparently logical formulation. It is thereby convincing to many who know little of modern science. Dennett, in his "Darwin's Dangerous Idea" has done a thorough job of removing any evidentiary foundation behind all variations of the first premise. And has thus rendered the Teleological Argument quite impotent as an argument for the existence of God.

Stuart Burns

back

Vicky asked:

What are some of the weaknesses of the Ontological Argument?

Ontological arguments are arguments, for the conclusion that God exists, from premises which are supposed to derive from some source other than observation of the world — e.g., from reason alone. In other words, ontological arguments are arguments from nothing but analytic, a priori and necessary premises to the conclusion that God exists. For some of the weaknesses in these arguments see — http://plato.stanford.edu/entries/ontological-arguments

Stuart Burns

back

Lelsie asked:

Please compare Hobbes' and Locke's political philosophy.

Thomas Hobbes (1588-1679) and John Locke (1632-1704) are described as Social Contract theorists. The legitimacy of political power and citizen's observance of it arises from a contract made between the future rulers and the future ruled.

The state of nature

Both maintain that prior to organised political society, human beings existed in a state of Nature. Hobbes maintains that due to human nature, human beings are incapable of living together in peace and order. Their nature inclines them to a restless striving for the satisfaction of their desires. The more power a person has at his disposal the greater the chances of securing and assuring their appetites. Competition conflict and the war of each against each ensue. In this natural state of men which is also a condition of war, life is 'solitary. poore, nasty, brutish and short'.

For Locke, Hobbes conflates the condition of war with the state of nature. People can indeed live without such tumults as they abide by natural Law and Right as prescribed by Reason and / or revelation by God of his commands. Transgressors of Natural Law and other disputes are judged and punished in accordance with its dictates as executed by individuals themselves. This is each individual utilising their executive power of Natural Law.

Political Society

Hobbes writes that the condition of war is left behind when people come together to make a covenant with each other. This covenant gives absolute authority to one of their number (or assembly of men) who then becomes sovereign and they, his subjects. The sovereign rules over them maintaining peace and security. Absolute power is necessary, as any limit of such power would provide scope for the emergence of the condition of war due to human nature. Subjects and their descendants, on the basis of the covenant they have made, have no right to disobey the sovereign or seek his/her replacement or removal. The sovereign nominates his/her successors.

With Locke, the state of nature is left when a social compact is made between people and the future governing power. This is not to avoid a state of war but to overcome the differences of opinion and judgements concerning the implementation of Natural Law. Each pools their power to execute natural law and transfers it to the government. The government enforces natural law through the very laws it makes for the common good of the citizens.

When the common good is no longer furthered by government, the citizens have the right to seek its replacement. Tyranny is immoral and has no basis in any Natural Right or Law.

Property

Property exists by the assent of the monarch, who distributes his lands as he sees fit, for the health of the Commonwealth. Just as he bestows property, he can take it away. Corporations, enterprises, laws of exchange and measure exist only through the charter and regulation of the monarch or his appointed magistrates. There is no intrinsic right of individuals to own property. It exists only at the behest of the Sovereign.

In the Lockean society, the protection of life, liberty and estate — the properties of citizens — is maintained through the observance of the common good which is based on Natural law. It is one of the reasons for the establishment of political society. Transgression of this common good constitutes a reason for the replacement of government. Property is defended by Locke on numerous grounds.

Firstly, because God commands men to labour and Natural Law requires their preservation, men labour in order to acquire the means of life. Secondly, labour, in picking an acorn or apple, has removed it from the state nature left it in. By this very act of removal, labour has 'mixed' itself with nature making it the property of he who so labours. Thirdly, although God may have given the world in common to all men, men by their labour remove things from this common stock. If permission had to be sought from all men to release it from that common state, men would have died of starvation which is contrary to the first point above. Fourthly, labouring, in removing an object (such as a field) from the common stock of nature, increases its fertility or productivity. This improves the chances of human survival as required by the first point. Finally, money is later used to exchange property, including human bodies/labour, at specific values.

Whilst both Hobbes and Locke can be described as Social — Contractarians, with Hobbes, once the contract has been made, Sovereignty lies exclusively with the Monarch or Assembly of Men. Government has power over the people. With Locke, sovereignty lies with the people — or at least, the owners of property. People have power over their government.

Martin Jenkins

back

Lucy asked:

I have two if that is ok.

1) Are liberal democracies more peaceful?

2) Is the language of rights the best way to protect peace?

I realize that these two answers are probably too short to really answer your questions, but -

1) No liberal democracy has ever attacked another liberal democracy. Does that make them more peaceful? I suppose it depends on what you mean by "peaceful". Liberal democracies have gone to war to protect themselves (WW1 & WW2, to cite just 2), they have gone to war to protect what they have deemed to be the "vital interests of their citizens" (Korea, Vietnam, to cite just 2), and they have gone to war to "protect the peace" (Iraq 1, Iraq 2, to cite just 2). Due to the political difficulties inherent in convincing a suitable portion of their citizenry that war is justified, liberal democracies are in general slow to go to war. The history of warfare in the 20th century would seem to suggest that liberal democracies have been, by far, responsible for fewer deaths in inter-state and intra-state conflicts that any other form of government.

2) The language of rights is the best way to protect the continued efficacy of liberal democracy. Liberal democracies are not defined only by the presence of a government by an elected majority. Liberal democracies are also defined by the civil concept that the rule of law is superior to the powers of both elected representatives and empowered civil servants. Key to that concept is the principle of citizen rights which cannot be trampled upon by the majority will. So I would suggest, if the above argument successfully defended the hypothesis that liberal democracies are more peaceful, that the language of rights is also the best way of protecting the peace.

Stuart Burns

back

Toni asked:

Are mind, brain, and soul all the same entity? Is it sensible to attempt to speak of anything other than brain at all?

Different world views produce different concepts of mind, brain and soul. From a dualist point of view, mind and brain are separate entities, though in some way related. For example mind is envisaged by some to be an emanation from the brain. Others harbour the idea that mind and brain run parallel to each other, changes in one reflect changes in the other, but there is no actual interaction between the two. Monist views press the notion that everything is constituted of either mind stuff or matter stuff ; in this case mind somehow invents the idea of matter to give the 'illusion' that we live in a material world, solving the problem of matter originating from nothing. On the other hand, if matter is 'real' then mind somehow arises from the interaction of the chemical and physical properties of the matter stuff.

With regard to 'soul', this, for many, is interchangeable with the term 'mind' . However, there often seems to be a religious aspect to 'soul', which is linked with the notion of survival after death of the body. Some schools of thought, particularly religious factions, recognise three dimensions of existence; these are brain, mind and soul; where brain represents the 'engine room' of the material body, which is controlled by the soul through the medium of the mind: mind in this case is intermediate between the physical and the spiritual. We can, though, from time to time feel the direct influence of the soul in what we call conscience, indicating to us what is 'right' and what is 'wrong'. If there was space and time to pursue this, we could find some links, by good use of the imagination, with the philosophy of Immanuel Kant.

Where the terms 'soul' and 'mind' are seen as interchangeable terms meaning virtually the same thing, then the problem is obviously less complex, and survival after death can be either referred to as survival of the mind or as the soul leaving the body. Generally speaking, the term soul has been handed down to us from ancient philosophy, e.g. Pythagoras, Plato, Aristotle, and others. This has been latched on to by religion to support the notion of life after death. Mind has become more prominent through science and 'modern' philosophy.

You may be able to deduce a satisfactory answer to your second question by consideration of the views outlined in answer to your first question. It depends on your world view, if you are a fundamental materialist, then you will have no hesitation in claiming that the brain is the centre of things and that the death of the brain means total death and no life hereafter. Minds and souls offer some spiritual optimism, and guide us towards the idea that there must be some purpose in what we call 'life '.

John Brandon


They might be the same entity if all entities are physical.

The mind is not physical, but might be an aspect of the physical. To talk about the brain is to use a different language than when talking about the mind. To talk about the brain is to use language of neuroscience. It might well be sensible to talk about the frontal lobe and the basal ganglia if you are a neuroscientist, but it doesn't convey much information to the ordinary person. In talking about the mind we use concepts which are familiar to all of us, such as thought, consciousness, feeling, intuition and sensation and although we don't have much of an idea what these are, they are strangely familiar.

Rachel Browne

back

Brandon asked:

How can people have free will? According to science everything is based on a fundamental law. Even in quantum physics where the molecules are supposedly acting randomly. Or when we assume we are telling our arms to move when its all just a law of impulses in the brain. I don't believe in free will. Rather I belive in restrained free will. What do you think?

I would highly recommend Daniel C. Dennett's Freedom Evolves. I considered this an excellently written exploration of this very question. I feel sure you will enjoy it.

Stuart Burns

back

Stephen asked:

What exactly constitutes a psychological theory?

It seems that such theories aren't limited to purely scientific (experimental) psychology. A psychotherapist, for example, may theorise regarding a client that the person is (unconsciously) avoiding getting close to people because of a loss of a parent during his childhood.

As well as being derived from the psychological theory of the unconscious (a general theory regarding the human mind and behaviour), the therapist in this case appears to be forming a theory (with this analysis) regarding the client's behaviour (and also, the client himself, if told the therapist's theory directly, may in fact openly disagree with it, as it may not seem intuitively correct to the client, given his own views of his own behaviour.)

Such a theory as the therapist's given above is obviously based on observation combined with background knowledge (from human psychology, which may in fact be scientific). As this theory is regarding a specific instance rather than a general one, it is clearly not a scientific theory (although general psychological theories can be scientific). However, it still appears to be a psychological theory, which raises the question — what exactly is a psychological theory?

To use an analogy with other areas of science, if, for example, a newly-built aircraft crashes, and its design is then examined and a flaw found, it could be theorised that due to this flaw, the aircraft was not built strictly enough according to the laws of aerodynamics, and thus that this is the reason for the aircraft's crash. But this analysis, although derived from scientific theory regarding aerodynamics, seems quite different from the psychological analysis mentioned above.

What confuses the matter more, is that psychological analyses often postulate emotional states in the person being analysed (e.g. 'he is scared of abandonment due to the early loss of his parent'). If asked, the person may reply that he is NOT scared of abandonment — this is how he feels (about how he feels). However, the psychological analysis (or theory) may also postulate that this is an 'unconscious fear', or that the client is 'in denial regarding his fear'. It seems strange for such a (specific) theory regarding a person's emotions to override how the person actually believes he or she feels.

Such 'psychobabble' is very common nowadays amongst how lay people (not just therapists) think about their own and others' behaviour. In cases where such theories appears to clash with a person's own intuitions regarding their own emotions (and motivation), however (which may well not prove the theories wrong), could it be said that such postulated theories regarding a person's emotions are almost in a different language to the person's own descriptions of their own emotions, which may openly contradict such postulated psychological theories?

a) Why does it matter what "psychological" means? Do you mean: what differentiates a theory involving mental states from one involving only physical observables? Now, that's a fairly interesting question. But if you just want a definition of "psychological"... well... yawn.

b) True enough. And?

c) True enough. And?

We all form theories, all the time. Look at these:

Gopnik, A., C. Glymour, D.M. Sobel, L.E. Schulz, T. Kushnir, and D. Danks. 2003. "A theory of causal learning in children: Causal maps and Bayes nets." Psychological Review In Press.

Gopnik, A., and A.N. Meltzoff. 1998. Words, thoughts, and theories. Edited by L. Gleitman, S. Carey, E. Newport and E. Spekle, Learning, Development, and Conceptual Change. Cambridge, MA: The MIT Press.

Gopnik, A., and D.M. Sobel. 2000. "Detecting Blickets: How Young Children Use Information about Novel Causal Powers in Categorization and Induction." Child Development 71 (5):1205-1222.

d) Who says it's not a "scientific" theory? What's a "scientific theory", such that it contrasts with a "psychological theory"? You seem to have some idea that there is something which you call a "scientific theory". Ok... what exactly is a scientific theory?

e) First, if it's an "other" area of "science", then what you're calling a "psychological" theory is by your own admission a "scientific theory". Whoops. You'd better get your ideas clear here; maybe you can, but I certainly don't know how to put a fence between such areas. As for your crash argument... well, psychologists would say that their theories explain "mental crashes" just as the aerodynamic theory explains physical crashes. Can you demonstrate them wrong?

f) Yes, they do that. And? Has anyone ever seen a quark? Will anyone ever see a quark? Take a look at the introduction to: Fodor, J. A. 1975. The language of Thought. Edited by J. J. Katz, D. T. Langendoen and G. A. Miller, The Language & Thought Series. Cambridge, MA: Harvard University Press, for an extended analysis and defense of constructs in a variety of fields.

g) Why is talking in non-layman language about constructs which are not readily observable "babble" of any sort? Technical language happens in all fields where the theories and/or data move people beyond ordinary usage. Is mathematics "mathobabble"? Why or why not? What about medical terminology? What if someone has some particular disease, but thinks they have something else. The doctors speaking of their symptoms are other "cases where such theories appears to clash with a person's own intuitions regarding their own", are they not?

I think that you are struggling with either or both of: 1) what justifies empirical knowledge, and 2) what justifies hypothesizing theoretical constructs. These are I hope the questions underlying the above. If all you're asking about is whether some few constructs in psychology are justified... that's a pretty trivial question. The answer: most are, to a reasonable extent; some aren't. Science investigates the basis of that justification. But if you're asking the former question(s), then you're on more interesting and more shifty ground... and it pertains to much more than just psychology. Fodor's book, above, will plunge you deeply into 2). 1) is really brutal; basically you need *lots of background in philosophy of science and epistemology to begin to approach it... and unfortunately you need much of #1 in order to really understand #2.

Oboy. Well, here, these will knock a few chips off those:

Hanson, N. R. 1965. "Notes toward a logic of discovery". In Perspectives on Peirce, edited by R. J. Bernstein. New Haven, CT: Yale University Press

Hanson, N. R. 1965. Patterns of discovery; an inquiry into the conceptual foundations of science. Cambridge [Eng.]: Cambridge University Press.

Kitcher, P. 1993. The advancement of science; science without legend, objectivity without illusions. New York, NY: Oxford University Press.

Klahr, D., and H. A. Simon. 2001. "What have psychologists (and others) discovered about the process of scientific discovery?" Current Directions in Psychological Science 10 (3):75-80.

Kuhn, T. S. 1996. The structure of scientific revolutions. 3rd ed. Chicago, IL: The University of Chicago Press. Original edition, 1962.

Piaget, J. 1971. Insights and illusions of philosophy. Translated by W. Mays. New York, NY: The World Publishing Co.

Popper, K. R. 1968. The logic of scientific discovery. Translated by K. R. Popper, J. Freed and L. Freed. English, 1958 ed. New York, NY: Harper & Row.

Toulmin, S. 1960. The philosophy of science: an introduction. New York, NY: Harper & Row. Original edition, 1953.

Steven Ravett Brown

back

Eman asked:

Hi professor, would you kindly tell me the differences between effectiveness and efficiency in management?

Give at least five differences and how to measure them.

"Efficiency" is doing something right. How to measure how right (well) you are doing, naturally depends on what it is you are doing. You might measure "widgets processed per unit time", or "cost per unit output", or perhaps "patients still alive".

"Effectiveness" is doing the right thing. You measure that by the contribution of your effort to the corporate bottom line (profit). It doesn't matter a whole lot whether your particular management unit is a profitable profit center or lowest cost cost center. What matters is whether your management unit can be shown to contribute directly to the corporate bottom line. To quote Peter Drucker — "There is nothing so useless as doing efficiently that which should not be done at all."

Since we are talking about two separate fields of discourse, there cannot be five differences, or ways to measure those differences.

Stuart Burns

back

May asked:

'People who want philosophy ladled out to them can go elsewhere. There are philosophical soup shops at every corner..." (C.S. Peirce).

What does Pierce mean exactly with that statement?

You can never know exactly what people may mean when they use metaphors but a good guess is that he means that reading his book will take some thought and effort on your part, unlike his contemporaries whom he obviously thinks are bad philosophers.

Kim Boley

back

Betty asked:

WHO is your favorite out of the following and why: Heraclitus, Aristotle, Plato, Machiavelli, Francis Bacon, Descartes, Socrates?

Is the infinite a relative truth or an absolute truth?

Neither! The question is ill formed and therefore does not permit an answer. (Perhaps you might like to rephrase the question?)

On the right hand side, you have used the concept "truth". But "truth" is a property of propositions. On the left hand side, all you have is "the infinite". But "infinite" is a property of something — in this case an unspecified noun. Unless you actually mean here "The Infinite" — as in God or some such. In neither case have you specified a proposition of which "truth" can be applied. It is therefore quite impossible to specify which subdivision of "truth" might apply.

Stuart Burns

back

Robb asked:

Why does Sartre refer to boredom in his novel Nausea?

Jean Paul Sartre (1905-1980) was more than just a philosopher. You could say that he tried to express his ideas as clearly as he could, and for that he used several mediums, like plays and novels. If you miss this part of his work, and focus only in the pure philosophical works, you miss some insight into things that could not be clearly defined by theoretical concepts. Nausea is, first of all, a great novel, but a dormant novel... the leitmotif is just what the title says, that could maybe be more exactly translated as tedium. This is a subject matter that is found in the great writers of the XX century, mainly in those of the existentialist school of thought, like Sartre, Albert Camus, or Fernando Pessoa. The tedium, or boredom that Sartre mentions, reflects his view of man as an abandoned being, which sees his own life as mainly absurd and without a clear meaning. If life can't be analyzed, because it does not make sense, a particular attitude toward events in one's life is born. Existentialists are often portrait — in a blatantly unfair stereotype — as depressed middle age men dressed in black. Often they are not middle aged at all, but much younger.

In a nutshell you could summon up some things that Sartre established, that makes him have this tedium towards living: 1) Existence precedes Essence — this means that we built our own destiny; 2) Man is condemned to be free — this means that we can't escape the obligation to face up to our responsibility; 3) Man is the only one responsible for his own decisions — again, an emphasis on the personal choices and responsibility. We can see that no clear meaning can come from analyzing human existence, because each and every one of us has to built his or her own future. The fact that we are free, makes us doubt the consistence of reality around us, as we can red in Nausea. If in fact we build our own reality, made of our own moral laws, concepts of good and evil, with no a priori rules, all his allowed — the main theme of another great book that Sartre read: Brothers Karamazov by Dostoyevsky. If in truth we are ultimately forced to make choices, and take responsibility, an aura of fog surrounds what once was considered all that was real,

Nuno Hipolito

back

Kobby asked:

Is philosophy worth pursuing as an intellectual discipline in developing countries?

Well, I didn't really feel qualified to answer this one... so I asked an urban anthropologist who specializes in developing cultures. Her answers were: a) yes, in just the same sense that developing cultures should have and be exposed to art, they should have and be exposed to philosophy; b) yes, because indigenous cultures have philosophies that are valuable to them and could be to others. In addition, I also think that the answer is yes, because developing countries 1) need exposure to the rest of the world beyond their TV shows, and 2) it is possible, given that philosophy has utility, that Western philosophy, after several millennia, actually has something useful and interesting to say. For example: freedom is better than slavery; children shouldn't be abused; women are capable of making significant contributions to economic and cultural development. And so forth.

Steven Ravett Brown

back

Andy asked:

The question of the the universe expanding and contracting has puzzled me. What is the universe contracting and expanding in? My concept of the universe has always been the it is the whole and there is nothing beyond the universe. If that is so what is outside the universe that it expands and contracts in? (Answers 21).

To which Jürgen Lawrenz replied:

I've asked myself the same question often enough and never found an answer. I can't give you one either, for it puzzles me as much as you why scientists come up with such obviously incongruent notions, which leave the word 'universe' out on a limb as a meaningless concept. Alternatively, of course, you could look upon it as an embarrassment of our understanding: we want to know if the universe is all there 'is', but we can't know, and so we look at atoms and electrons and quarks and leptons and imagine that in their rhythm a mirror image of the rhythm of the universe is displayed. Sorry: this is no answer. But there is no answer, and therefore the whole question is null and void. Perhaps that's one good reason why we still need philosophy!?

With apologies to Jürgen, when he says "there is no answer", he is incorrect. Therefore, the matter is not "an embarrassment of our understanding". (Although, perhaps Jürgen means just an embarrassment of his understanding?)

The word "universe", is used to signify two related but quite different concepts. One is the familiar one that Andy refers to — the totality of matter and energy (and, by extension, space) in existence. Understanding Andy's question with this as the meaning of "universe", there is a relatively simple answer. By definition, there is nothing outside the universe, and nothing that the universe expands or contracts "in". It would be like asking what is before "A" or after "Z" in the alphabet. The question commits a "frame error" of adopting an inappropriate frame of reference. (Perhaps this is what Jürgen means when he says the question is "null and void"?)

To help you adopt the proper reference frame, perhaps a popular analogy is in order (which you have probably encountered before). Visualise the surface of a balloon. The two dimensional surface of the balloon is finite and unbounded. When you blow up the balloon, any two spots on the surface will get further apart. Even if a two-dimensional observer (a "flat-lander") could not measure the surface area of his space (because he can "see" such a small part of it), he would be able to deduce, from the increasing distance between any two spots, that the total area is expanding. As three-dimensional observers, we can see the two-dimensional surface area expanding in our three dimensional space. But that "outsider's" perspective is not a necessary requirement for the validity of the flat-lander's conclusion that his space is expanding. Our three dimensional space is expanding just like the two-dimensional surface of that balloon. And it makes no more sense for us to ask "expanding in what" than it did for the flat-lander to ask about the expanding area of his two-dimensional space. (Assuming that to a flat-lander, a third spatial dimension is as unvisualisable as a fourth spatial dimension is to us.)

There is another line of reasoning that arrives at the same answer. To the best of our current understanding, our universe is infinite and unbounded. For the universe to be unbounded means that there is no "edge" (boundary) for there to be anything beyond. For the universe to be infinite in extent means that there is no distance beyond which there could be something that is not part of our universe. If our universe is, in fact, either infinite or unbounded, then there is no place for there to be anything "outside" of our universe, and nothing in which our universe could be expanding. And the same arguments apply to hypothetical "higher dimensions". If the universe is, in fact, four (or "n") dimensional, then the extents within these additional dimensions are also already a part of our universe.

If you object that you cannot visualise how a three (or four or "n") dimensional space could curve or expand without invoking a further dimension in which this curvature or expansion could take place — don't be too concerned. We have evolved as three dimensional beings with a mental capacity tuned to deal with the normal slings and arrows of our daily existence. That the challenge of visualising multi-dimensional mathematics is beyond your capacity should be considered normal. That is what mathematics is for, after all. To help us understand and describe things of this Reality that are beyond our capacity to visualise. (Who, after all, can visualise -1 or a 4-dimensional hyper-cube?)

There is, as I mentioned, another meaning of the word "universe". It is a meaning that is properly reserved to certain branches of Cosmology and Theoretical Physics, and to certain kinds of fiction (science fiction and fantasy fiction). Although, as Jürgen was perhaps alluding to, it is a meaning that is well abused in popular writing. Within the proper restricted contexts, the word "universe" is used to refer to the totality of matter and energy (and, by extension, space) in the reality with which we are familiar. Notice the nature of the restriction. It is useful when one is speculating on the possible existence of "alternate realities" of various sorts. Cosmologists employ this meaning of the word when they are speculating about possible ways that "our" universe may have been spawned by an "earlier/other" universe. Theoretical Physicists employ this meaning of the word when they are speculating about the "real" meaning of Quantum Indeterminacy (the Multiple Worlds interpretation). In both of these manners of employment, "our" universe is considered to be but one of an infinite number of universes embedded in a "supra-universe" (one that employs the "in existence" meaning of the word universe). But I don't believe that in any of these manners of speaking, there is envisaged anything outside of "our" universe in which "our" universe would be expanding. So, as far as I know, even within this restricted meaning of the word universe, the answer to Andy's question is still the "nothing" that I described above.

Stuart Burns


A quick response to Stuart. I think that on the one hand he's taking too sophisticated a line with Andy's question, on the other giving too much credit to purely theoretical propositions. His balloon example simply reinforces the point I made: what does the balloon expand into? We need, as you can see, an enveloping dimension. Infinite regress hovers over it. That was the gist of Andy's question. All the other stuff is speculative, non-obligatory toying with concepts of which to say that "we" understand them is the grand overstatement of all time. What e.g. does "the totality of matter and energy" mean — isn't this a perfectly meaningless statement? I think the "ayes" have it. And I could not see any point in rolling up Everett's fancies or the 26-dimensional structures of loop quantum gravity or similar arcana to impress my questioner. I preferred to answer a straightforward question in the plainest manner, incidentally intimating that, despite a deluge of scientific work on them, sound philosophical rationale is still a conspicuous casualty among a great many cosmological theories. I'm sorry to say that after examining Stuart's explanations, I find nothing that serves to contradict this point or my original assertion.

Jürgen Lawrenz

back

Scott asked:

Today I am sitting in my high school library at 9:55 am. So far in my life I have listened and watched my friends and myself go about life. I have come to a point in which I have fallen into a rut. Although I am not the best writer or student, all I can think about are questions about society and questions about our world. I write today not to find an answer to all my questions but to attempt to find a peace of mind. My questions are as follows.

Why are we capitalists? Is it human nature to be greedy or have we been corrupted?

Next, why should we go through school and life trying to accomplish as much as we can? Does this make us better people to society? or does this make us the people we were intended to become by god or whatever created us?

Last, what is point of everything? Why do we wake up every morning? because I'm not buying into the "we have our family our friends and society to live for" thing. I know it will be very hard to answer some of these questions.

Your first question is a presumption to which many people would not subscribe. In fact, my own instincts do not lean towards capitalism. I find it appalling that capitalism seems to have got completely out of hand, and is running riot in the world. My greatest concern is that it has now become linked with democracy, a capitalist state is now automatically considered to be a democratic state. 'Free market' and 'free people' are not interchangeable concepts!

Is it human nature to be greedy? A good question to follow the one on capitalism, the two terms are often linked. Perhaps there is something in the notion that a capitalist can never have enough. Greed shows itself in theft and murder, in embezzlement and profiteering. However, most people do not fall into these categories. Received knowledge from the society in which a person is brought up will probably have a great deal to do with whether they recognise greed as an essential attribute to make progress and achieve a 'good life'. A capitalist society is geared to produce such a person. However, whether a person accepts this world view is very much a matter of choice. A person entertaining sets of categories of what is right and what is wrong is likely to place greed in the latter category, and speak against it; but does a capitalist society encourage the majority to see some moral right in the former?

Greedy people are usually 'thick skinned' and selfish, so what others think about them is of little interest to them. Of course, what a greedy person does to hurt or upset others also makes little impression on their conscience. Unfortunately, this has now reached its zenith in the destruction of the environment, the greed for oil profits, the greed for timber profits, the handing over of vital utilities to the greed of privatisation, the greed of the car companies, the greed of pharmaceutical companies, supermarkets, etc. etc.. Encouraging capitalism as a morally correct and democratic process is a major factor in the demise of our planet. We also have to ask ourselves what part political systems that turn a blind eye to all this play, or why in some cases they endorse it. In fact many politicians actually have their fingers in the pie.

You ask, why should we go through school and life trying to accomplish as much as we can? The short answer is the reciprocal question: Why not? Again, it rather depends on our world view and our aims within that world. Your further question regarding making us better people in society and, becoming what God or the creator intended, are extensions of the original question. If you firmly believe that there is a purpose in life and that purpose has religious connotations, then you are likely to be considerate of your fellow beings, and find a purpose which aims to improve the life of society. Your questions seem to point to a state of introversion, which leads to a rather self-orientated and pessimistic view of life. The extrovert view which is concerned with recognising having a place in society and being concerned with the well being of others, leads to a more optimistic view of the world.

Success in society can be viewed as self achievement, aiming to stand on the pedestal and to glory in the awards. Alternatively, it can be seen as something achieved for the whole of society, the achievement being its own reward. If we wish to believe that this latter objective was God's purpose for our existence, then that is a very valuable bonus to the achievement.

You ask, what is the point of everything? Why do we wake up in the morning? I see this again as a very self-centred and depressed approach to life. We should not sit tight under the shadow of pessimism wondering what the point of everything is. We should be out there finding out for ourselves what it is. I suggest we wake up every morning to do just that. My advise to a person who finds no meaning in life is to become a philosopher and share in the excitement of trying to discover what the world and what life is all about. We can either be depressed with our shallow view of the world, or we can be stimulated by seeking the deeper reasons for what we perceive around us. And be warned, the concepts we form in life constitute the world we live in.

John Brandon


Having reviewed the answer that John Brandon provided to your questions, I am driven to provide an alternative view of the situation.

According to http://www.encyclopedia.com/html/c1/capitali.asp "capitalism" is "an economic system based on private ownership of the means of production, in which personal profit can be acquired through investment of capital and employment of labor." And according to the Encyclopaedia Britannica, "capitalism" "also called free market economy , or free enterprise economy — [is an] economic system, ... in which most of the means of production are privately owned and production is guided and income distributed largely through the operation of markets.

In order for a Capitalist economic system to function, the political environment within which the economic system exists must sustain two features. It must provide some minimal support to the legal principles of private property, and it must provide some minimal level of individual freedom. There cannot be private investment in the means of production unless there can be private ownership of the stuff being invested. There cannot be a functioning market to guide productive investment and income distribution unless there is a minimal amount of individual freedom to participate in markets at will.

So John's "greatest concern ... that [capitalism] has now become linked with democracy" is seriously misplaced. Contrary to his suggestion that "a capitalist state is now automatically considered to be a democratic state", the reverse is more properly correct. Any form of government that grants its citizens both a right to privately own property and a modicum of individual liberty will inevitably come to support a capitalist economy. An example of this inevitability can be seen currently in China. Although certainly not a democratic state, China is quickly becoming a capitalist economy. It is inevitable that a modern liberal democracy (one that, by definition, provides legal support for both private property and individual freedoms) will be a capitalist economy. "Free market" and "free people" might not be interchangeable concepts. But "free market" presupposes "free people". And "free people" includes within it "free markets". Given a modicum of legal support for private property, "free people" thus implies "capitalism". It is obvious, however, from the tone of your question and the nature of John's answer, that neither of you look kindly on the concept of private property.

So why are we capitalists? We are capitalists because we live in a liberal democracy where the vast majority of the people have discovered that private property and the workings of free markets are the most effective and efficient means of achieving the greatest economic welfare for the greatest number. History has demonstrated that when it comes to economic measures of well being, nothing is as successful as capitalism at delivering the Utilitarian ideal of "the greatest good for the greatest number".

Your second question is a very emotionally loaded one "Is it human nature to be greedy or have we been corrupted?" The very formulation of your question both presumes more than many would accept, and presupposes the answer. According to Microsoft's American Heritage Dictionary of the English Language, "greed" is "an excessive desire to acquire or possess more than what one needs or deserves, especially with respect to material wealth". Your question therefore presumes that most people's nature is to display an excessive desire to possess more than one needs. Your further suggestion that we do so because we have been corrupted suggests that you feel that not only do most people desire excessively more than they deserve, they do so because they are damaged goods.

I dispute both of your presumptions. Most people are not greedy. Nor is it human nature to be greedy. (Which is not to say, of course, that there are not greedy people about.) It is human nature for people to want more than they have. It is human nature to strive to improve their lot in life. It is human nature for people who live in a capitalist economic system to strive to earn more than they have. It is human nature for people who live in a social environment to seek to circumvent the rules established by others. It is human nature to co-operate with those they can co-operate with, and compete with the rest. This does not make human nature greedy. There is nothing about the natural human desire for more than one has that is excessive. And as long as the capitalist economic system functions properly, people will not receive more than they deserve.

The answers to the rest of your questions I think are best provided by addressing your question about "the point of everything". The answer I am about to provide you will not be met with agreement by many (I am sure John, for one, would strenuously disagree). It does, however, have the advantage of being consistent with all that we currently know about biology, evolution, and psychology.

Life is Action. "Life" is characterized by the unique fact that living things change and move — "act" — through the directed application of internally collected, stored, converted, and channeled energy.

Life's Actions are Teleological (Goal Oriented). At a very fundamental level, the goal of all living behaviour is the maintenance of the life that is behaving.

The Gene is the Unit of Life. It is that (not necessarily contiguous) stretch of the DNA (or RNA) molecule that can be labelled as a Gene that is what must be recognized as the entity that survives and proliferates — continuation of which is the goal of Life's Actions.

The Reproductive Imperative. The actually observed behaviour of all living creatures, both in general and individually, is highly flexible and variable but within the broad genetically defined limits of continued genetic survival.

Our Purpose. As an example of life, as an example of the species Homo sapiens, and as an individual consciousness, our purpose is to ensure the continued survival and proliferation of our genes.

The Definition of Good. To be "Good" at anything is to do a quality job at fulfilling the purpose of that thing. A good Human Being is efficient and effective, and fulfils with quality, the purpose for which the Human Being was built -- to ensure the continued survival and proliferation of our genes.

Better is Never Enough. To ensure the continued survival and proliferation of our genes is a never ending struggle. There is never enough assurance that the job is complete. There is always something extra that can be done, some marginal increase of assurance that can be found.

And Finally — Best?. The struggle continues whether or not the individual is consciously aware of why they are striving, or what they are striving for. Even if they are striving under misconceptions, misinformation, or mistaken assumptions, the human animal is built to strive. The best situation is to be consciously aware of why you are striving, and employ the best of your intellectual abilities to make conscious rational choices of what to strive for. Happiness comes from knowing you are doing a good job.

So — "the point of it all" is the welfare of your genetic descendants (over the long run, of course). Go to school because it is the best means available to you at this time, and in this place, to prepare you to do well by your children. You are not here to be good for society. You are not here to become whatever God might have intended. You wake up every morning and tackle the day because you have a function to perform. Friends, family, and society matter only to the extent that they can contribute to your ultimate purpose in life.

Finally, I fully agree with John's concluding remarks — "My advise to a person who finds no meaning in life is to become a philosopher and share in the excitement of trying to discover what the world and what life is all about. We can either be depressed with our shallow view of the world, or we can be stimulated by seeking the deeper reasons for what we perceive around us. And be warned, the concepts we form in life constitute the world we live in." Amen!

Stuart Burns

back

Jeffrey asked:

Do people recover from states of profound existential angst (i.e. constant overwhelming questioning, confusion, estrangement, anxiety, disillusioned sense of self etc.) or do they have to learn to live with it? are there any books on the subject?

In answer to your first question: both are true. If what you are terming "angst" is leading to problems in your life, dealing with people, with a job, etc., and it's been going on for some time, then you are likely suffering from clinical depression, and you need serious professional help, i.e., therapy. Alternatively, you might see a philosophical therapist... yes there are such people; just be sure to find one certified by one of the professional associations. If you don't want to go the chemical route (which in many cases is not a bad idea for the short term), i.e., take antidepressants as part of therapy, there are all sorts of books on the subject, in the "self-help" shelves of your local bookstore. On the whole, I don't recommend them; if the above is true, I'd say go for therapy.

Good cures for mild to moderate (for severe, see above) angst: exercise, exercise, exercise. A very good one. Next, meditation, i.e., some form of Buddhist meditative practice. Very good so long as you can avoid buying into the metaphysics... in my opinion. For this latter, either or both of reading and lessons in a recognized and legitimate school are the way to go. The next and perhaps best cure: get into a good intimate relationship; yes, just go do it, take a few chances (emotional... I don't mean physical).

Well, haha, what wonderful philosophical advice all this is, right? I'm afraid as far as philosophy goes, angst is the name of the game; even Buddhism's ultimate motivations are pretty depressing, when you get right down to them (but they have a pretty good cure for angst, if, as I say, it hasn't gone too far). If you want to pursue philosophy and you have angst problems, I'd advise a long vacation from it, with wine and song or the equivalents, followed by rethinking just what you're about in doing it.

Steven Ravett Brown

back

Harriet asked:

Why did emotivism come into existence?

And which philosophers deal with it and what points do they make?

Emotivism as an ethical theory (better a meta-ethical) theory is very much a creature of the 1930s and 40s. A version was espoused by the young Ayer in Language Truth and Logic though the fullest expression of the theory is to be found in the work of C.L. Stevenson. His classic text is Ethics and Language. There is a related set of collected articles by Stevenson entitled Facts and Values published in the early 1960s elaborating and refining some of the key themes associated with emotivism. Emotivism can be seen as a reaction against the ethical intuitionism of the early 20th century associated with G.E. Moore, and later, Ross and Prichard.

Justifiably or not, it was seen as leaving too much as too mysterious. The dogma that one cannot derive an ought from an is had been given compelling expression by Moore and formed the backcloth against which all subsequent reflections on the nature of ethics had taken place. Emotivists while accepting the naturalistic fallacy nevertheless recognised that moral discourse was directed — importantly, if not entirely — to influencing the actions and behaviour of others. They looked to the dynamic nature of moral discourse as holding the key to how it shaped and influenced others' actions and behaviours. Roughly to say X is good is to say 'I approve of X. Do so as well' This pattern of analysis did justice (so it was claimed) to both the descriptive and evaluative dimensions of moral language while at the same time avoiding any tendency to suppose the evaluative dimension is derived from the descriptive element informing others about attitudes towards X.. Ethical discourse in other words worked through the exciting of shared attitudes. Hare's later prescriptivism can be seen as a sophisticated variant upon emotivism.

A critique of this whole programme is beyond the remit of this answer but fairly clearly the plausibility of this approach turns upon 1. being prepared to accept that there is one peculiar function (the shaping of attitudes towards actions etc) that distinguished moral discourse from other kinds of discourse and 2. that our natural belief rational debate can be had on moral matters is quite misconceived as moral debate is really about persuading individuals no matter how to come to share your attitude towards a certain action or state of affairs. The merest perusal of the uses to which moral discourse is put will show the implausibility of the former suggestion. And we are reluctant to let go of the distinction between considerations that reason should recognise and those that merely persuade us, irrespective of their rationality . The analysis proffered by emotivists of the nature of moral discourse seems to do too much violence to the nature of that discourse.

Ian Gregory

back

Winta asked:

What does a mathematical proof prove?

Yes, indeed, very good. But here's one for you: what does "prove" mean? Establish beyond reasonable doubt? Establish with absolute certainty? Enable us to comprehend with certainty? Lets see... if we take a formal system, where terms are, we believe, "well-defined", then the postulates, elements, and operators in that system ("the basics", let us say), once they have been defined, determine the entirety of the system, within the limits established by Godel's Proof (given that our definitions are not contradictory, of course).

Thus, a proof in such a system does not create anything, but enables us to discover truths about that system, and further, enables us to ascertain that it is true that this discovery is implied by the basics, because we understand, by the fact that the proof takes the logic of that implication from start to conclusion by its smallest steps and thus that we can check each step for accuracy according to the most elementary rules of logic, that it is correct. So here, "prove" means a) establish with absolute certainty, b) establish beyond reasonable doubt (implied by a), but not necessarily c), because there are proofs which are so long and complex that humans cannot comprehend them even when they are reduced to their elementary steps. For example, Wiles' proof of Fermat's Last Theorem was that complex (except for him and a couple of others, I believe), and so was the proof of the Four-Color Theorem in topology. Computers were used to check and in the latter case, as I recall, to generate parts (through iterative map drawing) of those proofs. And I'm sure there have been others.

But all this assumes that we know what the basics mean. But do we? What is a point, for example? What does "union" mean, really? And so forth. If you read some of the naturalistic philosophers on the psychological derivations of mathematics, you find that the basics are, according to them, derived from abstractions of our experiences (see, e.g., Lakoff, G., and R.E. Nunez. Where Mathematics Comes From: How the Embodied Mind Brings Mathematics into Being. New York, NY: Basic Books, 2000). Most mathematicians and indeed most philosophers recoil in horror at the thought that their and similar arguments may be correct... but these viewpoints (which by the way I am sympathetic to) must, in my opinion, be taken seriously. It does remain that we use mathematics, quite well, to describe and manipulate the world... but we do that with our hands and with natural languages also, if less precisely. And it is that precision which perhaps distinguishes, and certainly makes invaluable, that particular way of symbolizing aspects of our comprehension of the world. However, if the naturalistic arguments are true, then we must indeed ask what a mathematical proof proves. Something, perhaps, about the way humans comprehend the world, and that's about it.

Steven Ravett Brown

back

Kim asked:

How would Rawls answer the question "what makes a society just?" Why is the veil of ignorance important in developing a contractarian theory of justice?

Rawls says that a just society should have a "basic structure" determined by principles that would be agreed by people in a hypothetical situation that he calls the "original position". The original position is a version of a contract procedure, and its purpose is simply to produce a fair procedure that doesn't build in existing patterns of privilege and advantage.

The "veil of ignorance" is a set of restrictions that prevent the contractors from devising principles and procedures that suit themselves. It guarantees impartiality. The people are conceived as choosing from a position of ignorance about their own talents and position in society, and also ignorance about their own conception of the good, their own views on what gives value to a life (including their religion and tastes).

Rawls maintains that the parties will come up with principles to govern the basic structure of society:

1. Each person has an equal right to the most extensive scheme of basic liberties which is compatible with a similar scheme of liberty for all.

2a. The Opportunity Principle: Social and economic inequalities must be attached to offices and positions open to all under conditions of fair equality of opportunity.

2b. The Difference Principle: Inequalities must be to the greatest benefit of the least advantaged members of society.

Rawls' theory has been much criticised but also much defended, including by himself. One area of doubt is whether he assumes liberal principles in seeking to establish them — impartiality, for example — and another is whether he assumes a particular conception of individuality, a Western. liberal conception of personhood. But Rawls always knew his theory of justice was to be tested against the intuitions of people living in liberal democracies.

Graham Nutbrown


Not all contractarians use the veil of ignorance — Hobbes, Locke, Kant and others (more recently Narveson, Gauthier) do not. That is a Rawlsean innovation, mainly so as to make room for what Rawls deemed to be very important to justice, namely, fairness. Presumably behind the veil of ignorance everyone lacks knowledge about himself or herself (indeed, whether they are male, female, tall, short, black, white, old, young — anything individual or particular) and so one can be impartial in figuring out what it is to be just. And presumably a basic intuition all of us have, regardless of who we are, would guide us to, among other things, stress fairness in a just society.

Tibor Machan

back

Andre, Oscar, Jair, Alfred, and Bobby asked:

B.F. Skinner purports that humans truly do not influence their environment, but are rather more influenced by it. Briefly, his philosophy holds that both human nature and an individual's personality can be explained in terms of their behavior and that this behavior is simply a result of environmental conditioning. That is, our nature as humans is nothing more than learned behavior. Accordingly, the environment — and not free will — has shown itself to be the cause of human behavior, which suggests that if we humans are to go on to greater heights, then we must reject free will and its corresponding concepts of freedom and dignity in their traditional senses.

Would you explain Skinner's theory of behaviorism more in depth? Would you say you agree or disagree with this conception of human behavior and criticism of free will?

In 1967 Arthur Koestler published his book The Ghost in the Machine, of which the last chapter is entitled, "On Flogging Dead Horses". The chapter is about Skinnerism, about the claims for its demise and Koestler's disbelief of this supposed fact, which motivated him to flog this "dead horse" yet again. So we are talking about the tail end of a dispute datable to about 35 years ago.

I mention this because your question indicates that Koestler was right in his opinion. Why would anyone be bothered digging up this morose and illogical doctrine yet again? To make people think they're little pigs, or dogs, or rats in human disguise? I recommend you go out and enjoy the sunshine and write a poem about it. If you can do that you will summarily disprove all of Skinners theories in one blow. But even if you can't, read Keats or Donne; or if your taste is not for poetry, how about the wonderful intricacies Ramanujan's equations? And then sit down again and give me a neat, but watertight proof of how "environmental conditioning" caused these artefacts to be spun by a human mind.

Jürgen Lawrenz


Well, look... I hate to throw cold water on all this... but no one takes Skinner very seriously any more in psychology, except some very very hardcore physiologists concerned with drug addiction as it relates to reinforcement schedules (and probably people researching gambling also). As far as philosophy goes, Skinner per se has really never been taken seriously for any length of time. Go look at this: Chomsky, N. "Review of Skinner's Verbal Behavior." In Readings in the Psychology of Language, edited by L.A. Jakobovits and M.S. Miron, 142-71. Englewood Cliffs, NJ: Prentice-Hall, 1967, for an excellent critique of Skinner.

Here's the situation, as I see it (and I'm not alone). We have a dual set of motivators, in a sense. On one very low level, we're motivated by reinforcement, as described by Skinner, Hull, etc. On another level, we're motivated by cognitive feedback effects. Those two systems are constantly interacting with one another, producing all sorts of effects of various types, from conflict to synergistic interactions.

The question of free will, as it relates to the brain, is much broader than implied in Skinner. If you want a contemporary take on the neurology, etc., of it, look at these:

Brown, J.W. "The Nature of Voluntary Action." Brain and Cognition 10, no. 1 (1989): 105-20.

Haggard, P., S. Clark, and J. Kalogeras. Voluntary Action and Conscious Awareness [WWW]. Nature Publishing Group, 2002 [cited March 2002]. Available from http://neurosci.nature.com.

Libet, B. "The Timing of Mental Events: Libet's Experimental Findings and Their Implications." Consciousness and Cognition 11 (2002): 291-99.

Libet, B. "Unconscious Cerebral Initiative and the Role of Conscious Will in Voluntary Action." The Behavioral and Brain Sciences 8 (1985): 529-66.

Libet, B., A. Freeman, and K. Sutherland, eds. The Volitional Brain: Towards a Neuroscience of Free Will. Edited by J.A. Goguen. Thorverton, UK: Imprint Academic, 1999.

Searle, J. R. "Consciousness, Free Action and the Brain." Journal of Consciousness Studies 7, no. 10 (2000): 3-22.

Searle, J. R. "Free Will as a Problem in Neurobiology." Philosophy 76 (2001): 491-514.

Wheeler, R.H. "Theories of the Will and Kinesthetic Sensations." Psychological Review 27 (1920): 351-60.

Beauregard, M., J. Levesque, and P. Bourgouin. "Neural Correlates of Conscious Self-Regulation of Emotion." The Journal of Neuroscience 21, no. RC165 (2001): 1-6.

Haggard, P., and M. Eimer. "On the Relation between Brain Potentials and the Awareness of Voluntary Movements." Experimental Brain Research 126 (1999): 128-33.

Logan, G. D., C. Taylor, and J.L. Etherton. "Attention and Automaticity: Toward a Theoretical Integration." Psychological Research 62 (1999): 165-81.

Wegner, D. M. The Illusion of Conscious Will. Cambridge, MA: The MIT Press, 2002.

Wegner, D. M., and J. A. Bargh. "Control and Automaticity in Social Life." edited by D. Gilbert, S. T. Fiske and G. Landzey. Boston, MA: McGraw-Hill, 1996.

Those should give you something to think about for a while.

Steven Ravett Brown

back

Rosie asked:

Can you explain in layman's term, interactionism and materialism. What are the strengths and weaknesses of each theory?

Materialism and Dualism are two competing theories in the philosophy of mind. Essentially, dualists believe that there are two distinct kinds of "stuff": physical and mental/ spiritual. By contrast, materialists think that the physical is all there is. So in terms of philosophy of mind, dualists think that the mind is not the same thing as the physical brain, but is something more/ beyond/ above, while materialists think that all there is in the mental realm is really physical.

Interactionism is a subcategory of dualism. All dualists think that the mental is separate from the physical. Interactionist dualists think that the mental is separate from the physical, and also that the two realms causally interact. So, for example, I might punch you in the face (physical) because I want to (mental). And if I did punch you in the face (physical) it might result in you feeling pain (mental).

Strength: Interaction is obviously a pretty common-sense view — most people think that mental things cause physical things and vice versa. Weakness: It's very difficult to explain how mental events could cause physical events and vice versa. If they exist in different realms, how are they connected? The greatest weakness with interactionism is that it's so mysterious — no one's ever given a very good explanation of how or why it works.

Interactionist dualism contrasts chiefly with two other forms of dualism: epiphenomenalism and parallelism. Very briefly, epiphenomenalism holds that mental states are caused by physical states, but not the other way 'round. So your pain is caused by my hitting you, but it's just an illusion that I hit you because I wanted to — actually, I hit you because certain physical neurons fired in my brain, causing my muscles to contract in a particular way.

Parallelism holds that there are no causal relationships between the mental and physical realms, but that our mental life just happens to coincide with our physical life (or is controlled by God to coincide).

For more, check out http://plato.stanford.edu/entries/dualism/.

Jonathan Ichikawa

back

Nwaopara asked:

Referencing to actuality and potentiality, which one takes precedence over the other or which one is more important than the other; the hen or the egg?

I'm hoping that you're asking a bit more general question than one about chickens... here's my very rough take on this. There's a huge amount written on "potentiality", and in my view most of it is pure speculative metaphysics, not worth much. The only place where potentiality, if you want to term it that, is treated seriously and reasonably, is, in my opinion, in statistics, particularly in Bayesian statistics. Now, given the point of view that one can use data from the past and present to predict the future, i.e., the statistical viewpoint, what do we say about potentiality vs. actuality? Clearly, since statistics, and indeed prediction in general, deals with less than certain events, and is based on data regarded as more certain (i.e., than future "possibilities") taken from the past and present, we must say, from that viewpoint, that actuality takes precedence over potentiality. Which is the chicken and which the egg I have no idea; they both seem quite actual to me.

Steven Ravett Brown

back

Kayla asked:

Please tell me why the law should have anything to do with relationships. I am 15 going on 16 this month and I want to understand why it is illegal for me to be with a 19 year old, we're not having sex but we like each other ALOT! I find it torpid to why the law should have anything to do with us and our relationship if our relationship is fine, content and we're happy with each other! it just doesn't make any sense at all to me why they should have a problem with it! i need answers and I want them soon because it tearing our relationship up when its completely normal for us and other people have to get in on it and try to make it fall apart because of the stupid law! i thought this was America "freedom" I'll find more so called "freedom" in some other country that don't have anything to do with two people in a relationship who are very happy with one another!

I couldn't agree with you more. It is ridiculous not to be able have sex until you are 16. The law is supposedly protecting "girls" under sixteen. Furthermore, if you do, you will not be held legally responsible but the man will. This assumption that you need protection and are not responsible for your actions is amazing.

You wouldn't be better off in England. Unless the law has changed recently, under English law a man is not guilty of a sexual offence when having intercourse with a girl under 16 if he has reasonable cause for belief that she is 16 and the offender is "under 24"!! How arbitrary can this be?

Since girls are now menstruating at a much earlier age the law is becoming even more ludicrous.

Surely it isn't illegal for you to just go out with someone?? Are you sure?

Rachel Browne

back

Violet asked:

Do you think that conscription is morally right? I started out thinking that prostitution is not morally wrong as long as the parties involved made a free choice to do so i.e. no gun to the head, no emotional/ financial blackmail etc. Then I realised that there are things which people are forced to do which are in fact for the 'good' of the greater population — such as conscription. Surely if an individual believes fighting for one's country is right, then he will make the choice to join up anyway.

It is easy to imagine a situation where not enough young people are willing to fight for their country. They have, perhaps, come to take security too much for granted, they are too self-interested, or they are moral relativists who cannot summon the intellectual motivation to fight against a different culture, or they are cynical about Government motives, or, as a generation, they disapprove of war. Whatever, the reasons, a country whose young people will not fight is obviously vulnerable. (Possibly, the young people would make a distinction between defending their own country from attack and invading another country.)

I live in the UK where there is no compulsory military service. It is simply not something that British people of, say, 18 to 25, consider. I'm sure the majority would be appalled by any suggestion that they should do military service. It would probably be seen as a regressive and right-wing move. And yet there is clearly a need here to channel and express patriotic feelings. We've seen that recently with the overwhelming response to the England Rugby team's victory in the World Cup.

I think compulsory military service could be positive. Firstly, because it could mean that the country is better prepared for a defensive war. Secondly, because it would spread the burden throughout the social classes in a more equitable way. Thirdly, it could provide additional training, including self-defence training, to all young people, of both sexes. Anyone reading this from a country that has military service may have a different view.

But the question was about conscription. If there was compulsory military service then there's a chance that there would be more volunteers when war is declared, I guess. In the end, I can't see that a country has any choice: if necessary it has to call upon the population to fight. However, it is not easy to imagine a modern, technologically sophisticated invasive war in which this would be practical.

Practically, the only use of either military service or conscription that I can envisage is in civil defence. When it comes down to it, a country has to defend itself or surrender, and assuming that the Government is legitimate and democratic, and that the invaders are ill-intentioned and aggressive, conscripting people who are able-bodied to contribute to civil defence is surely acceptable. Of course, this means temporarily overriding certain liberal principles for the sake of defending them in the long term.

Graham Nutbrown

back

Alexandra asked:

'Our technologies establish the truth of many of our scientific laws.' Is there any comparable means of establishing moral rules and norms?

Yes but first a controversy needs to be treated, namely, what's the point of morality (we tend to accept that being useful is at least one basic point of science)? If it is to enhance living a human life, as some like Aristotle thought, then the test is whether a set of moral principles lead one to be happy (not just to feel good but to see oneself as a success at living a human life); if preparing for the after life is the basic point of morality, then I do not believe there is a knowable test for it (which accounts for why there are so many different religions, I think).

Tibor Machan

back

Arthur asked:

1. If it makes sense to say that the earth is not flat, then it also makes sense to say that it is flat.

2. If it makes sense to say that the earth is flat, then it is possible that the earth is flat.

3. If it is possible that the earth is flat, then it is not certain that the earth is not flat.

4. Therefore, if it makes sense to say that the earth is not flat, then it is not certain that the earth is not flat.

Does this prove that it is doubtful whether the earth is round?

And it also proves that it is doubtful that the moon is not made of green cheese.

Steven Ravett Brown


No. In my view the argument depends upon an ambiguous use of 'possible', without which it would be clearly implausible to move from 2 to 3. There are different senses of 'possible'. The earth being flat is something that is 'possible' in a sense that does nothing to compromise our certainty that it is, in fact, not flat.

Should we accept the claim contained in 2 that somethings being intelligibly statable ('makes sense to say') depends upon it being 'possible'? Well, we might, if we were given an argument for it. It would be for Arthur to explain what he is thinking of here, and without such an explanation one might be entitled to challenge his train of thought by simply rejecting 2 as baseless.

But we might try to imagine what Arthur is thinking of or justify his thought for him. The only plausible argument I can think of to put into Arthur's mouth just now is that if something is logically impossible, then it cannot be intelligibly stated. For example, it might be argued that it does not 'make sense to say' the words 'the square circle' because a square circle is logically impossible. That is, it is not just for reasons of incompetence that we are unable to make square circles, but rather because of what 'square' and 'circle' mean, that the phrase 'the square circle' is senseless.

Now if we were to take this as the argument making the deduction in 2 attractive to Arthur, then it would be clear that in 2, when he speaks of 'possible', what Arthur actually means is 'logically possible'. And this has bad consequences for the plausibility of the move to 3. For the fact that the earth being flat is logically possible does not do anything to undermine our certainty that the earth is in fact not flat.

Let us see that in detail.

If we were to say that it is 'possible that the earth is not flat' in the sense that it might be expected as one possible of outcome to some current enquiry into the matter, then this would indeed amount to a doubt of some degree about whether the earth is flat.

But when we say that the earth being flat is 'logically possible' we say nothing to suggest that there is a current enquiry about the matter, and therefore nothing to suggest that there is a doubt. We merely report that there would be no contradiction in terms involved in entertaining the idea should we be moved to do so. Whether the idea is in fact entertained is a separate and additional question.

Thus, that 'it is [logically] possible that the earth is flat' does not establish what 3 pretends it establishes. It does not show that there is any doubt or uncertainty.

David Robjant

back

Yuventius asked:

What is the meaning of "meaning"?

Haha... I'm only going to touch this one with the proverbial 10-foot pole. Take a look at: Ogden, C. K., and I. A. Richards. The Meaning of Meaning. 8th ed. New York, NY: Harcourt, Brace & World, Inc., 1968. That will get you off to a reasonable start on 3000 years of debate, anyway.

Steven Ravett Brown


All sorts of things can have meaning. Humans invest objects, gestures and actions with meanings, so "meaning" doesn't just refer to linguistic meaning. A handshake (in some cultures) can "mean" a greeting, a farewell, a reconciliation, an agreement, and so on. Clearly a number of other concepts are relevant: understanding, explanation, belief, convention, appropriateness, context, etc.

But philosophers have concentrated on linguistic meaning, and especially on the meaning of declaratory (or propositional) sentences. They have tended to associate the meaning of a statement with its truth conditions. So, the meaning of "The pencil is red" is its reference to the fact that the pencil is red. Whether or not this is an adequate account of declaratory sentences, it is certainly not adequate for sentences that do not express propositions that are to be taken literally. We need an account, for example, of irony and metaphor. We also have the problem of understanding how the meaning of individual words and the meaning of sentences that incorporate those words are related. Which is prior?

An important distinction is between the "semantic meaning" of the sentence and the "speaker's meaning". If you ask a child what colour that pencil is, not knowing that she is colour blind, and she thinks you are pointing to a green pencil and answers that is it red, she may have answered your question correctly, accidentally. The semantic meaning and her own meaning have divided. If we say that the meaning is first and foremost what the speaker has in her head, then we will be entering a maze of questions about internalism (such as those raised by Wittgenstein). If we say that meaning is first and foremost whatever the external truth conditions are, regardless of the speaker's beliefs or intentions, we are severing the link between meaning and mind. The consequences of that need thinking through.

Graham Nutbrown

back

Paul asked:

Regarding the human senses, although not a tactile sensation, might the sixth be considered thought?

Well, actually, you can think of the 6th as balance. Think about it: what do we feel when we get physically unbalanced?

Steven Ravett Brown

back

Alex asked:

What's wrong with lying? What would a Kantian and a Utilitarian say?

KANTIAN: Kant's categorical imperative states in effect that we're only morally permitted to do things that we could reasonably will everyone to do. So it's immoral to lie, because if everybody always lied, then communication would become impossible and society would collapse. So for Kant, lying is always wrong, because it violates the categorical imperative.

UTILITARIAN: For a utilitarian, an action is wrong if it has bad (technically non-best) consequences. Lying tends to cause bad things and unhappiness — people are disappointed when promises aren't kept, people get angry when they discover they've been lied to, people feel ashamed about having lied, people count on others' word, and suffer from them not following through, etc. For a utilitarian, it's usually wrong to lie, because usually, lying has bad consequences (and it's very difficult to recognize the situations where it won't). In some extreme cases, though, a utilitarian will encourage lying. If, for example, a gun-wielding maniac who is trying to murder your children asks you where they are, you should lie to him. (Kant famously thought you should tell the truth even then.)

Jonathan Ichikawa

back

Kristyn asked:

I apologize for lack of professionalism of this, however I am stumped. I am trying to use philosophy to look at the art of ceramics. I know there are philosophers and teachers who see ceramics as very aesthetic, but I do not know where to start! Can I question it as a spiritual experience, or attachment to the piece? I am a ceramics major, but it is easier to do what I do and feel what I feel, than it is for me to explain it.

Well, there is a huge amount written on aesthetics in philosophy; it's a field in itself. But if you want something a bit different, try these:

Herrigel, E. 1971. Zen in the art of archery. Translated by R. F. C. Hull. New York, NY: Vintage Books. Original edition, 1953.

Okakura, K. 1964. The book of tea. New York, NY: Dover Publications, Inc. Ross, N.W., ed. 1960.

The way of Zen: an East-West anthology. New York: Random House. 1977.

Tanizaki, J. In praise of shadows. Translated by T. J. Harper. New Haven, CT: Leete's Island Books, Inc.

Steven Ravett Brown

back

Chloe asked:

When listening to the radio one day i heard a discussion about fate and chance. The scenario was that a bald man went for a walk and was killed when an bird, who was carrying a turtle, mistook his head for a rock (on which he could crack the turtles shell), dropped the turtle, and consequently killed the man.

I am unsure as to whether this is fate for the man to die this way, or chance. At first I thought it was chance that he so happened to be on a walk when the eagle flew over and deliberately dropped the turtle, but friends suggested it was fate — this man was always going to die this way. Now I think that maybe it is a result of fate (maybe this man went for this walk because of a hard day at work, it was hard because he had an academically demanding job, he had this job because he worked hard at school in this area etc) with a serious of chances (e.g. a car accident involving others that blocked his normal walking route and caused him to take this route instead). I am still unsure. What do you think?

To believe that it was fate that brought about this man's death you would have to believe that there is a purposeful force in the universe that makes that has predetermined things to happen for particular reasons. This force could be God, or it could be whatever force it is that believers in astrology believe in. The point is that for believers in fate, events are not random or accidental; they are planned and/ or they fit into a wider scheme. There is an intelligence behind them, which could be either benign or malevolent.

To believe his death was a chance event, you need only believe in ordinary scientific determinism. The causes stretch back and back, and in theory his death could have been predicted if only we knew every relevant fact at a certain time; but the point is we never do, so events of this kind strike as as being accidental or chance events. You could build some purposefulness into scientific determinism. For example, you could say the chain of causes was started by an omniscient intelligence with a particular purpose in mind, which is what deists believe. This is a kind of fatalism. But if you think determinism operates blindly and that the universe is a bleakly purposeless place, that there is no telos built into the scheme of things, then I would say you believe in chance rather than fate.

An alternative version of chance would be to believe that things happen randomly, either spontaneously or as a result of an intelligence that causes them to happen without any order or purpose, like a typist hitting the keys randomly.

An alternative belief is signified by the phrase "character is fate": what happens to you is determined by your character and you cannot escape from its influence. Again, you could believe your character is God-given or that it is chance in the sense that it is caused by nature, your genetic inheritance, for example. Some kind of consistency and order is implied by the concept of character, so if your actions are caused by your character they can't really be said to be random or accidental.

One thing that makes Shakespeare such a great writer is that he was able to explore all these possibilities in his plays, most notably, perhaps, in King Lear, the conclusion of which, in my view, is that the universe is bleak and purposeless, however much we like to fool ourselves that it is ruled by Fortune or the gods.

My own view is that the man's death was a chance event. It was not fated to happen by an Intelligence or by his own character (although, his character contributed to him being in that exact place at that time), but nor was it a spontaneous or random event. It could, possibly, in theory have been predicted — but note that to accept that idea you have to be happy with the implication that both the eagle's and the man's behaviour was utterly mechanistic.

Graham Nutbrown

back

Todd asked:

I contend that a towel thrown into a pool is not wet until it is taken out of the pool.

Is it dry?

Steven Ravett Brown


I assume your contention that the towel is "not wet" means that it is dry therefore I ask you to prove this by drying yourself with said towel. If you cannot do this, which I believe you cannot, then I'll stay with the assumption that an object immersed in water is "wet" whether or not the primary definition of "wet" means a wet object in a dry environment, as your contention correctly identifies.

Kim Boley


It's the water that is wet, not the towel.

Brian Tee


I agree with Todd that, consulting ordinary usage, one does not say that a towel is 'wet' while it is immersed in water. In this respect, the state of being wet might be compared with the state of being dusty. You wouldn't say that a book was dusty if it was completely buried in a large mound of dust. It is dusty once it emerges from the dust.

What Todd contends would not be true of, say, a sponge. A wet sponge is wet either in or out of the water. So what makes the difference?

'Wet', like 'large' or 'heavy' is an attributive adjective. A large mouse is still small compared to other animals. Similarly, a towel only needs to be damp in order to be described as 'wet'. A towel which is dripping with water we do not describe as 'wet' but as 'soaked'.

However, ordinary usage quickly runs into problems. If, in answer to Steven's question, Todd agrees that the towel immersed in water is neither wet nor dry but soaked this would be considered a very strange thing to say. Most people would agree to the general proposition that if an object — any object — is soaked, then it is wet.

I think it is a mildly interesting question for the philosophy of language how one responds to this challenge. Is ordinary usage just fine as it stands, with all its apparent contradictions? Does the fact that we don't have a consistent way to describe the situation with the words we would normally use require the intervention of philosophers to legislate on usage? Personally, I don't think so.

Geoffrey Klempner

back

Corey asked:

What is the support for light being a particle? Also what is the support for light being a wave? What certain properties does light contain to fit into both these categories?

Well, you know, this one has been batted around for decades now... you really should go read someone like Asimov on this. However. To answer your question, sort of: light is neither of those things. It sometimes behaves as if it's a particle, and sometimes behaves as if it's a wave. Go look up the "double-slit diffraction" experiments on this. Basically, the issue of "what" light is has not been resolved satisfactorily, despite the fact that the theory of quantum electrodynamics (QED) is probably the most accurate ever devised. You might look here: http://physics.hallym.ac.kr/education/mbody/mbody2.html; or here: http://www.aip.org/history/heisenberg/p08r.htm for some more information.

Steven Ravett Brown

back

Paris asked:

How can a person define reality? I mean we think that we live in reality, but how can we be sure? Is there a universal reality, or do we each have our own personalized reality?

Paris also asked:

If there is a god and he created all things, wouldn't that also include evil and Satan? Which ultimately means god created evil.

Why do you want to define reality? Haven't you been bitten by a flea? Stumbled over a rock in your path? Spilt the milk? Now these are not philosophical answers and really quite trivial: but still, it needs saying every now and then that reality comes before theorising about it. How real reality is you can judge easily enough by noting that in your First Aid Kit you probably have dozens of medicines to help you with one or another ailment, for instance a toothache. How can you be sure a certain tablet will soothe your toothache? Because for all the talk about "personalised reality", "qualia" and so on, there is an overarching reality that comes first, and in this reality all people have well defined toothaches which can all be soothed within certain well understood tolerances.

As to your second question: if there is a God and this is really the issue to be solved first then yes, he created Satan. He also gave us knowledge of (not: created) evil, so that we should have some means of knowing what good is and strive for it. I think there is something for you to learn in this. People have been asking this question for at least 2000 years and never solved the problem. This is because the assumption has always been, how could God be so mindless as to subject us poor humans to evil? Now I might leave this little thought with you, but (if there is a God) do allow that this God may not be the simple minded, human-centred little garden god of suburban religiosity.

Jürgen Lawrenz

back

Ira asked:

My son is 13 and studied philosophy this summer at Johns Hopkins's CTY program, with an emphasis on existentialism. He's interested in reading more and has begun doing so, but I'm looking for a list of good introductory books (not too high but not too low) that would be appropriate for him, but without success. Could you provide me with such a list, or direct me to one?

He has read Faust, The Republic, Man's Search for Meaning, Singer's Practical Ethics, Waiting for Godot, Metamorphosis, and selections from Kant, Wittgenstein, Dostoyevsky. Perhaps he is ready in fact for an adult list, but I think an introductory list or one clearly geared towards the non-specialist would be best.

Good going for a 13-year old... you know, bizarre as this sounds, if he doesn't do this already, I'd recommend he read some science fiction as well. Try Neal Stephanson, Rudy Rucker, Arthur Clarke (the early books), the early Asimov, the early Benford... for a start. Good sci-fi takes on a lot of philosophical questions. Perhaps Heinlein also.

As far as philosophy goes, there's always Christina's World for an introduction to metaphysics, but it might be a bit childish for him. In addition, I'd highly recommend he start something like hard-core introductions to physics... again, Asimov comes to mind. I strongly believe that philosophy has to be tempered with data, and it's too easy to float off into never never land with some of that stuff. Perhaps he'd also enjoy The Double Helix. You might yourself take a look at some of the early novels (not the later works) of Colin Wilson to see what you think he should read... some of them are rough going, but they're fascinating philosophical novels. For him I'd at least recommend The Glass Cage and Necessary Doubt.

Steven Ravett Brown

back

Jean asked:

1. Contrasting "action" with "labor" and "work" what is the description for the nature of equality and freedom and the explanation of how these features condition according to Arendt in the interactive process?

2. What connection does Arendt see between the rise of Cartesian doubt and the triumph of the homo faber mentality within the modern world?

One of the great things about literacy is that people can write down their beliefs and have others read them. One of the really magnificent inventions in the history of humankind was the printing press, which made it possible for books not just to be written, but to be copied in identical form thousands and more of times. This means that debate and discussion become possible because people can refer to these identical copies of books, papers etc. It also means, of course, that in order to have such debates, one should actually read those writings. A great deal of trouble in discussion and debate results from people discussing writings they've not read ("ah: Joe Bloggs told me!"); and then you also find, quite frequently, overworked and underpaid students using primers to crib their "knowledge" for the sake of passing an exam, which of course means they know again only Joe Bloggs opinion. All this by way of informing you that Arendt has actually written a book of relatively modest dimensions, so that reading it would not take an inordinate amount of time to tell you and me all about the subject matter of your question. The book is called The Human Condition; it is extremely well written (she had an outstanding literary person as her editor) and easy to understand; and if this is what you are interested in, don't let yourself be fobbed off by second and third hand accounts, but get the book!

Jürgen Lawrenz

back

Gurnow asked:

What is the source for the Epicurus quotation: "Either God wants to abolish evil, and cannot; Or he can, but does not want to; Or he cannot and does not want to. If he wants to, but cannot, he is impotent. If he can, but does not want to, he is wicked. But, if God both can and wants to abolish evil, Then how come evil in the world?"

I have read through many collections of his works and cannot find this selection. It is often-quoted yet rarely cited.

Try this: David Hume, Dialogues Part X, para. 26. I believe that's where the exact quote comes from. I can't find a text by Epicurus which has this quote, and the fact that Hume attributes it to Epicurus doesn't mean that Hume found it either.

Steven Ravett Brown


A quick google search on (epicurus problem of evil) yields this web site, http://www.atholbooks.org/magazines/heresiarch/evil.html which offers this answer:

De Ira Dei. trans. Sr. M. F. McDonald O. P., The Catholic University of America Press, 1965, pp 91-93

Jonathan Ichikawa

back

Joe asked:

When ethical relativists preach tolerance aren't they suggesting that tolerance is the only ethical truth, therefore contradicting their own edict that there are indeed no universal ethical truths?

Yes, well... you're not the first to point this out, believe me. But a real dyed-in-the-wool relativist will simply say that they never claimed to be consistent; that in fact if they did, they couldn't be relativists. And we're off again. No, this one is in my opinion a very good point but one which does not weigh with the relativists, who are mostly, in my rather biased opinion, more concerned with adhering to contemporary notions of "political correctness" or some such than with doing clear and consistent philosophy.

Steven Ravett Brown

back

Joe asked:

What is history?

Some 2500 years before the present era, a fellow called Herodotos travelled around the eastern Mediterranean countries and collected what he could of peoples memoirs of notable events and began to write them down. The idea, he explained, what to prevent these things from being forgotten. Well, he was lucky: we still have his book to read now, so that many people are better informed about the World in 2500 BC than about what happened in their own backyard much more recently. He was luckier still in respect of his immortality in that his manner was immediately imitated and a rash of histories followed in his wake. The Romans carried on with this, and so has every western literate country ever since.

From this you get the idea that history is essentially story telling. Herodotus has been called The Father of History as well as the Father of Lies. There you have the gist of it. It is desirable, of course, that the stories you tell are or were true, but this has never been an enforceable rule. So you find one Henry Fielding (among many others) writing a "History" of Tom Jones which is plain fiction. I make this point because the fiction in this case is pretty obvious, but there are many histories throughout history that are also plain fictions, yet are taken to be history because they happen to be the only account of a particular event which we believe to have actually occurred and keep to it because there are no other witnesses. Then there is what's known as the "scissors and paste" history, which is the lazy historians alternative and consists of cutting up other peoples stories and gluing them back together in a different way. Sometimes the truth is a casualty in these exercises as well. Finally there is the "scientific" history, which is a bit like stamp collecting. If your field of interest is fairly old, this can be fun: there are so many churches, castles and homesteads, not to mention deserts, where scraps of old facts and stories are buried. But if your field is the 20th century, the "facts" on tap can be a little overwhelming and might need drastic trimming before you tell a story about them.

I suppose I should clarify the other meaning of the word "history" as well. Commonly this is understood as "that which actually happened in the past". But on account of the dubious literary legacy noted above, what we actually know of this "history" prior to Gutenberg is not worth fighting over. At any rate, in this sense, "history" is the great swathe of darkness over the human past, which is only irregularly lit by the odd reliable fact here and there. Owing to vested interest, most of this "history" is feudal and church history; what really happened to real people interested no-one and therefore we don't know. When I went to school, I still had to learn long lists of Kings and Queens and Popes, along with such maverick exceptions as Robin Hood. Once I asked my teacher if these were real people or just names invented to torment young students; and I pointed to examples in my history book where a medieval woodcut showed Alexander the Great and Pythagoras dressed in medieval style robes, while some modern French revolutionaries wore Roman togas etc. He said (words deserving of immortal fame): "If it wasn't for them, there would be nothing worth knowing!"

Jürgen Lawrenz

back

Verbiest asked:

Is there life after death?

If I said "no", would you believe me? If I said "yes", would you believe me? Why? Do you want a 25-word proof that there is or isn't? You do have some idea, I assume, of the amount written on this subject in the last, say, 3000 years?

Steven Ravett Brown


This is an age old question, which has to be answered relating it to two major concepts: God and Soul. First of all, God and Soul are not Christian concepts, but do exist in all major Eastern religions, so I use them only to be more easily understood. Secondly, this is not a question of a pure metaphysical nature. Sure it implies you do extrapolations based on knowledge you can't possess: the afterlife isn't exactly our own reality; but it also deals with plain physics as well. You live and you die. When you die, does all of "you" die? If not, there is a part of "you" that is immortal. This substance, the Soul, would continue living, or would start a "new life" on another plane of reality. Most religions base that other plane on the existence of God. So if you prove that God does not exist, than an afterlife would be impossible, as it depends on God. To describe all the theories trying to prove the existence of God would take too long, so I'll talk about a theory that tries to prove that the Soul does not exist, and one that does the opposite

1) Julien De La Mettrie (1709-1751) talked about something that is today a hot scientific issue: Soul and Body cannot be viewed as two separate things. Starting with Descartes work, La Mettrie writes about L'homme Machine, the machine-man that functions basically as a machine, and has no real metaphysical attributes that might outlive him. Today, eminent neurosurgeons argue that in fact the Soul is an integral part of the body, and is made of the social and emotional experiences that we have as human beings. And stating this, the Soul dies with the body.

2) You might think — and you would be right to do so — that the former do not answer with full certainty the question. Pascal's Wager argued by Blaise Pascal (1623-1662) then tells us that "the existence of God is a gamble in which we must wager our bet. Pure reason does not provide an answer, so if we bet on the existence of God we have everything to win and nothing to lose, even if we have but a very small chance to win. But if we bet against it, we can lose everything: eternal life"

Nuno Hipolito

back

Maxine asked:

What is the difference between a theistic and atheistic understanding of how the universe exists?

A theistic understanding of the universe requires the presence of a god or creator responsible for the creation and maintenance of the universe. The laws governing the universe are, therefore, accepted to be the laws laid down by the deity responsible for its creation. In such a creation all living things came into being at the 'beginning', hence there is no recognition of 'evolutionary' progress, in the Darwinian sense. However, a historical recognition of human development from a primitive position is accepted to be the case.

Theistic claims for the existence of the universe are generally anthropocentric, with progress of the human race seen as the primary objective, finally culminating in a vision of heaven or utopia where all will be revealed, and good will be shown to have prospered over evil. The several religions hold their varied views on how this will be achieved. In the meantime, though the earth is the 'Lord's', man, made in his image, is considered to be responsible for looking after His creation, including the management and care of all animals and plants, (protection of ecology and biodiversity throughout the world). Probably, the primitive religions, including sun, mountain and idol worshippers, where these objects are seen as gods, would fall into the theist category, although with some radical deviation.

Atheistic views of the universe have no spiritual content. They are usually materialistic in structure and progress is made by seemingly accidental events; although the whole takes place within what is regarded as 'natural law'. Control of events is sought through mathematical and scientific prediction, there is no appeal to a creator or a god. So far as origins are concerned, matter somehow 'appears' and becomes immediately subject to laws of physics and mathematics, or, to put it another way, the laws of physics and mathematics are intrinsic within the appearance of matter. Life arises from primitive material conditions, subject to the laws of chemistry and physics, and progresses by a series of fortuitous accidents, until, in the form of 'mind' it is able to consider its own historical development. However, the future in this case is the rather depressing concept that all will eventually return to whence it came. Wherever that happens to be!

John Brandon


Fundamentally the answer is that in theistic existence the question has to be asked, where does the universe come from? The assumption here is that matter is dead and with all our knowledge of science, we have never managed to explain why there is matter to begin with. If you see a house standing in a street, you assume that someone built it: the bricks didn't assemble themselves. If you see a living thing, whether animal or vegetable, you assume the pre-existence of some kind of seed, a "parent", for again the thing did not come into existence by itself. Continue this train of thought until you arrive at fundamental particles, but when you get there, the same question remains still unanswered. So theists suppose a creative agent (God). Without an act of creative will, nothing would exist at all.

Atheistic explanations do just the opposite. Here it is supposed that existence does not need an answer. For all we know, matter has always existed and the question does not arise why it should ever have been created. Since living things are also ultimately matter, the interest of atheists ends with the explanation that some matter is in possession of special properties which in the event of special conditions occurring (i.e. a special kind of environment such as prevails on earth) will give rise to life forms.

Some atheists nonetheless make the attempt to explain how the universe began without special creation. The Big Bang, the Steady State cosmology and the Quantum Flutter of quantum cosmology are some attempts to rationalise the idea that matter may have come from nothing, also in virtue of special conditions applying at some stage in the history of the whole universe. This special condition is named a "singularity" in the big bang theory, while in the steady state theory matter is assumed to be circulating persistently by its conversion into energy and vice versa.

In the end, therefore, the issue revolves around the alternative question of whether we think the universe had a beginning and has always existed and will continue to do so. The second alternative is not taken seriously in western philosophy, and in science we believe to have sufficient proof that "eternity" is not a realistic notion. Theists exempt God from this restriction: God is actually eternal, outside of time. Atheists deny this and say that time came into being with creation; but some theorists maintain that in effect (due to time retardation on the approach to a singularity) the universe may be eternal after all. This last point is a bit difficult to understand, but if you can accept that time is perhaps nothing other than our creature experience of the periodicity of natural phenomena, then you may comprehend that in the subatomic realm, time does not in fact exist and the whole issue of time disappears.

There is a paper by Adolf Grunbaum entitled "The Poverty of Theistic Cosmology" that you can access in the Stanford Online Encyclopedia of Philosophy. Grunbaum obviously defends the atheistic point of view, but in the process discusses in depth the other sides arguments. Be warned, however, that this is an exceedingly difficult and technical paper. Unless you've got a good working knowledge of theoretical physics, you may not get a lot out of it.

Jürgen Lawrenz

back

Ivia asked:

Do you think an effect found by experiment is made by a scientist and not by nature itself?

Offhand, I can't think of anything that's been done by scientists, or anyone else, for that matter, which doesn't occur "naturally", from nuclear fusion to the development of drugs. I suppose you might argue that if we do something like create organisms which use DNA that has different base pairs than occur naturally on Earth that this hasn't happened "naturally", but really, we don't know that this hasn't somewhere else in the universe.

Let's look at this more generally. First, to contrast "scientists" and "nature" begs the question of what is natural. Human beings evolved naturally on this planet, and we're just as much a part of the ecosystem, etc., as any other creature on it. So there's nothing at all we can do which nature is not doing, since we're part of nature, like it or not. Second, it's a big universe out there, and whatever manipulations we do are really just organizations of the environment, in effect, which have some likelihood of happening in other places as well. Third, what scientists do is not so much create "new" effects as attempt to isolate causes of effects, in order to find out what they are without interference form extraneous factors. This is termed a "controlled" experiment. We attempt to control the uncertainty in the factors producing an effect by having it happen in very restricted situations: isolating its causes, in order to get a handle on what actually produces that effect in much more chaotic situations.

So it's not really that scientists produce new things so much as they pare down what produces old ones in order to find the essential causes. Then when those are found (if they are and/ or can be), we can play around with those causes, knowing what they are, in order to find out exactly how their effects result.

Steven Ravett Brown

back

Robert asked:

Can "Philosophy" prove absolutely that God exists and that God is greater than everything that exists? My father studied divinity — winning several prizes for his studies — and once when I was a child he confused my dim mind with a simple logical statement — a 3 prong argument that could logically prove God existed based upon the assumption that nothing greater than God can exist and something along the lines of what was greater — that what exists or that which does not? However, I am probably confused in my recollection.

Can anyone help me to elaborate on this theme as I was very taken with its pleasing simplicity.

What you're enquiring about is known as the "ontological proof". The one your father told you about sounds like the argument from perfection.

Step 1: I'm not perfect. Neither are you, nor anyone else I know of. Step 2: But I can imagine someone much better, more perfect than myself, Indeed if I stretch my imagination, I can think of a Being so perfect that whatever faults I find in myself and in other humans, this Being does not have them. Further whatever my limitations or those of humans in general, this Being would have powers such that I can only think of, but never possess. Now a Being of this order of perfection is unlikely to be human, so let me call this Being "God". Step 3: One problem remains: Although I can imagine such a Being, it is after all only my imagination, yes? Well, if that's the case, then this perfect Being lacks the one perfection that is the most crucial of all, namely existence. But surely if I can conceive of such perfection then it cannot be unreal: where would my mind get this notion of perfection from if not from the fact that such a Being exists? Therefore such a Being, which has all perfections I can think of, and which obviously includes the perfection of existence, must necessarily exist. Therefore God exists. QED.

The only problem with this proof is that you can "prove" the existence of a fish god the same way. But your father probably did not tell you about this little impairment!?

Jürgen Lawrenz

back

Courtenay asked:

Geoffrey Klempner offered the following thesis:

I have an objection to the definition of God as 'all knowing'. I'll leave you to consider whether or not you think that it is convincing. Being all knowing, God sees things from every point of view, including yours and mine. He knows what it is like to be you, and he knows what it is like to be me. But it seems to me that I know something God does not, and cannot know. What God knows is only what things are like for someone satisfying my total description. He knows, for example, what it is like to be struggling with this question. But what God cannot know is what it is like for the individual satisfying that total description to be I. From God's point of view, every individual is 'I'. From my point of view, only one is (Answers 5).

Please comment on this response from Paul Monfils a Catholic Deacon:

The statement "what God cannot know is what it is like for the individual satisfying that total description to be I" would be more accurately restated: "from my limited perspective, based upon nothing more than my limited human intellect, it seems to me that God could not know what it is like for the individual satisfying that total description to be I". Or to put it another way, "using nothing more than my flawed and finite human nature, I am incapable of comprehending how God could possibly know what it is like for the individual satisfying that total description to be I". Stated in this way it is apparent that no limitation on the part of God is suggested, but rather the sheer futility of a creature's efforts to understand his Creator; of the finite seeking to comprehend the infinite by finite means. Any statement which begins "what God cannot know" presumes far too much — that I can validly equate my finite logical perception of God's capacity to know with His actual infinite capacity to know.

Once this philosophical quandary is stated more accurately, as my inability to comprehend how God could have access to particular knowledge, the answer is obvious — "of course you can't understand how God could know this, but you can nevertheless know with certainty that God does know, because He has revealed His infinite and omniscient nature, and knows ALL, which would necessarily include everything which you are incapable of comprehending His capacity to know". Not only does God know "what it is like to be I", but He knows it perfectly, while I myself only know it imperfectly.

I'm afraid I don't know much of God, but do believe knowledge to be a human concept about a particular animal relation to the world which implies the possibility of false belief and mistake. Mistake and false belief wouldn't apply to God, as you see him.

As far as I understand anything about God, God is not an animal in relation to the world and so you shouldn't be thinking of what God "knows".

Rachel Browne


Given what the good Deacon says I'd like to ask him why I should follow his faith? As he points out he hasan imperfect knowledge of his Creator so why should I believe his explanation?

Kim Boley


First, let me say for clarity that I agree with this thesis. One general comment on the comment of Paul Monfils, certainly not meant personal.. I know that language is personal, but using so many difficult phrases cannot be 'good'. It in fact hides his opinion more, than opening it up.

First it should be clear what either of them means when using the word 'God'.

1. Obviously for Geoffrey Klempner the notion 'God' is relative. 2. On first sight Paul uses an enlightened absolute view on the notion 'God'

Consequently Geoffrey's god is personal, and knows only one I, that of Geoffrey. Because in this view everybody creates his/her own image of 'god' such gods know only the different 'I's.

In the same way Paul's god is absolute. In the Enlightenment this was called a priori. Such a god per definition knows truth (his own single truth). And as a consequence if his/her followers use this truth then per definition he/she knows their truth.

So whatever words they use, both Geoffrey and Paul are right, ONLY they apply different rules. IF they want to communicate, than they should compare their parallel rules (their 2 logics), and find a mutual set of basic rules. That might involve that both should slightly change their own set. But Im convinced the differences are not that big. It would be a nice and educational exercise. Because I think both views need each other, and therefore should come to real communication, instead of risking remaining to parallel paradigms

Henk Tuten


I am perfectly happy to preface every statement I make in the context of philosophical discourse (including this one) with the disclaimer, "From my limited perspective, based upon nothing more than my limited human intellect it seems to me that...". That used to be the accepted literary style for philosophers, e.g. Augustine in the Confessions. "O God, I know I am a complete ignoramus, and you understand things infinitely better than I do, but might it possibly be the case that...?", and so on. But such self-effacing language did not prevent Augustine from doing ground-breaking work on the philosophy of time.

A disclaimer which applies equally to every statement one makes has no force whatsoever.

There appears to be a special problem with philosophical arguments which rule out the possibility of something. Isn't it harder to prove a negative? It is hard to prove that there are no unicorns, because this would require investigating every place in the universe that unicorns might be. However, in philosophy, we prove things by means of logical argument. (Again, it is our best judgement concerning what is "logical" or a "proof", but this goes without saying.) And there can be logical arguments to the effect that "such-and-such is an impossibility".

My argument concerns a definition of God's 'omniscience' offered by a philosopher. On a certain, plausible view of 'omniscience', I argue, an omniscient God cannot know the difference between my existence and G.K.s existence. The argument may be considered as a challenge. Of course, the theologian is free to say that if and when philosophy fails us, when it cannot meet the challenges set to it in the way that we would like, there is still room for faith. (That sounds close to Kant's view, although Kant held something stronger: that it is the task of his philosophy to demonstrate the limits of reason, in order to make room for faith.) However, one should never be complacent about the fact that one does not understand something. Especially in this case, where — at least in the Judeo-Christian tradition — it is a matter of central importance that God knows me as this unique I. That is why those who accept the comforts of religion have no reason to scorn philosophy.

Geoffrey Klempner